Download as pdf or txt
Download as pdf or txt
You are on page 1of 100

केन्द्रीय विद्यालय संगठन

KENDRIYA VIDYALAYA SANGATHAN


जयपुर संभाग / JAIPUR REGION

STUDY MATERIAL FOR TERM -II (2021-22)


CLASS XII MATHEMATICS

CHIEF PATRON PATRON


Sh. B.L. MORODIA Sh. D R MEENA
DEPUTY COMMISSIONER ASSISTANT COMMISSIONER

COORDINATOR
Sh RAJESH KANTHARIA
PRINCIPAL, KV NO-3, JAIPUR
1
CONTENT TEAM MEMBERS
SN NAME OF PGT NAME OF KV
1 Sh RAJEEV KUMAR GUPTA KV, No-5, JAIPUR
2 Sh SAMEER SAXENA KV, No-1, KOTA
3 Sh PREM SINGH SHEKHAWAT KV, No-1, ALWAR
4 Dr RAJENDRA VADHWANI KV, No-2, JAIPUR
5 Sh HEM SINGH KV, BHARATPUR
6 Dr B S INDOLIA KV, No-3, JAIPUR
7 Sh VINOD KUMAR KV, No-1, AJMER
8 Sh MAHESH KUMAR GUPTA KV, JHALAWAR
9 Sh SURESH KUMAR YADAV KV, CHURU
10 Sh MUKESH KUMAR KV, No-4, JAIPUR

INDEX
SN TOPICS/ CHAPTER PAGES
1 SYLLABUS TERM II 3
2 INDEFINITE INTEGRATION 4 to 17
3 DEFINITE INTEGRATION 18 to 27
4 APPLICATION OF INTEGRATION 28 to 32
5 DIFFERENTIAL EQUATIONS 33 to 41
6 VECTORS 42 to 50
7 THREE-DIMENSIONAL GEOMETRY 51 to 63
8 PROBABILITY 64 to 74
9 SAMPLE QUESTION PAPER-1 75 to 80
10 SAMPLE QUESTION PAPER-2 81 to 88
11 SAMPLE QUESTION PAPER-3 89 to 92
12 SAMPLE QUESTION PAPER-4 93 to 96
13 SAMPLE QUESTION PAPER-5 97 to 100

2
SYLLABUS TERM II (2021-22)
No. UNITS MARKS
III CALCULUS 18
IV VECTORS AND THREE- DIMENSIONAL GEOMETRY 14
VI PROBABILITY 8
TOTAL 40
INTERNAL ASSESSMENT 10
TOTAL 50

Unit-III: Calculus
1. Integrals : Integration as inverse process of differentiation. Integration of a variety of
functions by substitution, by partial fractions and by parts, Evaluation of simple integrals
of the following types and problems based on them.
dx dx dx dx dx dx
∫ a2+ x2 ∫ a2 − x2 ∫ x2− a2 ∫ √a2− x2 ∫ √a2+ x2 ∫ √x2− a2
∫ √a2 − x 2 𝑑𝑥 ∫ √x 2 + a2 𝑑𝑥 ∫ √x 2 − a2 𝑑𝑥
Fundamental Theorem of Calculus (without proof). Basic properties of definite integrals and
evaluation of definite integrals.
2. Applications of the Integrals: Applications in finding the area under simple curves,
especially lines, parabolas; area of circles /ellipses (in standard form only) (the region
should be clearly identifiable).
3. Differential Equations: Definition, order and degree, general and particular solutions of
a differential equation. Solution of differential equations by method of separation of
variables, solutions of homogeneous differential equations of first order and first degree
of the type: 𝑑𝑦/ 𝑑𝑥 = 𝑓(y/x). Solutions of linear differential equation of the type: dy /dx +
py = q, where p and q are functions of x or constant
Unit-IV: Vectors and Three-Dimensional Geometry
1. Vectors : Vectors and scalars, magnitude and direction of a vector. Direction cosines and
direction ratios of a vector. Types of vectors (equal, unit, zero, parallel and collinear
vectors), position vector of a point, negative of a vector, components of a vector, addition of
vectors, multiplication of a vector by a scalar, position vector of a point dividing a line
segment in a given ratio. Definition, Geometrical Interpretation, properties and application
of scalar (dot) product of vectors, vector (cross) product of vectors.
2. Three - dimensional Geometry: Direction cosines and direction ratios of a line joining
two points. Cartesian equation and vector equation of a line, coplanar and skew lines,
shortest distance between two lines. Cartesian and vector equation of a plane. Distance of a
point from a plane.
Unit-VI: Probability
1. Probability: Conditional probability, multiplication theorem on probability, independent
events, total probability, Bayes’ theorem, Random variable and its probability distribution.

3
INDEFINITE INTEGRALS
INTEGRATION: Integration is an inverse process of differentiation.
Types of Integrals: Indefinite Integrals
Definite Integrals
INDEFINITE INTEGRALS

Important Points of Indefinite Integrals:


• Integration as an Inverse Process of Differentiation.
• Integration by Substitution
• Integration by using trigonometric identities
• Integration by Partial fraction
• Integration by Parts
• Integration of some standard types of functions

Integration as an Inverse Process of Differentiation-


𝑑
Let F(x) and f(x) are functions such that {𝐹(𝑥)} = f(x), then F(x) is called Integral of f(x) or
𝑑𝑥
anti-derivative or Indefinite integral.
Indefinite integral represents family of curves of a function.

∵ 𝑑𝑥𝑑 {𝐹(𝑥)} = f(x)


𝑑
{𝐹(𝑥) + 1} = f(x)
𝑑𝑥
𝑑
{𝐹(𝑥) + 2} = f(x)
𝑑𝑥
𝑑
Similarly 𝑑𝑥 {𝐹(𝑥) + 𝐶} = f(x)
Thus, 𝐹(𝑥) + 𝐶 is also an anti-derivative (Indefinite Integral) of f(x), Where C is an arbitrary
constant and it is called constant of Integral.

Symbol of Integration-∫ indicates the sign of integration.

Constant of integration (C) should be written for indefinite integrals.


Important table-

Symbol/ Term/Phrases Meaning


Integral of f with respect to x
∫ 𝑓(𝑥)𝑑𝑥
f(x) in ∫ 𝑓(𝑥)𝑑𝑥 Integrand
Integration The Process of finding the integral
C Constant of integration (any real number)

4
Integration of a function f (x) with respect to x then we write I = ∫ 𝒇(𝒙)𝒅𝒙
Example: Find anti-derivative/ integral of ‘sin x’ with respect to x?
Solution:-
∵ given function f(x) = sinx, I = ∫ 𝑠𝑖𝑛𝑥 𝑑𝑥 = - cosx + C
Some Important Formulae:

S. N. Derivatives Indefinite Integrals/Anti-


derivatives
1 𝑑 𝑥 𝑛 +1 𝑥 𝑛+1
( ) = 𝑥 𝑛 , Where n ≠ 1 ∫ 𝑥 𝑛 𝑑𝑥 = + C, where n ≠ 1
𝑑𝑥 𝑛+1 𝑛+1

2 𝑑
(𝑥) = 1 ∫ 𝑑𝑥 = x + c
𝑑𝑥

3 𝑑
(sin 𝑥) = cosx ∫ 𝑐𝑜𝑠𝑥 𝑑𝑥= sin x + C
𝑑𝑥

4 𝑑
(−𝑐𝑜𝑠𝑥) = sinx ∫ 𝑠𝑖𝑛𝑥 𝑑𝑥 = - cos x + C
𝑑𝑥

5 𝑑
(𝑡𝑎𝑛𝑥) = 𝑠𝑒𝑐 2 𝑥 ∫ 𝑠𝑒𝑐 2 𝑥 𝑑𝑥 = tan x + C
𝑑𝑥

6 𝑑
(−𝑐𝑜𝑡𝑥) = 𝑐𝑜𝑠𝑒𝑐 2 𝑥 ∫ 𝑐𝑜𝑠𝑒𝑐 2 𝑥 𝑑𝑥 = -cot x + C
𝑑𝑥

7 𝑑
(𝑠𝑒𝑐𝑥) = sec x tanx ∫ sec x tanx𝑑𝑥 = sec x + C
𝑑𝑥

8 𝑑
(−𝑐𝑜𝑠𝑒𝑐𝑥) = cosecxcotx ∫ cosecx cotx𝑑𝑥 = −𝑐𝑜𝑠𝑒𝑐𝑥 + C
𝑑𝑥

9 𝑑 1 1
𝑑𝑥
(𝑠𝑖𝑛−1 𝑥) =
√1− 𝑥 2
∫ √1− 𝑥 2 𝑑𝑥 = 𝑠𝑖𝑛−1 𝑥 + C
10 𝑑 1 1
𝑑𝑥
(−𝑐𝑜𝑠 −1 𝑥) =
√1− 𝑥 2
∫ √1− 𝑥 2 𝑑𝑥 = −𝑐𝑜𝑠 −1 𝑥 + C
11 𝑑 1 1
𝑑𝑥
(𝑡𝑎𝑛−1 𝑥) =
1+ 𝑥 2
∫ 1+ 𝑥 2 𝑑𝑥 = 𝑡𝑎𝑛−1 𝑥 + C
12 𝑑 1 1
𝑑𝑥
(−𝑐𝑜𝑡 −1 𝑥) =
1+ 𝑥 2
∫ 1+ 𝑥 2 𝑑𝑥 = −𝑐𝑜𝑡 −1 𝑥 + C
13 𝑑 1 1
𝑑𝑥
(𝑠𝑒𝑐 −1 𝑥) =
|𝑥|√𝑥 2 −1
∫ | 𝑥 |√𝑥 2−1 𝑑𝑥 = 𝑠𝑒𝑐 −1 𝑥 + C

14 𝑑 1 1
𝑑𝑥
(−𝑐𝑜𝑠𝑒𝑐 −1 𝑥) =
|𝑥|√𝑥 2 −1
∫ |𝑥|√𝑥 2−1 𝑑𝑥 = −𝑐𝑜𝑠𝑒𝑐 −1 𝑥 + C

15 𝑑
(𝑒 𝑥 ) = 𝑒 𝑥 ∫ 𝑒 𝑥 𝑑𝑥 = 𝑒 𝑥 + C
𝑑𝑥

16 𝑑 𝑎𝑥 𝑎𝑥
( ) = 𝑎 𝑥 , where a > 0, a ≠ 1 ∫ 𝑎 𝑥 𝑑𝑥 = +C
𝑑𝑥 𝑙𝑜𝑔𝑒 𝑎 𝑙𝑜𝑔𝑒 𝑎

5
17 𝑑 1 1
𝑑𝑥
(log |𝑥|)} =
𝑥
∫ 𝑥 𝑑𝑥 = log|x| + C
18 𝑑 1 1 1 1
𝑑𝑥
(− ) =
𝑥 𝑥2
∫ 𝑥 2 𝑑𝑥 = − 𝑥 + C
19 𝑑 1 1
𝑑𝑥
(√𝑥) =
2√𝑥
∫ √𝑥 𝑑𝑥 = 2√𝑥 + C

Properties of Indefinite Integrals

• ∫ 𝑑𝑥 = x + C

• ∫ 𝑘 𝑓(𝑥)𝑑𝑥 = 𝑘 ∫ 𝑓(𝑥)𝑑𝑥 + C

• ∫{𝑓(𝑥) + 𝑔(𝑥)}𝑑𝑥 = ∫ 𝑓(𝑥)𝑑𝑥 + ∫ 𝑔(𝑥)𝑑𝑥 + C


∫{𝑓(𝑥) − 𝑔(𝑥)}𝑑𝑥 = ∫ 𝑓(𝑥)𝑑𝑥 - ∫ 𝑔(𝑥)𝑑𝑥 + C
Note: Only one constant of Integration is used for integral of more than one functions.

Some Important Examples:


Evaluate the following:
1 𝑐𝑜𝑠2𝑥 +2𝑠𝑖𝑛2 𝑥 𝑥 3 − 𝑥 2 +𝑥−1
(i)∫ 𝑥 2 (1 − ) 𝑑𝑥 (ii)∫ 𝑑𝑥 (iii)∫ 𝑑𝑥
𝑥2 𝑐𝑜𝑠2 𝑥 𝑥−1
1 𝑥 2+1 1
Solutions: (i) I = ∫ 𝑥 2 (1 − ) 𝑑𝑥= ∫(𝑥 2 − 1)𝑑𝑥 = ∫ 𝑥 2 𝑑𝑥-∫ 𝑑𝑥 = 2+1 - x +C =3 x3 - x + C
𝑥2
𝑐𝑜𝑠2𝑥 +2𝑠𝑖𝑛2 𝑥 1−2𝑠𝑖𝑛2 𝑥 +2𝑠𝑖𝑛2 𝑥 1
(ii) I = ∫ 𝑑𝑥 = ∫ 𝑑𝑥 = ∫ 𝑐𝑜𝑠2 𝑥 𝑑𝑥 = ∫ 𝑠𝑒𝑐 2 𝑥 𝑑𝑥 = tan x + C
𝑐𝑜𝑠2 𝑥 𝑐𝑜𝑠2 𝑥

𝑥 3 − 𝑥 2 +𝑥−1 (𝑥−1)(𝑥 2 + 1) 1
(iii) I = ∫ 𝑑𝑥 = ∫ 𝑑𝑥 = ∫(𝑥 2 + 1)𝑑𝑥 = 3 x3 + x + C
𝑥−1 𝑥−1

Integration by substitution: -
The method of reducing a given integral into one of the standard integrals by a proper
substitution is called method of substitution
• Substitution Method is used to find integration of composite functions.
Like as: sin3x, 𝑒 𝑠𝑖𝑛𝑥 , sin (logx), √𝑥 + 1 ……….etc.
• In composite function x is replaced by another function.
• Generally, substitution method is used if differentiation is given along with function.
• Procedure to find integral by substitution method-
Let I = ∫ 𝑓{𝑔(𝑥)}. 𝑔′ (𝑥)𝑑𝑥
Step-I Substitute g(x) = t,
Step- II Differentiate both sides; so that g’(x) dx = dt

6
Step-III Now I = ∫ 𝑓{𝑡} 𝑑𝑡 = F(t) + C = F{g(x)] + C

• Integration of tanx, cotx, secx and cosecx :


(i) ∫ 𝑡𝑎𝑛𝑥 𝑑𝑥 = - log |cos x| + C or log|secx| + C
(ii) ∫ 𝑐𝑜𝑡𝑥 𝑑𝑥 = log|sinx| + C or - log|cosecx| + C
𝜋 𝑥
(iii) ∫ 𝑠𝑒𝑐𝑥 𝑑𝑥 = log|secx + tanx| + C or log|tan(4 + )|+ C
2
𝑥
(iv) ∫ 𝑐𝑜𝑠𝑒𝑐𝑥 𝑑𝑥 = log| cosecx – cotx| + C = log|tan2|+ C

Some important Deductions:

𝟏
The following integrals can be derived by substitution (ax +b)= t⇒ dx = dt
𝒂
(𝑎𝑥+𝑏)𝑛+1 1
𝑛
1. ∫(𝑎𝑥 + 𝑏) 𝑑𝑥= + C, Where n ≠ 1 9. ∫ 𝑒 (𝑎𝑥+𝑏) 𝑑𝑥 = 𝑎 𝑒 (𝑎𝑥+𝑏) + C
𝑎(𝑛+1)
2.∫ 𝑎𝑑𝑥 = ax + c 𝑎𝑝𝑥+𝑞
10. ∫ 𝑎𝑝𝑥+𝑞 𝑑𝑥 = 𝑝(𝑙𝑜𝑔𝑎) + C
1 1
3. ∫ cos (𝑎𝑥 + 𝑏) 𝑑𝑥= 𝑎sin(a x+b) + C 11. ∫ 𝑥 𝑑𝑥 = log |𝑥| + C
1 1
4. ∫ sin (𝑎𝑥 + 𝑏) 𝑑𝑥 = - 𝑎cos(ax+b) + C 12. ∫ tan(𝑎𝑥 + 𝑏) 𝑑𝑥= 𝑎log|sec(ax+b)| + C
1 1
5. ∫ 𝑠𝑒𝑐 2 (𝑎𝑥 + 𝑏)𝑑𝑥 = 𝑎tan (ax+b) + C 13. ∫ cot (𝑎𝑥 + 𝑏) 𝑑𝑥= 𝑎log |sin(a x+b)| + C
1
6. ∫ 𝑐𝑜𝑠𝑒𝑐 2 (𝑎𝑥 + 𝑏) 𝑑𝑥 = - 𝑎cot (ax+b) + C 14. ∫ sec(𝑎𝑥 + 𝑏) 𝑑𝑥
1
=𝑎{log|sec(a x+b)+ tan(ax+b)|} + C
1
7. ∫ sec (ax + b) tan(ax + b)𝑑𝑥 = 𝑎sec( ax +b)+ C 15. ∫ cosec(𝑎𝑥 + 𝑏) 𝑑𝑥
1
= 𝑎{log|cosec(a x+b)- cot(ax+b)|} + C

8. ∫ cosec(ax + b) cot (ax + b)𝑑𝑥 16. ∫ √𝑎2


1
𝑑𝑥 = 𝑠𝑖𝑛−1 𝑎 + C
𝑥
1 − 𝑥2
= − 𝑎 𝑐𝑜𝑠𝑒𝑐(𝑎𝑥 + 𝑏) + C

Important Examples: Evaluate the following:


𝑠𝑖𝑛2𝑥 2𝑥 𝑥
(i)∫ 𝑑𝑥 (ii) ∫ 𝑡𝑎𝑛8 𝑥 𝑠𝑒𝑐 4 𝑥 𝑑𝑥 (iii) ∫ 22 22 2𝑥 𝑑𝑥
√4− 𝑐𝑜𝑠 4 𝑥
𝑠𝑖𝑛2𝑥 𝑠𝑖𝑛2𝑥
Solutions (i) I = ∫ 𝑑𝑥 = ∫ 𝑑𝑥
√4− 𝑐𝑜𝑠 4 𝑥 √22 −(𝑐𝑜𝑠 2 𝑥)2

Let 𝑐𝑜𝑠 2 𝑥 = t ⇒ - 2sinx cos dx = dt ⇒ sin2x dx = -dt


𝑐𝑜𝑠 2 𝑥
= - sin-1 + 𝐶 = - sin-1(
−𝑑𝑡 𝑡
Now I = ∫ )+ 𝐶
√22 − (𝑡) 2 2 2

(ii) I = ∫ 𝑡𝑎𝑛8 𝑥 𝑠𝑒𝑐 4 𝑥 𝑑𝑥 = ∫ 𝑡𝑎𝑛8 𝑥 (𝑡𝑎𝑛2 𝑥 + 1) 𝑠𝑒𝑐 2 𝑥 𝑑𝑥


= ∫ 𝑡𝑎𝑛10 𝑥𝑠𝑒𝑐 2 𝑥 𝑑𝑥 + ∫ 𝑡𝑎𝑛8 𝑥 𝑠𝑒𝑐 2 𝑥 𝑑𝑥
7
Now put tan x = t ⇒𝑠𝑒𝑐 2 𝑥 𝑑𝑥 = dt
𝑡 11 𝑡9 𝑡𝑎𝑛11 𝑥 𝑡𝑎𝑛9 𝑥
= ∫ 𝑡 10 𝑑𝑡 + ∫ 𝑡 8 𝑑𝑡 ⇒ + +C= + +C
11 9 11 9
2𝑥 𝑥
(iii) I = ∫ 22 22 2𝑥 𝑑𝑥
𝑥 𝑥 𝑥 𝑑𝑡
Now put 22 = t ⇒22 2𝑥 (𝑙𝑜𝑔2)2 𝑑𝑥 = 𝑑𝑡⇒22 2𝑥 𝑑𝑥 =
(𝑙𝑜𝑔2)2
2𝑥
2𝑡 𝑑𝑡 2𝑡 22
=∫ ⇒ +C⇒ +C
(𝑙𝑜𝑔2)2 (𝑙𝑜𝑔2)3 (𝑙𝑜𝑔2)3

Integration by using trigonometric identities: -

Some important trigonometric identities to find integrations


2 sinAcosB = sin(A+B) + sin(A-B) Cos2A = 1 - 2 sin2A ⇒ Sin2 A =
1−cos 2𝐴
2

2 cosASinB = Sin(A+B) – Sin(A-B) Cos2A = 2 cos2A – 1 ⇒ cos2A =


1+cos 2𝐴
2
2 cosAcosB = cos(A-B) + cos(A+B) sin3A = 3 sinA - 4 𝑠𝑖𝑛3 𝐴 ⇒ Sin3 A =
3𝑠𝑖𝑛𝐴 −𝑠𝑖𝑛3𝐴
4

2 sinASinB = cos(A-B) – cos(A+B) cos3A = 4 𝑐𝑜𝑠 3 𝐴 − 3 𝑐𝑜𝑠𝐴⇒Cos3 A =


cos 3𝐴 +3𝑐𝑜𝑠𝐴
4
𝐶+𝐷 𝐶−𝐷 ∫ 𝑠𝑖𝑛 𝑥 𝑐𝑜𝑠 𝑥 𝑑𝑥 , If p is odd then put sinx = t or q
𝑝 𝑞
sinC + sinD = 2 sin cos
2 2 is odd then put cosx = t
𝐶+𝐷 𝐶−𝐷 ∫ 𝑠𝑖𝑛𝑝 𝑥 𝑐𝑜𝑠 𝑞 𝑥 𝑑𝑥 , If p + q is a negative even
sinC - sinD = 2 cos sin
2 2 integer then divide numerator and denominator
by coskx and convert in term of tanx and sec2x,
put tanx = t
𝐶+𝐷 𝐶−𝐷 ∫ 𝑡𝑎𝑛𝑝 𝑥 𝑠𝑒𝑐 𝑞 𝑥 𝑑𝑥 If p and q are even positive
Cos C + Cos D = 2 cos cos
2 2 integers then put tan x = t

𝐶+𝐷 𝐶−𝐷 ∫ 𝑡𝑎𝑛𝑝 𝑥 𝑠𝑒𝑐 𝑞 𝑥 𝑑𝑥 If p and q are odd positive


Cos C - Cos D = - 2 sin sin integers then convert in term of secx and
2 2
(secxtanx) then put secx = t

Some Important Examples: Solve the following:


(𝑐𝑜𝑠5𝑥+𝑐𝑜𝑠4𝑥)𝑑𝑥 𝑡𝑎𝑛3 𝑥 𝑐𝑜𝑠2𝑥−𝑐𝑜𝑠2𝛼
(i)∫ (ii) ∫ 3 𝑑𝑥 (iii) ∫ 𝑑𝑥
1−2 𝑐𝑜𝑠3𝑥 𝑐𝑜𝑠 𝑥 𝑐𝑜𝑠𝑥−𝑐𝑜𝑠𝛼
9𝑥 𝑥
(𝑐𝑜𝑠5𝑥+𝑐𝑜𝑠4𝑥)𝑑𝑥 (2 cos cos )𝑑𝑥
Solutions:(i)I = ∫ =∫ 2
2 3𝑥
2
(using cosC +CosD =
1−2 𝑐𝑜𝑠3𝑥 1−2 (2 𝑐𝑜𝑠 – 1)
2

𝐶+𝐷 𝐶−𝐷
2 cos cos and Cos2A = 2 cos2A – 1 )
2 2
9𝑥 𝑥 9𝑥 𝑥 3𝑥
(2 cos cos )𝑑𝑥 2 cos cos cos 𝑑𝑥 3𝑥
=∫ 2 2
3𝑥 =-∫ 2
3𝑥
2 2
3𝑥 (multiply and divide by cos )
3−4 𝑐𝑜𝑠 2 4 𝑐𝑜𝑠 3 −3 cos 2
2 2 2

8
9𝑥 𝑥 3𝑥
2 cos cos cos 𝑑𝑥
=- ∫ 2 2
3𝑥
2
(using cos3A = 4 𝑐𝑜𝑠 3 𝐴 − 3 𝑐𝑜𝑠𝐴)
cos 3 ( )
2
𝑥 3𝑥
=-∫ 2 cos 2 cos 2
𝑑𝑥 {Using 2 cosAcosB = cos(A-B) + cos(A+B)}
1
=- ∫(cos 2𝑥 + cos 𝑥) 𝑑𝑥 = - 2 sin 2𝑥 − sin 𝑥 + C
𝑡𝑎𝑛3 𝑥
(ii) I = ∫ 𝑐𝑜𝑠3 𝑥 𝑑𝑥 = ∫ 𝑡𝑎𝑛3 𝑥 𝑠𝑒𝑐 3 𝑥𝑑𝑥 = ∫ 𝑡𝑎𝑛2 𝑥 𝑠𝑒𝑐 2 𝑥 (𝑠𝑒𝑐𝑥 tan 𝑥) 𝑑𝑥

= ∫(𝑠𝑒𝑐 2 𝑥 − 1)𝑠𝑒𝑐 2 𝑥 (𝑠𝑒𝑐𝑥 tan 𝑥) 𝑑𝑥= ∫(𝑠𝑒𝑐 4 𝑥 − 𝑠𝑒𝑐 2 𝑥) (𝑠𝑒𝑐𝑥 tan 𝑥) 𝑑𝑥


Now put secx = t ⇒ secxtanx dx = dt
1 5 1 3 1 1
= ∫(𝑡 4 − 𝑡 2 )𝑑𝑡 = 𝑡 − 𝑡 + C = 𝑠𝑒𝑐 5 𝑥 − 𝑠𝑒𝑐 3 𝑥 + C
5 3 5 3
𝑐𝑜𝑠2𝑥−𝑐𝑜𝑠2𝛼 2 cos2 x – 1 −( 2cos2 α – 1) 2 cos2 x – 2cos2 α
(iii) I = ∫ 𝑑𝑥= ∫ 𝑑𝑥 = ∫ 𝑑𝑥
𝑐𝑜𝑠𝑥−𝑐𝑜𝑠𝛼 𝑐𝑜𝑠𝑥−𝑐𝑜𝑠𝛼 𝑐𝑜𝑠𝑥−𝑐𝑜𝑠𝛼
2 (𝑐𝑜𝑠+𝑐𝑜𝑠𝛼)(𝑐𝑜𝑠𝑥−𝑐𝑜𝑠𝛼)
=∫ 𝑑𝑥 = 2 ∫(cos 𝑥 + cos 𝛼) 𝑑𝑥 = 2 sin x + 2 x cos 𝛼 + C
𝑐𝑜𝑠𝑥−𝑐𝑜𝑠𝛼

Some Standard Integrals: -

dx 1 𝑥 dx 1 𝑎+𝑥 dx 1 𝑥−𝑎
1. ∫ a2 + x2 = 𝑎 𝑡𝑎𝑛−1 𝑎 + 2. ∫ a2 − x2 = 2𝑎 log | 𝑎−𝑥 |+ C 3. ∫ x2 − a2 = 2𝑎 log |𝑥+𝑎|+ C
C
dx 𝑥 dx dx
4. ∫ √a2 2 = 𝑠𝑖𝑛−1 𝑎 + 5. ∫ √a2 2 = log | x + √a2 + x 2 | + C 6. ∫ √x2 2 = log | x + √x 2 − a2 | + C
−x +x −a
C
dx
7. ∫ For finding the integral, make perfect square the quadratic polynomial ax 2 + bx + c , using
ax2 + bx+c
the following identities (i) a2 + 2ab + b2 = (a + b)2 (ii) a2 - 2ab + b2 = (a − b)2
• To make perfect square the coefficient of x 2 should be + 1.
• After making perfect square, integrand will be changed in any one form given below.

dx dx dx
∫ or ∫ or ∫
A2 + X2 A2 − X2 X2 − A2
𝐝𝐱
8. ∫ After making perfect square, integrand will be changed in any one form given
√𝐚𝐱 𝟐 + 𝐛𝐱+𝐜
below.

dx dx dx
∫ or ∫ or ∫
√A2 − x 2 √A2 + x 2 √X 2 − A2
𝐩𝐱+𝐪
9. ∫ dx
𝐚𝐱 𝟐 + 𝐛𝐱+𝐜
𝒅
Steps of Solving (i) Put px + q = A (𝐚𝐱 𝟐 + 𝐛𝐱 + 𝐜) + B ⇒ px + q = A(2ax + b) + B
𝒅𝒙
(ii) Find the values A and B by comparing coefficients and constants.

9
(iii) Integrand will be split in two parts. One part will be solved by substitution method and
second part will be solved by making perfect square.

𝐩𝐱+𝐪
10. ∫ dx
√𝐚𝐱 𝟐 + 𝐛𝐱+𝐜
𝒅
Steps of Solving (i) Put px + q = A (𝐚𝐱 𝟐 + 𝐛𝐱 + 𝐜) + B ⇒ px + q = A(2ax + b) + B
𝒅𝒙
(ii) Find the value A and B by comparing coefficients and constants.
(iii) Integrand will be split in two parts. One part will be solved by substitution method and
second part will be solved by making perfect square.
𝑥 𝑎2 𝑥
11. ∫ √a2 − x 2 𝑑𝑥 = √a2 − x 2 + 𝑠𝑖𝑛−1 + C
2 2 𝑎

𝑥 𝑎2
12. ∫ √x 2 + a2 𝑑𝑥 = √x 2 + a2 + log | x + √x 2 + a2 | + C
2 2
𝑥 𝑎2
13. ∫ √x 2 − a2 𝑑𝑥 = √x 2 − a2 - log | x + √x 2 − a2 | + C
2 2
Method of making perfect square of a quadratic polynomial-
Quadratic Polynomial = ax 2 + bx + c
b c b b b c
= a (x 2 ± x + ) = a (x 2 ± 2. x + ( )2 − ( )2 + )
a a 2a 2a 2a a
b √𝑏 2 −4𝑎𝑐
= a {(𝑥 ± 2a) − 2
( 2𝑎 )2 }
b √𝑏2 −4𝑎𝑐 2
Hence, quadratic polynomial ax 2 + bx + c = a {(𝑥 ± 2a) 2 − ( ) }
2𝑎
3 5
Example: 2x 2 + 3x + 5 = 2 (x 2 + x + 2)
2
3 3 3 5 3 √31 2
= 2{(x 2 + 2. x + (4)2 − (4)2 + )} = 2 {(𝑥 + 4) 2 + ( ) }
4 2 4
Some Important Examples: Solve the following:
𝑑𝑥 1 𝑥+2
(i)∫ (ii) ∫√ 𝑑𝑥 (iii) ∫ 𝑑𝑥(iv) ∫ √3 − 2x − x2 𝑑𝑥
2 𝑠𝑖𝑛2 𝑥+5 𝑐𝑜𝑠2 𝑥 5−4x−2x2 √ x2 + 5x+6
𝑑𝑥
Solutions (i) I = ∫
2 𝑠𝑖𝑛2 𝑥+5 𝑐𝑜𝑠2 𝑥
𝑠𝑒𝑐 2 𝑥 𝑑𝑥
=∫ (divide numerator and denominator by cos2x)
2 𝑡𝑎𝑛2 𝑥+5
Now put tanx = t ⇒ 𝑠𝑒𝑐 2 𝑥 𝑑𝑥 = dt
𝑑𝑡
= ∫ 2 𝑡 2+5 = 2 ∫
1
𝑡 2+
𝑑𝑡
5
1
= ∫
2
𝑑𝑡
5
= 1
2
2
√ 𝑡𝑎𝑛−1 √ 𝑡
5
2
5
+ C = √110 𝑡𝑎𝑛−1 (√25 𝑡𝑎𝑛𝑥) + C
2 𝑡 2 + (√ )2
2
1 1 1 1
(ii) I = ∫ 𝑑𝑥 = ∫ 𝑑𝑥 = ∫ 𝑑𝑥 = ∫ 𝑑𝑥
√5−4x−2x2 √ 5+2−2−4x−2x2 √7−2−4x−2x2 √7−2(1+2x+x2 )

∵ 1 + 2x + x 2 = (𝑥 + 1)2

10
1 1 1 2
= ∫ 𝑑𝑥 = 𝑠𝑖𝑛−1 √ (𝑥 + 1) + C
√2 7
√ − (𝑥+1)2 √2 7
2
𝑥+2 𝑑
(iii)I = ∫ 𝑑𝑥 Put x + 2 = A (x 2 + 5x + 6) + B ⇒ x + 2 = A(2x + 5) + B
√x2 + 5x+6 𝑑𝑥
1 1 1 1
Now by comparing A = 2& B = - 2 , Now x + 2 = 2 (2x + 5) + - 2
1 1
𝑥+2 (2x + 5) + − 1 2𝑥+5 1 1
I= ∫ 𝑑𝑥= ∫ 2 2
𝑑𝑥 =2 ∫ 𝑑𝑥- 2 ∫ 𝑑𝑥
√ x2 + 5x+6 √x2 + 5x+6 √x2 + 5x+6 √x2 + 5x+6

1 2𝑥+5 1 1
I= I1 - I2…………………….(eq.1) , where I1 = 2 ∫ √ 𝑑𝑥and I2 = 2 ∫ 𝑑𝑥
x2 + 5x+6 √x2 + 5x+6
1 2𝑥+5
Now I1 = 2 ∫ 𝑑𝑥Put x 2 + 5x + 6 = t 2 ⇒(2𝑥 + 5) dx = 2t dt
√x2 + 5x+6
1 2𝑡 𝑑𝑡
=2∫ = ∫ 𝑑𝑡 = t ⇒I1= √x 2 + 5x + 6 + c1
√t2

1
I2 = 2 ∫
√x2 + 5x+6
1
𝑑𝑥 ∵ x2 + 5
5x + 6 = = x 2 + 2. 2 x + (2)2 − (2)2 + 6)}
5 5

5 1 2
= {(𝑥 + ) 2 − ( ) }
2 2
1 1 1
=2 ∫ 𝑑𝑥= 2log |x + √x 2 + 5x + 6|+ c2
2 2
√(x+5) − (1)
2 2

Now put the value of I1 and I2 in equation (1), we get –


1
I =√x 2 + 5x + 6 + C1 - 2log |x + √x 2 + 5x + 6|+ C2
1
I =√x 2 + 5x + 6 - 2log |x + √x 2 + 5x + 6|+ C, where C = C1 + C2
(iv) I = ∫ √3 − 2x − x 2 𝑑𝑥(CBSE 2019)
= ∫ √4 − (x + 1)2 𝑑𝑥= ∫ √22 − (x + 1)2 𝑑𝑥
Now put x+1 =t ⇒ dx =dt = ∫ √22 − (t)2 𝑑𝑥
𝑥 𝑎2 𝑥
(Using ∫ √a2 − x 2 𝑑𝑥 = 2 √a2 − x 2 + 𝑠𝑖𝑛−1 𝑎 + C )
2
𝑡 22 𝑥 x+1 𝑥+1
= 2 √22 − t 2 + 𝑠𝑖𝑛−1 2 + C = √3 − 2x − x 2 + 2𝑠𝑖𝑛−1 +C
2 2 2
Integration by Partial Fraction:
• Partial fraction method is used to find integration of Rational functions.
• Rational Function: A function in the form of 𝑔(𝑥) , where f(x) and g(x) are polynomials in x
𝑓(𝑥)

and g(x) ≠ 0, is called rational function.


• Types of Rational Functions:
(i)Proper Rational Function- If the degree of f(x) < Degree of g(x) then the function
is called proper rational function.
𝑥+1 𝑥+1
Example- , …….etc.
𝑥3 − 𝑥 x (𝑥2 + 1)

(ii) Improper Rational Function- If the degree of f(x) ≥ Degree of g(x) then the
function is called improper rational function.

11
𝑥+1 x2 + 3x+5
• Example- 𝑥− 1 , (𝑥 − 1) …….etc.

• If a function is Improper rational function, then first convert it into Proper rational
function using division method.
• Factorise the denominator ‘g(x)’. Factors of g(x) may be linear or quadratic.
• A Proper rational function will decompose in the following forms given in table –
S.No. Form of the rational (Proper) Form of the Partial fraction
Function
1 𝑓(𝑥) 𝐴 𝐵
(𝑥 ±𝑎)(𝑥 ±𝑏)
+
(𝑥 ± 𝑎) (𝑥 ± 𝑏)
Find the values of A & B by comparing
coefficients and constants both sides.
2 𝑓(𝑥) 𝐴 𝐵
(𝑥 ±𝑎)2
+
(𝑥 ± 𝑎) (𝑥 ± 𝑎)2
𝑓(𝑥) 𝐴 𝐵𝑥 + 𝐶
, +
3 (𝑥±𝑎)(ax2 + bx+c) 2
(𝑥 ± 𝑎) (ax + bx + c)
Find the values of A & B by comparing
Where ax 2 + bx + c cannot be coefficients and constants both sides.
factorise.
𝑓(𝑥) 𝐴𝑥 + 𝐵 𝐶𝑥 + 𝐷
4 +
(ax2 + bx+c)2 (ax 2 + bx + c) (ax 2 + bx + c)2

Some Important Examples-


𝑥 3+ 1
Solve the following: (i)∫ 𝑑𝑥 (ii) ∫
1
x (𝑥3 + 8)
𝑑𝑥 (iii) ∫
𝑥+1
x (𝑥2 + 1)
𝑑𝑥
𝑥 3− 𝑥
𝑥3+ 1 𝑥3+ 1 𝑥+ 1 1
Solutions (i) I = ∫ 𝑑𝑥∵ =1+ =1 +
𝑥3− 𝑥 𝑥3− 𝑥 𝑥(𝑥 2 − 1) 𝑥(𝑥− 1)
1 1
I = ∫(1 + ) 𝑑𝑥 = ∫ dx + ∫ ) 𝑑𝑥…………………..(1)
𝑥(𝑥− 1) 𝑥(𝑥− 1)
1 𝐴 𝐵
Now = + …………………(2) (partial fraction form)
𝑥(𝑥− 1) 𝑥 (𝑥− 1)
1 = A (x-1) + B.x, By solving , we get A = -1 & B = 1
1 −1 1
From equation (2) = +
𝑥(𝑥− 1) 𝑥 (𝑥− 1)
1 1
Now I = ∫ dx − ∫ 𝑑𝑥 + ∫ 𝑑𝑥 = x – log|x| + log|x-1| +C
𝑥 𝑥−1
𝑥2
(ii)I = ∫ x (𝑥31+ 8) 𝑑𝑥 =∫ 3 3
𝑥 (𝑥 + 8)
𝑑𝑥 (Multiply and divide by 𝑥 2 )
1
Now put 𝑥 3 = 𝑡 ⇒ 3𝑥 𝑑𝑥 = 𝑑𝑡⇒𝑥 𝑑𝑥 = 𝑑𝑡 2 2
3
1 𝑑𝑡 1 𝐴 𝐵
I= ∫
3 t(t+8)
∵ ,
𝑡(𝑡+8)
=
𝑡
+
(𝑡+ 8)
…………………….(1)
1 1
⇒ 1 = A (t + 8) + B.t After solving, we get A = & B = −
8 8

12
1 1 1 1 1
Put values of A & B in equation (1) = −
𝑡(𝑡+8) 8 𝑡 8 (𝑡+ 8)

1 𝑑𝑡 1 𝑑𝑡 1 1
I= ∫ - ∫ ⇒I= log|t| - log|t+8| + C
3 8t 3 8(t+8) 24 24
1 1 1 𝑥3
= log|𝑥 3 | - log|𝑥 3 + 8| + C = log|
𝑥 3 +8
|+C
24 24 24
𝑥+1
(iii) I = ∫ 𝑑𝑥
x (𝑥2 + 1)

∵ x (𝑥𝑥+1
2 + 1) =
𝐴
x
+
𝐵𝑥+𝐶
(𝑥2 + 1)
…………………………(1)

X + 1 = A (𝑥2 + 1) + (Bx + C) x (After solving , we get A = 1, B = -1 & C = 1)


𝑥+1 1 (𝑥−1)
Putting the values of A, B & C in equation (1), we get = –
x (𝑥2 + 1) x (𝑥2 + 1)

1 𝑥−1 1 1 2𝑥 1
I = = ∫ x 𝑑𝑥 − ∫ 𝑑𝑥 = ∫ x 𝑑𝑥 − 2 ∫ 𝑑𝑥 + ∫ 𝑑𝑥
(𝑥2 + 1) 2
( 𝑥 + 1) 2
( 𝑥 + 1)

= log|x| -log|𝑥 2 + 1| + 𝑡𝑎𝑛−1 𝑥 + C


Integration by Parts:
• Integration by parts is used to find integration of the following types of the functions
(i) Product of two functions: f(x). g(x), Example- x sinx, exsinx………etc
I = ∫ 𝑓(𝑥)𝑔(𝑥)𝑑𝑥 = ∫ 𝑢. 𝑣 𝑑𝑥where u and v are functions of x.
(ii) Logarithmic functions : logx, log(x+1)……etc
(iii) Inverse trigonometric functions : sin-1x, cos-1x……..etc

• First select the function as I & II with the help of the word ‘ILATE’

Select the first function which comes In the word ‘ILATE’


first as per alphabetical order in the
word ILATE.
I= Inverse trigonometric functions
L = Logarithmic functions
ILATE A = Algebraic Functions
T = Trigonometric functions
E = Exponential functions
• Now apply the formula to find integral of ∫ 𝑓(𝑥)𝑔(𝑥)𝑑𝑥 = ∫ 𝑢. 𝑣 𝑑𝑥, where u and v are
functions of x.

𝑑𝑢
∫ 𝑢. 𝑣 𝑑𝑥 = u ∫ 𝑣 𝑑𝑥-∫{𝑑𝑥 ∫ 𝑣 𝑑𝑥}𝑑𝑥 , where u = first function & v = second function.

• Some Important Properties:


(i) ∫ 𝑒 𝑥 {𝑓(𝑥) + 𝑓 ′(𝑥)}𝑑𝑥 = ex f(x) + C
13
𝑎𝑥
(ii)∫ 𝑒 𝑎𝑥 𝑠𝑖𝑛𝑏𝑥 𝑑𝑥 = (𝑎2𝑒+𝑏2) (𝑎𝑠𝑖𝑛𝑏𝑥 − 𝑏 𝑐𝑜𝑠𝑏𝑥) + 𝐶
𝑒 𝑎𝑥
(iii) ∫ 𝑒 𝑎𝑥 𝑐𝑜𝑠𝑏𝑥 𝑑𝑥 = (𝑎2+𝑏2) (𝑎𝑐𝑜𝑠𝑏𝑥 + 𝑏 𝑠𝑖𝑛𝑏𝑥) + 𝐶

Some Important Examples:-


𝑙𝑜𝑔𝑥
Solve the following(i)∫ 𝑥 2 𝑙𝑜𝑔𝑥 𝑑𝑥 (ii) ∫(𝑠𝑖𝑛−1𝑥)2 𝑑𝑥 (iii) ∫
(1+𝑙𝑜𝑔𝑥)2
𝑑𝑥
(iii) ∫ 𝑒 𝑥 (𝑐𝑜𝑠𝑥 − 𝑠𝑖𝑛𝑥)𝑐𝑜𝑠𝑒𝑐 2 𝑥 𝑑𝑥
Solutions- (i) I = 𝑥 2 𝑙𝑜𝑔𝑥 𝑑𝑥

II I
𝑑(𝑙𝑜𝑔𝑥)
= logx∫ 𝑥 2 𝑑𝑥 - ∫{ 𝑑𝑥 ∫ 𝑥2 𝑑𝑥}𝑑𝑥
1 1 1 1 1
= logx . 𝑥 3 − ∫ . 𝑥 3 𝑑𝑥 = 𝑥 3 .logx − ∫ 𝑥 2 𝑑𝑥
3 𝑥 3 3 3
1 1 1 1 1
= 𝑥 3 .logx − . 𝑥 + C = 𝑥 .logx − 𝑥 + C 3 3 3
3 3 3 3 9
(ii) I = ∫(𝑠𝑖𝑛 −1
𝑥)2 𝑑𝑥
Put 𝑠𝑖𝑛−1𝑥 = t ⇒ x = sint⇒ dx = cost dt
Now I = ∫ 𝑡 2 𝑐𝑜𝑠𝑡 𝑑𝑥 , taking 𝑡 2 = 𝑓𝑖𝑟𝑠𝑡 𝑓𝑢𝑛𝑐𝑡𝑖𝑜𝑛 & 𝑐𝑜𝑠𝑡 = 𝑠𝑒𝑐𝑜𝑛𝑑 𝑓𝑢𝑛𝑐𝑡𝑖𝑜𝑛
𝑑(𝑡2 )
I = 𝑡 2 ∫ 𝑐𝑜𝑠𝑡 𝑑𝑡 - ∫{ 𝑑𝑡 ∫ 𝑐𝑜𝑠𝑡 𝑑𝑡}𝑑𝑡 = 𝑡 2 . 𝑠𝑖𝑛𝑡- ∫ 2𝑡 . 𝑠𝑖𝑛𝑡 𝑑𝑡 +C
= 𝑡 2 . 𝑠𝑖𝑛𝑡- 2{𝑡 ∫ 𝑠𝑖𝑛𝑡 𝑑𝑡 +∫ 1. 𝑐𝑜𝑠𝑡 𝑑𝑡 }+ C (Again using by parts)
=𝑡 2 . 𝑠𝑖𝑛𝑡- 2{𝑡 (−𝑐𝑜𝑠𝑡) + sint }+ C
= 𝑡 2 . 𝑠𝑖𝑛𝑡+ 2𝑡 𝑐𝑜𝑠𝑡 -2 sint + C (put t = 𝑠𝑖𝑛−1 𝑥)
2
= x (𝑠𝑖𝑛−1 𝑥) +2√1 − 𝑥 2 𝑠𝑖𝑛−1 𝑥 - 2x + C

𝑙𝑜𝑔𝑥
(iv) I=∫ 𝑑𝑥
(1+𝑙𝑜𝑔𝑥)2
𝑙𝑜𝑔𝑥+1−1 𝑙𝑜𝑔𝑥+1 1
=∫ 𝑑𝑥 = ∫ 𝑑𝑥 - ∫ 𝑑𝑥
(1+𝑙𝑜𝑔𝑥)2 (1+𝑙𝑜𝑔𝑥)2 (1+𝑙𝑜𝑔𝑥)2
1 1
=∫ . 1 𝑑𝑥 - ∫ 𝑑𝑥
1+𝑙𝑜𝑔𝑥 (1+𝑙𝑜𝑔𝑥)2
1 −1 1 1
= ∫ 1 𝑑𝑥 - ∫ (1+𝑙𝑜𝑔𝑥)2 . 𝑥 × 𝑥 𝑑𝑥 − ∫ (1+𝑙𝑜𝑔𝑥)2 𝑑𝑥 (using by parts)
1+𝑙𝑜𝑔𝑥
𝑥 1 1 𝑥
= +∫ 𝑑𝑥 − ∫ 𝑑𝑥 = +C
1+𝑙𝑜𝑔𝑥 (1+𝑙𝑜𝑔𝑥)2 (1+𝑙𝑜𝑔𝑥)2 1+𝑙𝑜𝑔𝑥

(v) I = ∫ 𝑒 𝑥 (𝑐𝑜𝑠𝑥 − 𝑠𝑖𝑛𝑥)𝑐𝑜𝑠𝑒𝑐 2 𝑥 𝑑𝑥


(𝑐𝑜𝑠𝑥−𝑠𝑖𝑛𝑥)
= ∫ 𝑒 𝑥 𝑠𝑖𝑛𝑥 𝑐𝑜𝑠𝑒𝑐𝑥 𝑑𝑥 = ∫ 𝑒 𝑥 (𝑐𝑜𝑡𝑥 − 1) 𝑐𝑜𝑠𝑒𝑐𝑥 𝑑𝑥
=∫ 𝑒 𝑥 (𝑐𝑜𝑡𝑥 𝑐𝑜𝑠𝑒𝑐𝑥 − 𝑐𝑜𝑠𝑒𝑐𝑥) 𝑑𝑥 = - ∫ 𝑒 𝑥 (𝑐𝑜𝑠𝑒𝑐𝑥 − 𝑐𝑜𝑠𝑒𝑐𝑥𝑐𝑜𝑡𝑥 ) 𝑑𝑥
Now using ∫ 𝑒 𝑥 {𝑓(𝑥) + 𝑓 ′ (𝑥)}𝑑𝑥 = ex f(x) + C (∵f(x) = cosecx &𝑓 ′ (𝑥) = −𝑐𝑜𝑠𝑒𝑐𝑥𝑐𝑜𝑡𝑥)
I = -𝑒 𝑥 𝑐𝑜𝑠𝑒𝑐𝑥 + C
Miscellaneous Questions

14
Level –I (2 Marks )
(1+𝑥)2 2 4
1. ∫ 𝑑𝑥 (NCERT) Ans. I = 2√𝑥 + 𝑥 5/2 + 𝑥 3/2 +C
√𝑥 5 3
𝑥3 𝑥2 𝑥3
2. ∫ 𝑑𝑥 (NCERT EXEMPLAR) Ans. I = x - + - log |x+1| + C
(𝑥+1) 2 3
sin (𝑥−𝑎)
3. ∫ 𝑑𝑥 (CBSE 2019) Ans. I = x cos2a –sin2a log|sin(x+a)|+C
sin (𝑥+𝑎)
1
4. ∫ 𝑠𝑒𝑐 3 𝑥 𝑑𝑥 (CBSE 2020) Ans. I = 𝑠𝑒𝑐𝑥𝑡𝑎𝑛𝑥 +
2
1
log |𝑠𝑒𝑐𝑥𝑡𝑎𝑛𝑥| +C
2
(x−5)ex ex
5. ∫ 𝑑𝑥 (CBSE 2019) Ans. I= +C
(x−3)3 (x−3)2
3 3
𝑑𝑥 2
6. ∫ Ans. I = {(𝑥 + 𝑎)2 − (𝑥 + 𝑏)2 } + 𝐶
√𝑥+𝑎+ √𝑥+𝑏) 3 (𝑎−𝑏)

𝑠𝑖𝑛3 𝑥+𝑐𝑜𝑠 3 𝑥
7. ∫ 𝑑𝑥(CBSE 2019) Ans. I = secx –cosecx+C
𝑠𝑖𝑛2 𝑥𝑐𝑜𝑠 2 𝑥
2𝑥 3
8. ∫ 3 𝑑𝑥(CBSE 2020)Ans. (𝑥 2 + 1)2/3 +C
√𝑥 2 +1 2

√𝑡𝑎𝑛𝑥
9. ∫ 𝑑𝑥 (NCERT) Ans. 2√𝑡𝑎𝑛𝑥 +C
𝑠𝑖𝑛𝑥 𝑐𝑜𝑠𝑥
sinx−x cosx
10. ∫ 𝑑𝑥 Ans. I = log|x| - log|x+sinx| + C
x (x+ sinx)

ex ( 1+sinx) x
11. ∫ 𝑑𝑥 Ans. I= ex tan +C
(1+𝑐𝑜𝑠𝑥) 2

x cos−1 x
12. ∫ 𝑑𝑥 Ans. I= - √1 − 𝑥 2 cos −1 x - x + C
√1− 𝑥 2
e2x −e−2x 1
13. ∫ 2x −2x 𝑑𝑥 Ans. I= log | e2x + e−2x | + C
e +e 2
(x+1)(x+logx)2 1
14. ∫ 𝑑𝑥 Ans. I= (x + logx)3 + C
x 3
1 1
15. ∫ 𝑠𝑖𝑛2 𝑥 𝑐𝑜𝑠 2 𝑥𝑑𝑥 Ans. I = - 𝑐𝑜𝑠 3 𝑥 + 𝑐𝑜𝑠 5 𝑥 + C
3 5

Level –II (3 Marks)


𝑥 𝑥 √𝑥 𝑥
16. ∫ 𝑑𝑥 Ans. I = 2( − + √𝑥 -log |√𝑥 +1|)+C
√𝑥+ 1 3 2
sin 𝑥
17. ∫ 𝑑𝑥 Ans. I = x cosa –sina log|sin(x+a)|+C
sin (𝑥+𝑎)

15
𝑑𝑥 1 −1 𝑎𝑡𝑎𝑛𝑥
18. ∫ 2 2 𝐴𝑛𝑠. 𝐼 = 𝑡𝑎𝑛 ( )+C
𝑎 𝑠𝑖𝑛 𝑥+ 𝑏2 𝑐𝑜𝑠 2 𝑥 𝑎𝑏 𝑏
𝑥+1 3
19. ∫ 𝑑𝑥 (CBSE 2020) Ans. I= log|x| - log|1-2x| + C
𝑥(1−2𝑥) 2

1 1 −1 3𝑥+1
20. ∫ 2 𝑑𝑥 𝐴𝑛𝑠. 𝐼 = 𝑡𝑎𝑛 ( )+C
9x + 6x+5 6 2

(x2 −5)ex ex
21. ∫ 𝑑𝑥 Ans.𝐼 = ex − 2 +C
(x+1)2 (𝑥+1)
ex
22. ∫ (x+1) {1 + (𝑥 + 1) log(𝑥 + 1)}𝑑𝑥 Ans.I = ex log (x + 1) +C
1 1 cos (𝑥−𝛽)
23. ∫ 𝑑𝑥 Ans. 𝐼 = log | |+C
cos (𝑥−𝛼)cos (𝑥−𝛽) sin (𝛽−𝛼) cos (𝑥−𝛼)

1 2 sin (x+𝛼)
24. ∫ 𝑑𝑥 Ans. 𝐼 = − √ + C
√sin3 x sin(x−𝛼) sin𝛼 sinx

(2x+1) −1 𝑥−1
25. ∫ 𝑑𝑥 (CBSE 2020) 𝐴𝑛𝑠. 𝐼 = − 2√3 + 2x − x 2 + 3𝑠𝑖𝑛 ( )+C
√3+2x− x2 2

2𝑥+1 − 5𝑥−1 1 1 2 1
26. ∫ 𝑑𝑥 (CBSE 2020) Ans.I = - +C
10𝑥 5 log 2 2𝑥 log 5 5𝑥

𝑥2 1 1+x 1
27. ∫ 𝑑𝑥 Ans.I = log | | - 2 𝑡𝑎𝑛−1 𝑥 + C
1− 𝑥 4 4 1−x
𝑐𝑜𝑠2𝑥 𝑐𝑜𝑠6𝑥 𝑐𝑜𝑠4𝑥
28. ∫ 𝑠𝑖𝑛𝑥. 𝑠𝑖𝑛2𝑥. 𝑠𝑖𝑛3𝑥 𝑑𝑥 Ans. I = - + – +C
8 24 16
𝑒𝑥 𝑒 𝑥+ 2
29. ∫ 𝑑𝑥 Ans. I = 𝑠𝑖𝑛−1 ( )+C
√5−4𝑒 𝑥 − e 2x 3
x2 2 −1 𝑥 3 𝑥
30. ∫ (x2 dx Ans.I = − 𝑡𝑎𝑛 + 5 𝑡𝑎𝑛−1 3 + C
+4)(x2 + 9) 5 2

Level –III (4 Marks)


(x+2) 1 1 −1
31. ∫ 2 𝑑𝑥 Ans.I= log|2x 2 + 6x + 5| + 𝑡𝑎𝑛 (2𝑥 + 3) +
2x +6x+5 4 2

C
(x2 −1) 1 x2 +1−x
32. ∫ 4 𝑑𝑥 Ans.I= log | |+C
x +x+1 2 x2 +1+x

33. ∫ √𝑡𝑎𝑛𝑥 𝑑𝑥 (Hint: put tanx = 𝑡 2 )


1 𝑡𝑎𝑛𝑥−1 1 tanx−√2𝑡𝑎𝑛𝑥 +1
Ans.I= 𝑡𝑎𝑛−1 ( )+2 log | |+C
√2 √ 2𝑡𝑎𝑛𝑥 √2 tanx+√2𝑡𝑎𝑛𝑥 +1
𝑑𝑥
34. ∫ 4 (Hint: Divide numerator and denominator by cos4x)
𝑐𝑜𝑠 𝑥+ 𝑠𝑖𝑛4 𝑥
1 𝑡𝑎𝑛2 𝑥−1
Ans.𝐼 = 𝑡𝑎𝑛−1 ( )+ C
√2 √2𝑡𝑎𝑛𝑥
35. ∫ 𝑒 −3𝑥 𝑐𝑜𝑠 3 𝑥 𝑑𝑥(Hint: Apply Integration by parts )

16
1 13 3
Ans. I =- 𝑒 −3𝑥 (𝑠𝑖𝑛3 𝑥 + 𝑐𝑜𝑠 3 𝑥) + 𝑒 −3𝑥 𝑠𝑖𝑛𝑥 − 𝑒 −3𝑥 𝑐𝑜𝑠𝑥 + 𝐶
6 60 20
(x2 +x+1) 3 1 1
36. ∫ (x+2)(x2 𝑑𝑥 Ans. I= log|x + 2| + log|x 2 + 1| + 𝑡𝑎𝑛−1 𝑥 + C
+ 1) 5 5 5
2𝑐𝑜𝑠𝑥
37. ∫ (1−sinx)(2−cos2 𝑑𝑥(CBSE2019)(Hint: use cos 2 x = 1 − sin2 x and put sinx =t)
x)
1
Ans. I= − log|1 − sinx| + log|1 + sin2 x| +| + 𝑡𝑎𝑛−1 (𝑠𝑖𝑛𝑥) + C
2

𝑎
38. ∫ 𝑠𝑖𝑛−1 √ 𝑑𝑥(Hint: put x = atan2t)
𝑎+𝑥

𝑥 𝑥 𝑥 𝑥
Ans. I = a( 𝑡𝑎𝑛−1 √ − √ + 𝑡𝑎𝑛−1 √ ) + 𝐶
𝑎 𝑎 𝑎 𝑎
𝑠𝑖𝑛2𝑥
39. ∫ 2 𝑑𝑥(Hint: use sin2x= 2sinx cosx and put sinx = t)
(sin x+1)(sin2 x+3)
1 sin2 x+1
𝐴𝑛𝑠.I = log | |+C
2 sin2 x+3
√1+𝑥 2 {log(𝑥 2 +1)−2𝑙𝑜𝑔𝑥} 1 1 3/2 1 2
40. ∫ 𝑑𝑥 Ans. - (1 + ) {log (1 + ) − 3} +C
x4 3 𝑥2 𝑥2
41. ∫(√𝑐𝑜𝑡𝑥 + √𝑡𝑎𝑛𝑥)𝑑𝑥 ( Hint: Change Sinx & Cosx)
𝐴𝑛𝑠. 𝐼 = √2 𝑠𝑖𝑛−1 (𝑠𝑖𝑛𝑥 − 𝑐𝑜𝑠𝑥) + 𝐶
𝑑𝑥
42. ∫ 𝑑𝑥 (Hint: Use sin2x = 2sinx cosx and Put cosx= t)
sinx+sin2x
1 1 2
Ans. I= log|1 + 𝑐𝑜𝑠𝑥| + log|1 − 𝑐𝑜𝑠𝑥| − log|1 + 2 𝑐𝑜𝑠𝑥| + 𝐶
2 6 3

17
DEFINITE INTEGRATION
𝑏
The definite integral is denoted by ∫𝑎 𝑓(𝑥)𝑑𝑥, where 𝑎 is the lower limit of the
integral and 𝑏 is the upper limit of the integral.
𝑏
We have defined ∫𝑎 𝑓(𝑥)𝑑𝑥 as the area
of the region bounded by the curve 𝑦 =
𝑓(𝑥) the ordinates x=a, x=b and x-axis.
Let 𝑥 be a given point in the interval
𝑥
[𝑎 , 𝑏]. Then ∫𝑎 𝑓(𝑥)𝑑𝑥 represents the
area of light shaded region in the fig 7.3

the area of this shaded region depends


upon the value of 𝑥. i.e. the area of this
shaded region is a function of 𝑥. We
denote this area function by 𝐴(𝑥)
𝑥
i.e. Area function 𝐴(𝑥) = ∫𝑎 𝑓(𝑥)𝑑𝑥
First Fundamental Theorem of Calculus.
Theorem 1: Let 𝑓 be a continuous function on the closed interval [𝑎 , 𝑏]and let
𝐴(𝑥)be the area function. Then 𝐴′ (𝑥) = 𝑓(𝑥)for all 𝑥𝜖[𝑎 , 𝑏]

Second Fundamental Theorem of Calculus.


Theorem 2: Let f be a Continuous function defined on the closed
interval [a ,b] and F be an antiderivative of 𝑓. Then
𝑏
∫𝑎 𝑓(𝑥)𝑑𝑥 = [𝐹(𝑥)]𝑏𝑎 =F(b)-F(a)

PROPERTIES OF DEFINITE INTEGRALS:


𝑏 𝑏
𝑃0: ∫ 𝑓(𝑥)𝑑𝑥 = ∫ 𝑓(𝑡)𝑑𝑡
𝑎 𝑎
𝑏 𝑎 𝑎
𝑃1: ∫𝑎 𝑓(𝑥)𝑑𝑥 = − ∫𝑏 𝑓(𝑥)𝑑𝑥 in particular ∫𝑎 𝑓(𝑥)𝑑𝑥 = 0
𝑏 𝑐 𝑏
𝑃2: ∫𝑎 𝑓(𝑥)𝑑𝑥 = ∫𝑎 𝑓(𝑥)𝑑𝑥 + ∫𝑐 𝑓(𝑥)𝑑𝑥 where 𝑎 < 𝑐 < 𝑏
𝑏 𝑏
𝑃3: ∫ 𝑓(𝑥)𝑑𝑥 = ∫ 𝑓(𝑎 + 𝑏 − 𝑥)𝑑𝑥
𝑎 𝑎
𝑎 𝑎
𝑃4: ∫ 𝑓(𝑥)𝑑𝑥 = ∫ 𝑓(𝑎 − 𝑥)𝑑𝑥
0 0
2𝑎 𝑎 𝑎
𝑃5: ∫ 𝑓(𝑥)𝑑𝑥 = ∫ 𝑓(𝑥)𝑑𝑥 + ∫ 𝑓(2𝑎 − 𝑥)𝑑𝑥
0 0 0
2𝑎 𝑎
𝑃6: ∫ 𝑓(𝑥)𝑑𝑥 = 2 ∫ 𝑓(𝑥)𝑑𝑥 , 𝑖𝑓 (2𝑎 − 𝑥) = 𝑓(𝑥)
0 0
=0, 𝑖𝑓 (2𝑎 − 𝑥) = −𝑓(𝑥)
𝑎 𝑎
𝑃7: (𝑖) ∫ 𝑓(𝑥)𝑑𝑥 = 2 ∫ 𝑓(𝑥)𝑑𝑥, 𝑖𝑓 𝑓(𝑥) 𝑖𝑠 𝑎𝑛 𝑒𝑣𝑒𝑛 𝑓𝑢𝑛𝑐𝑡𝑖𝑜𝑛 𝑖. 𝑒. 𝑓(−𝑥) = 𝑓(𝑥)
−𝑎 0
𝑎
(𝑖𝑖) ∫−𝑎 𝑓(𝑥)𝑑𝑥 = 0, 𝑖𝑓 𝑓(𝑥) 𝑖𝑠 𝑎𝑛 𝑜𝑑𝑑 𝑓𝑢𝑛𝑐𝑡𝑖𝑜𝑛 𝑖. 𝑒. 𝑓(−𝑥) = −𝑓(𝑥)

18
LEVEL 1 (2 MARKS)
1 4
𝑥
𝐸𝑣𝑎𝑙𝑢𝑎𝑡𝑒: ∫ 𝑑𝑥
2 𝑥2 +1
Sol. 4 𝑥
Let I=∫2 𝑥 2 +1 𝑑𝑥
1 4 2𝑥 1
= 2 ∫2 𝑥 2 +1 𝑑𝑥 = 2 [log(𝑥 2 + 1)]42
1 1 17 𝑎
= 2 (𝑙𝑜𝑔17 − 𝑙𝑜𝑔5) = 2 𝑙𝑜𝑔 ( 5 ) Using 𝑙𝑜𝑔𝑎 − 𝑙𝑜𝑔𝑏 = log (𝑏)
2 1
𝑡𝑎𝑛−1 𝑥
𝐸𝑣𝑎𝑙𝑢𝑎𝑡𝑒: ∫ 𝑑𝑥
0 1 + 𝑥2
Sol. 1 tan−1 𝑥 (tan−1 𝑥)
2 1
(tan−1 1)
2
(tan−1 0)
2 1
(𝜋/4)2 𝜋2
∫0 1+𝑥 2 𝑑𝑥=[ 2
] =[ 2
−[ 2
] ]= 2
= 32
0 0
3 𝜋/4
𝐸𝑣𝑎𝑙𝑢𝑎𝑡𝑒: ∫ 𝑡𝑎𝑛 𝑥 𝑑𝑥
0
Sol. 𝜋/4
∫0 tan 𝑥 𝑑𝑥 = [𝑙𝑜𝑔|𝑠𝑒𝑐𝑥|]0
𝜋/4 𝜋
= 𝑙𝑜𝑔 |𝑠𝑒𝑐 ( 4 )| − 𝑙𝑜𝑔|𝑠𝑒𝑐 (0)|
1
= 𝑙𝑜𝑔|√2| − 𝑙𝑜𝑔|1|= 𝑙𝑜𝑔|√2| 𝑜𝑟 2 𝑙𝑜𝑔2
𝑎
4 𝐼𝑓 ∫0 3𝑥 2 𝑑𝑥 = 8, then find ‘a’.
𝑎
Sol. 𝑎
3𝑥 3
2
∫ 3𝑥 𝑑𝑥 = 8 𝑔𝑖𝑣𝑒𝑠 [ ] = 3 𝑖. 𝑒. 𝑎3 = 8 , 𝑠𝑜 𝑎 = 2
0 3 0
5 1
𝐸𝑣𝑎𝑙𝑢𝑎𝑡𝑒: ∫ 𝑥17 𝑐𝑜𝑠 4 𝑥 𝑑𝑥
−1
Sol. 1 17 4
∫−1 𝑥 𝑐𝑜𝑠 𝑥 𝑑𝑥 =0 , using property ∫−𝑎 𝑓(𝑥)𝑑𝑥 = 0,
𝑎

𝑖𝑓 𝑓(𝑥)𝑖𝑠 𝑎𝑛 𝑜𝑑𝑑 𝑓𝑢𝑛𝑐𝑡𝑖𝑜𝑛 𝑖. 𝑒. 𝑓(−𝑥) = −𝑓(𝑥)


Here 𝑓(𝑥) = 𝑥17 𝑐𝑜𝑠 4 𝑥 is an odd function as 𝑓(−𝑥) = (−𝑥)17 𝑐𝑜𝑠 4 (−𝑥) = −𝑓(𝑥)
𝜋
6 2 𝑠𝑖𝑛𝑥 − 𝑐𝑜𝑠 𝑥
𝐸𝑣𝑎𝑙𝑢𝑎𝑡𝑒: ∫ 𝑑𝑥
0 1 + 𝑠𝑖𝑛𝑥𝑐𝑜𝑠𝑥
𝜋
Sol. 𝑠𝑖𝑛𝑥−cos 𝑥
Let I = ∫0 2 𝑑𝑥 ………….(1)
1+𝑠𝑖𝑛𝑥𝑐𝑜𝑠𝑥
𝑎 𝑎
Using property∫0 𝑓(𝑥)𝑑𝑥 = ∫0 𝑓(𝑎 − 𝑥)
𝜋 𝜋 𝜋 𝜋
sin( −𝑥)−cos( −𝑥) cos 𝑥−sin 𝑥
I= ∫0 2 2
𝜋
2
𝜋 𝑑𝑥 I= ∫02 1+sin 𝑥 cos 𝑥 𝑑𝑥 …………………(2)
1+sin( −𝑥) cos( −𝑥)
2 2
Adding (1) & (2), we get
𝜋
2I= ∫02 0. 𝑑𝑥 = 0 So I = 0
𝜋 𝜋
7 𝑑𝑥 √𝑠𝑖𝑛𝑥
Evaluate: 𝐼 = ∫0 2 𝑂𝑅 𝐼 = ∫02 𝑑𝑥
1+√𝑐𝑜𝑡𝑥 √𝑠𝑖𝑛𝑥+√𝑐𝑜𝑠𝑥
𝜋 𝜋
Sol. 2 𝑑𝑥 2 √𝑠𝑖𝑛𝑥
We have I = ∫ OR I = ∫ dx
0 1+ √𝑐𝑜𝑡𝑥 0 √𝑠𝑖𝑛𝑥 + √𝑐𝑜𝑠𝑥
𝜋
√𝑠𝑖𝑛𝑥
I = ∫02 dx ……………. (1)
√𝑠𝑖𝑛𝑥+√𝑐𝑜𝑠𝑥
𝑎 𝑎
Using property∫0 𝑓(𝑥)𝑑𝑥 = ∫0 𝑓(𝑎 − 𝑥), we get
𝜋
𝜋 √𝑠𝑖𝑛( −𝑥) 𝜋
2 √𝑐𝑜𝑠𝑥
𝐼 = ∫0 2 𝑑𝑥 = ∫02 𝑑𝑥 ……………………(2)
𝜋
√𝑠𝑖𝑛( −𝑥)+√𝑐𝑜𝑠( −𝑥)
𝜋 √𝑐𝑜𝑠 𝑥+√𝑠𝑖𝑛 𝑥
2 2
Adding (1) & (2) ,we get

19
𝜋 𝜋 𝜋
√𝑠𝑖𝑛 𝑥+√𝑐𝑜𝑠 𝑥 𝜋 𝜋
2I = ∫02 𝑑𝑥 = ∫02 𝑑𝑥= (𝑥)02 = So, 𝐼 =
√𝑠𝑖𝑛 𝑥+√𝑐𝑜𝑠 𝑥 2 4
8 Evaluate:
1
∫0 𝑥(1 − 𝑥) dx 𝑛

Sol. I
1
=∫0 𝑥(1 − 𝑥)𝑛

1 𝑎 𝑎
=∫0 (1 − 𝑥)(1 − (1 − 𝑥))𝑛 𝑑𝑥 [using property ∫0 𝑓(𝑥)𝑑𝑥 = ∫0 𝑓(𝑎 − 𝑥)𝑑𝑥]
𝑛+2 1
1 𝑥 𝑛+1 𝑥 1
= ∫0 (1 − 𝑥)𝑥 𝑑𝑥 𝑛
= ( 𝑛+1 − 𝑛+2 ) = (𝑛+1)(𝑛+2)
0

9 8 √𝑥
Evaluate: ∫2 dx
√10−𝑥+√𝑥
Sol. Let 𝐼 = ∫2
8 √𝑥
dx……………………..(1)
√10−𝑥+√𝑥
𝑏 𝑏
By using property∫𝑎 𝑓(𝑥)𝑑𝑥 = ∫𝑎 𝑓(𝑎 + 𝑏 − 𝑥)𝑑𝑥
8 √8+2−𝑥
𝐼 = ∫2 dx
√10−(8+2−𝑥)+√8+2−𝑥
8 √10−𝑥
𝐼= ∫2 √𝑥+√10−𝑥dx……………………………(2)
adding (1) & (2)
8
2𝐼 = ∫2 1. 𝑑𝑥 = 6  I =3
10 𝜋/2
Evaluate: ∫
−𝜋/2
𝑠𝑖𝑛2 𝑥 𝑑𝑥
𝜋
Sol.
Let I= ∫ 2𝜋 𝑠𝑖𝑛2 𝑥 𝑑𝑥 As 𝑠𝑖𝑛2 (−𝑥) = (sin(−𝑥))2 = (−𝑠𝑖𝑛 𝑥)2= sin2 𝑥, therefore, sin2 𝑥 is

2
an even function.
𝑎 𝑎
It is known that if f(x) is an even function, then ∫−𝑎 𝑓(𝑥) 𝑑𝑥 = 2 ∫0 𝑓(𝑥) 𝑑𝑥
𝜋 𝜋 𝜋
1−𝑐𝑜𝑠2𝑥
I= 2 ∫0 𝑠𝑖𝑛 𝑥 𝑑𝑥
2 2
=2 ∫0 2 𝑑𝑥 =∫0 (1 − 𝑐𝑜𝑠2𝑥)𝑑𝑥
2
2

𝑠𝑖𝑛2𝑥 𝜋/2 𝜋 𝜋 𝜋
= [𝑥 − ] = − 𝑠𝑖𝑛𝜋 = − 0 =
2 0 2 2 2
𝜋
11 3 √𝑡𝑎𝑛 𝑥
Evaluate: ∫ 𝜋 𝑑𝑥
1+ √𝑡𝑎𝑛 𝑥
6
𝜋
Sol. √tan 𝑥
Let I=∫𝜋3 1+ tan 𝑥 𝑑𝑥………………………………….(1)
√ 6
π π
3 dx dx
I= ∫ π = ∫π3 1+
π π
6 1+√tan( + −x) 6
√cot x
6 3
𝜋
3 √tan 𝑥
 Ι=∫ 𝜋 𝑑𝑥 …………………………(2)
1+ √tan 𝑥
6
adding (1) & (2)
𝜋 𝜋
1+√tan 𝑥
 2I = ∫𝜋3 1+ 𝑑𝑥 ⇒ 2I = ∫𝜋3 1. 𝑑𝑥
6
√tan 𝑥 6
𝜋 𝜋
 2I = 6 ⇒ I = 12
π
12 cosx
Evaluate: ∫02 dx
(1+sinx)(2+sinx)

20
π
Sol. cosx
Let I=∫02 (1+sinx)(2+sinx) dx…………………..(1)
Put 𝑠𝑖𝑛𝑥 = 𝑡 ⟹ 𝑐𝑜𝑠𝑥𝑑𝑥 = 𝑑𝑡
𝜋
Also when 𝑥 = 0, 𝑡 = 0 𝑎𝑛𝑑 𝑤ℎ𝑒𝑛 𝑥 = 2 , 𝑡 = 1
(1) Reduces to
1 1
𝐼=∫0 (1+𝑡)(2+𝑡) 𝑑𝑡
Using partial fractions
1
1 1
𝐼=∫ { − } 𝑑𝑡 = [log|1 + 𝑡| − log|2 + 𝑡|]10
0 1+𝑡 2+𝑡
4
= [𝑙𝑜𝑔2 − 𝑙𝑜𝑔3] − [𝑙𝑜𝑔1 − 𝑙𝑜𝑔2] = log ( )
3
𝜋
13 2 𝑥 + 𝑠𝑖𝑛 𝑥
𝐸𝑣𝑎𝑙𝑢𝑎𝑡𝑒: ∫ 𝑑𝑥
0 1 + 𝑐𝑜𝑠 𝑥
𝜋
Sol. 𝑥+𝑠𝑖𝑛 𝑥
𝐿𝑒𝑡 𝐼 = ∫0 2 𝑑𝑥
1+𝑐𝑜𝑠 𝑥

𝜋 𝑥 𝑥 𝜋 𝜋
𝑥+2 𝑠𝑖𝑛2 𝑐𝑜𝑠2 2𝑥 𝑥 𝑥
I= 2
∫0 2 𝑐𝑜𝑠2𝑥 𝑑𝑥 = ∫0 2 𝑠𝑒𝑐2 2 𝑑𝑥 + ∫02 𝑡𝑎𝑛 2 𝑑𝑥
2
Using by parts for 1st integral, we get
𝜋 𝜋
𝑥 𝜋⁄2 𝑥 𝑥
= [𝑥. 𝑡𝑎𝑛 2] − ∫02 𝑡𝑎𝑛 2 𝑑𝑥 +∫02 𝑡𝑎𝑛 2 𝑑𝑥
0
𝑥 𝜋⁄2 𝜋 𝜋 𝜋
= [𝑥. 𝑡𝑎𝑛 2] = 2 (tan ( 4 )) − 𝑡𝑎𝑛0 =
0 2
𝜋
14 3 + 5 𝑠𝑖𝑛 𝑥 2
𝐸𝑣𝑎𝑙𝑢𝑎𝑡𝑒: ∫ 𝑙𝑜𝑔 ( ) 𝑑𝑥
0 3 + 5 𝑐𝑜𝑠 𝑥
𝜋
Sol. 3+5 𝑠𝑖𝑛 𝑥
Let I= ∫02 𝑙𝑜𝑔 (3+5 𝑐𝑜𝑠 𝑥) 𝑑𝑥…………………………(1)
𝜋 𝜋
3+5 𝑠𝑖𝑛( −𝑥) 𝑎 𝑎
I= ∫02 𝑙𝑜𝑔 ( 2
𝜋 ) 𝑑𝑥 [By using property∫0 𝑓(𝑥)𝑑𝑥 = ∫0 𝑓(𝑎 − 𝑥)𝑑𝑥]
3+5 𝑐𝑜𝑠( −𝑥)
2
𝜋
3+5 cos 𝑥
𝐼 = ∫02 𝑙𝑜𝑔 ( 3+5 sin 𝑥 ) 𝑑𝑥……………………………(2)
a
Now Using 𝑙𝑜𝑔 = 𝑙𝑜𝑔 𝑎 – 𝑙𝑜𝑔 𝑏 and adding (1) & (2)
b
we get, 2I = 0 so, 𝐼 = 0
15 8
𝐸𝑣𝑎𝑙𝑢𝑎𝑡𝑒: ∫ |𝑥 − 5| 𝑑𝑥
2
Sol. Let 𝐼 =
8
∫2 |𝑥 −
5
5| 𝑑𝑥 = ∫2 |𝑥 − 5| 𝑑𝑥 + ∫5 |𝑥 − 5| 𝑑𝑥
8

5 8
= ∫ −(𝑥 − 5) 𝑑𝑥 + ∫ (𝑥 − 5) 𝑑𝑥
2 5
5 8
𝑥2 𝑥2
= [− + 5𝑥] + [ − 5𝑥] = 9
2 2
2 5
Level III (4 Marks)
1 4
Evaluate ∫1 |𝑥 − 1| +|𝑥 − 2| + |𝑥 − 3| dx.
Sol. 𝐿𝑒𝑡 𝑓(𝑥) = [|𝑥 − 1| + |𝑥 − 2| + |𝑥 − 3|],
𝑓(𝑥) = [|𝑥 − 1| + |𝑥 − 2| + |𝑥 − 3|]

21
(𝑥 − 1) − (𝑥 − 2) − (𝑥 − 3) = 4 − 𝑥, 𝑖𝑓 1 < 𝑥 < 2
= { (𝑥 − 1) + (𝑥 − 2) − (𝑥 − 3) = 𝑥, 𝑖𝑓 2 < 𝑥 < 3
(𝑥 − 1) + (𝑥 − 2) + (𝑥 − 3) = 3𝑥 − 6, 𝑖𝑓 3 < 𝑥 < 4
𝑏 𝑐 𝑑 𝑏
𝑊𝑒 𝑘𝑛𝑜𝑤 𝑡ℎ𝑎𝑡, ∫ 𝑓(𝑥)𝑑𝑥 = ∫ 𝑓(𝑥)𝑑𝑥 + ∫ 𝑓(𝑥)𝑑𝑥 + ∫ 𝑓(𝑥)𝑑𝑥 ,
𝑎 𝑎 𝑐 𝑑
where 𝑎 < 𝑐 < 𝑑 < 𝑏
4 2 3
So ∫1 [|𝑥 − 1| + |𝑥 − 2| + |𝑥 − 3|]𝑑𝑥 = ∫1 [|𝑥 − 1| + |𝑥 − 2| + |𝑥 − 3|]𝑑𝑥 + ∫2 [|𝑥 −
4
1| + |𝑥 − 2| + |𝑥 − 3|]𝑑𝑥 + ∫3 [|𝑥 − 1| + |𝑥 − 2| + |𝑥 − 3|]𝑑𝑥
2 3 4
= ∫ (4 − 𝑥)𝑑𝑥 + ∫ 𝑥𝑑𝑥 + ∫ (3𝑥 − 6)𝑑𝑥
1 2 3
𝑥2 𝑥2 3 3𝑥 2
= (4𝑥 − )2 +( 2 )2 +( 2 − 6𝑥)43
2 1
1 9 27
= (8 − 2) − (4 − 2) + (2 − 2) + (24 − 24) − ( 2 − 18)
19
after simplifying, we get = 2
2 Evaluate:
𝜋/4
∫0 log ( 1 + tan 𝑥)𝑑𝑥
Sol. I = ∫0
𝜋/4
log ( 1 + tan 𝑥)𝑑𝑥
𝜋
𝑎 𝑎
I=∫04 log {1 + tan (𝜋/4 − 𝑥)}𝑑𝑥; by property∫0 𝑓(𝑥)𝑑𝑥 = ∫0 𝑓(𝑎 − 𝑥)𝑑𝑥
𝜋/4 1−𝑡𝑎𝑛 𝑥 𝜋/4 2
I =∫0 log (1 + 1+𝑡𝑎𝑛 𝑥) 𝑑𝑥 = ∫0 log (1+𝑡𝑎𝑛𝑥 ) 𝑑𝑥
𝜋
𝜋 𝜋 log 2
I= ∫04 log 2dx − 𝐼 ⇒ 2I =log 2. 4 ⇒ 𝐼 = .
8

3 Evaluate ∫0
𝜋 𝑥𝑡𝑎𝑛𝑥
𝑑𝑥
𝑠𝑒𝑐𝑥+tan 𝑥

Sol. Let I= ∫0
𝜋 𝑥𝑡𝑎𝑛𝑥
𝑑𝑥…………………….(1)
𝑠𝑒𝑐𝑥+tan 𝑥
𝜋 (𝜋−𝑥) 𝑡𝑎𝑛(𝜋−𝑥) 𝑎 𝑎
I=∫0 𝑑𝑥 (by using property, ∫0 𝑓(𝑥)𝑑𝑥 = ∫0 𝑓(𝑎 − 𝑥)𝑑𝑥)
𝑠𝑒𝑐(𝜋−𝑥)+𝑡𝑎𝑛(𝜋−𝑥)
𝜋 −(𝜋−𝑥) 𝑡𝑎𝑛 𝑥 𝜋 (𝜋−𝑥) 𝑡𝑎𝑛 𝑥
𝐼 = ∫0 𝑑𝑥 = ∫0 𝑑𝑥………………(2)
−(𝑠𝑒𝑐 𝑥+𝑡𝑎𝑛 𝑥) (𝑠𝑒𝑐 𝑥+𝑡𝑎𝑛 𝑥)
Adding (1) & (2) we get
𝜋 𝜋
𝜋. 𝑡𝑎𝑛𝑥 𝜋. 𝑠𝑖𝑛𝑥
2𝐼 = ∫ 𝑑𝑥 = ∫ 𝑑𝑥
0 𝑠𝑒𝑐𝑥 + tan 𝑥 0 1 + sin 𝑥
𝜋 𝜋
𝑠𝑖𝑛𝑥(1 − 𝑠𝑖𝑛𝑥) 𝑠𝑖𝑛𝑥 − 𝑠𝑖𝑛2 𝑥
= 𝜋∫ 𝑑𝑥 = 𝜋 ∫ 𝑑𝑥
0 (1 + 𝑠𝑖𝑛 𝑥)(1 − 𝑠𝑖𝑛𝑥) 0 𝑐𝑜𝑠 2 𝑥
𝜋 𝜋
= 𝜋 ∫ (𝑠𝑒𝑐 𝑥 𝑡𝑎𝑛 𝑥 − tan2 𝑥)𝑑𝑥 = 𝜋 ∫ (𝑠𝑒𝑐 𝑥 𝑡𝑎𝑛 𝑥 − 𝑠𝑒𝑐 2 𝑥 + 1)𝑑𝑥
0 0
= 𝜋[𝑠𝑒𝑐𝑥 − 𝑡𝑎𝑛𝑥 + 𝑥]𝜋0
𝜋
= 𝜋{(𝑠𝑒𝑐𝜋 − 𝑡𝑎𝑛𝜋 + 𝜋) − (𝑠𝑒𝑐 0 − 𝑡𝑎𝑛0 + 0) = 𝜋(−2 + 𝜋) ⇒𝐼= (𝜋 − 2)
𝜋
2
4
Evaluate: 𝐼 = ∫02 𝑙𝑜𝑔𝑠𝑖𝑛 𝑥 𝑑𝑥
Sol. We have
𝜋
𝐼 = ∫02 𝑙𝑜𝑔𝑠𝑖𝑛 𝑥 𝑑𝑥 …………………………..(1)

22
𝜋
𝜋 𝑎 𝑎
= ∫02 𝑙𝑜𝑔𝑐𝑜𝑠( 2 − 𝑥) 𝑑𝑥 [𝑈𝑠𝑖𝑛𝑔 ∫0 𝑓(𝑥)𝑑𝑥 = ∫0 𝑓(𝑎 − 𝑥)𝑑𝑥 ]
𝜋
= ∫02 𝑙𝑜𝑔𝑐𝑜𝑠 𝑥 𝑑𝑥 ……………………………(2)
Adding (1) & (2), we get
𝜋 𝜋
2𝑠𝑖𝑛𝑥.𝑐𝑜𝑠𝑥
2I= ∫02 𝑙𝑜𝑔𝑠𝑖𝑛𝑥. 𝑐𝑜𝑠 𝑥 𝑑𝑥 == ∫02 log ( ) 𝑑𝑥
2
𝜋 𝜋
2 2
= ∫0 𝑙𝑜𝑔𝑠𝑖𝑛2𝑥. 𝑑𝑥 − 𝑙𝑜𝑔2 ∫0 1 𝑑𝑥
𝜋
𝜋 𝜋
= ∫02 𝑙𝑜𝑔𝑠𝑖𝑛2𝑥. 𝑑𝑥 − 2 𝑙𝑜𝑔2 = I 1 − 2 𝑙𝑜𝑔2 ……………….(3)
𝜋
Put 2𝑥 = 𝑡 in I 1 we get, 2𝑑𝑥 = 𝑑𝑡, when 𝑥 = 0, 𝑡 = 0 and when 𝑥 = , 𝑡 = 𝜋
2
1 𝜋
I 1 = ∫0 log 𝑠𝑖𝑛𝑡 𝑑𝑡 Now as sin(𝜋 − 𝑡) = 𝑠𝑖𝑛𝑡
2
𝜋/2
1
𝐼 1 = 𝑋 2 ∫ log 𝑠𝑖𝑛𝑡 𝑑𝑡 ⇒ 𝐼1 = 𝐼
2 0
So, (3)  2𝐼 = 𝐼 − 2 𝑙𝑜𝑔2  I=− 2 𝑙𝑜𝑔2
𝜋 𝜋

5 2
Evaluate: ∫−1|𝑥 3 − 𝑥| 𝑑𝑥

Sol. 2
Evaluate: ∫−1|𝑥 3 − 𝑥| 𝑑𝑥
𝑥 3 − 𝑥, 𝑖𝑓𝑥 3 − 𝑥 ≥ 0
x3 − x = { , x3-x=0x = 0 and x = 1
−(𝑥 3 − 𝑥), 𝑖𝑓𝑥 3 − 𝑥 < 0
Here the intervals are (-1, 0), (0,1) and (1,2), since the limit is -1 to 2
Intervals x 3 − x =x (x-1) (x+1) function
(-1, 0) (-)(-)(+)=Positive x3 − x
(0, 1) (+)(-)(+)=Negative − (𝑥 3 − 𝑥)
(1, 2) (+)(+)(+)=Positive 𝑥3 − 𝑥
2 0 1 2
𝑆𝑜 ∫−1|𝑥 3 − 𝑥| dx = ∫−1(𝑥 3 − 𝑥)dx +∫0 −(𝑥 3 − 𝑥)dx + ∫1 (𝑥 3 − 𝑥)dx
0 1 2
𝑥4 𝑥2 𝑥2 𝑥4 𝑥4 𝑥2
=[ − ] +[ − ] +[ − ]
4 2 −1 2 4 0 4 2 1
1 1 1 1 16 4 1 1
= (0) − ( − ) + ( − ) − (0) + ( − ) − ( − )
4 2 2 4 4 2 4 2
1 1 1 3 11
= + +2+ = 2+ =
4 4 𝜋 4 4 4
6
Evaluate: ∫0 (2 𝑙𝑜𝑔𝑠𝑖𝑛 𝑥 − 𝑙𝑜𝑔𝑠𝑖𝑛 2 𝑥)𝑑𝑥
2
𝜋
Sol.
𝐿𝑒𝑡 𝐼 = ∫02 (2 𝑙𝑜𝑔𝑠𝑖𝑛 𝑥 − 𝑙𝑜𝑔𝑠𝑖𝑛 2 𝑥)𝑑𝑥

(Using 𝑚 𝑙𝑜𝑔𝑛 = 𝑙𝑜𝑔𝑛𝑚 & 𝑙𝑜𝑔𝑎 − 𝑙𝑜𝑔𝑏 = log (𝑎)


𝜋
𝑠𝑖𝑛2 𝑥
𝐼 = ∫02 𝑙𝑜𝑔 (2 𝑠𝑖𝑛 𝑥 𝑐𝑜𝑠 𝑥) 𝑑𝑥 𝑏

…………………….(1) (by property∫𝑎 𝑓(𝑥)𝑑𝑥 = ∫𝑎 𝑓(𝑎 − 𝑥)𝑑𝑥)


𝜋
𝑡𝑎𝑛 𝑥
𝐼 = ∫02 𝑙𝑜𝑔 ( ) 𝑑𝑥 0 0
2

𝜋
𝜋 𝑡𝑎𝑛( −𝑥)
𝜋
𝑐𝑜𝑡 𝑥 ………………………(2)
𝐼 = ∫0 𝑙𝑜𝑔 (2 2
) 𝑑𝑥 = ∫02 𝑙𝑜𝑔 ( ) 𝑑𝑥
2 2

Adding (1) & (2), we get

23
𝜋 𝜋
𝑡𝑎𝑛𝑥 𝑐𝑜𝑡𝑥 1 𝜋 1
2𝐼 = ∫02 log ( . ) 𝑑𝑥 = ∫02 log (4) 𝑑𝑥 = 2 log (4)
2 2
𝜋 1 𝜋
𝐼 = 4 𝑙𝑜𝑔 (4)=− 2 𝑙𝑜𝑔2
𝜋
7 𝑥 𝑠𝑖𝑛 𝑥
𝐸𝑣𝑎𝑙𝑢𝑎𝑡𝑒: ∫ 𝑑𝑥
0 1 + 𝑠𝑖𝑛 𝑥

Sol. Let I =∫0


𝜋 𝑥 𝑠𝑖𝑛 𝑥
𝑑𝑥 ………….(1)
1+𝑠𝑖𝑛 𝑥
𝜋 (𝜋−𝑥) 𝑠𝑖𝑛(𝜋−𝑥) 𝑎 𝑎
=∫0 𝑑𝑥 (by property∫0 𝑓(𝑥)𝑑𝑥 = ∫0 𝑓(𝑎 − 𝑥)𝑑𝑥)
1+𝑠𝑖𝑛(𝜋−𝑥)
𝜋 (𝜋−𝑥) 𝑠𝑖𝑛 𝑥
=∫0 𝑑𝑥 ………(2)
1+𝑠𝑖𝑛 𝑥

adding (1) & (2)

 2I =∫ 𝜋 𝑥 𝑠𝑖𝑛 𝑥
0 1+𝑠𝑖𝑛 𝑥
𝜋 (𝜋−𝑥) 𝑠𝑖𝑛 𝑥
𝑑𝑥 +∫0 1+𝑠𝑖𝑛 𝑥 𝑑𝑥 =∫0
𝜋 𝜋 𝑠𝑖𝑛 𝑥
1+𝑠𝑖𝑛 𝑥
𝑑𝑥

=  ∫0 𝑑𝑥 =  ∫0
𝜋 𝑠𝑖𝑛 𝑥 𝜋 𝑠𝑖𝑛 𝑥(1−𝑠𝑖𝑛 𝑥)
𝑑𝑥
1+𝑠𝑖𝑛 𝑥 1−𝑠𝑖𝑛2 𝑥

=  ∫0 𝑑𝑥 =  ∫0 (𝑠𝑒𝑐 𝑥 . 𝑡𝑎𝑛 𝑥 − 𝑡𝑎𝑛2 𝑥)𝑑𝑥


𝜋 𝑠𝑖𝑛 𝑥(1−𝑠𝑖𝑛 𝑥) 𝜋
𝑐𝑜𝑠2 𝑥
𝜋
= 𝜋 ∫ (𝑠𝑒𝑐 𝑥 𝑡𝑎𝑛 𝑥 − 𝑠𝑒𝑐 2 𝑥 + 1)𝑑𝑥 = 𝜋[𝑠𝑒𝑐𝑥 − 𝑡𝑎𝑛𝑥 + 𝑥]𝜋0
0
𝜋
= 𝜋{(𝑠𝑒𝑐𝜋 − 𝑡𝑎𝑛𝜋 + 𝜋) − (𝑠𝑒𝑐 0 − 𝑡𝑎𝑛0 + 0) = 𝜋(−2 + 𝜋) ⇒𝐼= (𝜋 − 2)
2
8 1
𝐸𝑣𝑎𝑙𝑢𝑎𝑡𝑒: 𝐼 = ∫ 𝐶𝑜𝑡 −1 (1 − 𝑥 + 𝑥 2 )𝑑𝑥
0
Sol. 1
𝐿𝑒𝑡 𝐼 = ∫ 𝐶𝑜𝑡 −1 (1 − 𝑥 + 𝑥 2 )𝑑𝑥
0
1 −1 (1−𝑥)+𝑥 1
∫0 𝑡𝑎𝑛 { } 𝑑𝑥 (as cot −1 𝑥 = tan−1( ) and 1 − 𝑥 + 𝑥 2 = 1 − 𝑥(1 − 𝑥) )
1−𝑥(1−𝑥) 𝑥
1 1 𝑥+𝑦
= ∫0 𝑡𝑎𝑛−1 ( 1 − 𝑥)𝑑𝑥 + ∫0 𝑡𝑎𝑛−1 𝑥 𝑑𝑥 [𝐴𝑠 tan−1 1−𝑥𝑦 = tan−1 𝑥 + tan−1 𝑦]
1 1
= ∫0 𝑡𝑎𝑛−1 [1 −( 1 − 𝑥)]𝑑𝑥 + ∫0 𝑡𝑎𝑛−1 𝑥 𝑑𝑥
𝑎 𝑎
(by using the property∫0 𝑓(𝑥)𝑑𝑥 = ∫0 𝑓(𝑎 − 𝑥)𝑑𝑥)
1 1 1
−1 −1 −1
= ∫ 𝑡𝑎𝑛 𝑥 𝑑𝑥 + ∫ 𝑡𝑎𝑛 𝑥 𝑑𝑥 = 2 ∫ 𝑡𝑎𝑛 𝑥 𝑑𝑥
0 0 0
1
= 2 ∫0 𝑡𝑎𝑛−1 𝑥 . 1 𝑑𝑥 On integrating by parts, we get
1
1
= 2. [{tan−1 𝑥. 𝑥}01 − ∫ 2
. 𝑥 𝑑𝑥]
0 1+𝑥
1 𝜋 1 𝜋
= 2[𝑥𝑡𝑎𝑛−1 𝑥 − log(1 + 𝑥 2 )]10 = 2 [ − 𝑙𝑜𝑔2] ⇒ 𝐼 = − 𝑙𝑜𝑔 2
2 4 2 2
9 𝜋/4
𝐸𝑣𝑎𝑙𝑢𝑎𝑡𝑒: ∫ (√𝑡𝑎𝑛 𝑥 + √𝑐𝑜𝑡 𝑥)𝑑𝑥
0
Sol. Let I
𝜋/4
=∫0 (√𝑡𝑎𝑛 𝑥 + √𝑐𝑜𝑡 𝑥)𝑑𝑥 = ∫0
𝜋/4 𝑠𝑖𝑛𝑥+𝑐𝑜𝑠𝑥
𝑑𝑥
√𝑠𝑖𝑛𝑥.𝑐𝑜𝑠𝑥
𝑠𝑖𝑛 𝑥+𝑐𝑜𝑠 𝑥 𝑑𝑡
Consider I 1 =∫ 𝑑𝑥 =∫ 2
√𝑠𝑖𝑛 𝑥.𝑐𝑜𝑠 𝑥
√1−𝑡
2

24
𝑤ℎ𝑒𝑟𝑒 𝑡 = 𝑠𝑖𝑛𝑥 – 𝑐𝑜𝑠𝑥  𝑑𝑡 = (𝑠𝑖𝑛𝑥 + 𝑐𝑜𝑠𝑥)𝑑𝑥
𝑡 2 = 1 − 𝑠𝑖𝑛( 2𝑥) using this substitution and solving we get
So 𝐼 1 = √2 sin−1 (𝑠𝑖𝑛𝑥 − 𝑐𝑜𝑠𝑥)
𝜋
𝜋 √2
𝑆𝑜 𝐼 = [√2 sin−1 (𝑠𝑖𝑛𝑥 − 𝑐𝑜𝑠𝑥)]04 = √2 {sin−1 0 − sin−1(−1)} = √2 [0 + 2 ] =𝜋 2.

10 𝑥2 2
Evaluate: 𝐼 = ∫ 𝑥
𝑑𝑥
−2 1 + 5

Sol. 2 𝑥2
Let 𝐼 = ∫−2 1+5𝑥 𝑑𝑥 ……………………………….(1)
𝑏 𝑏
[using ∫𝑎 𝑓(𝑥)𝑑𝑥 = ∫𝑎 𝑓(𝑎 + 𝑏 − 𝑥)𝑑𝑥]
2 (−2+2−𝑥)2 2 𝑥2
𝐼 = ∫−2 1+5(−2+2−𝑥) (𝑑𝑥) = ∫−2 1+5−𝑥 𝑑𝑥
2 𝑥2 2 5𝑥 .𝑥 2
= ∫−2 1 𝑑𝑥 = ∫−2 1+5𝑥 𝑑𝑥 …………………(2)
1+ 𝑥
5
Adding equations (1) and (2), we get
2 𝑥2 5𝑥 .𝑥 2 2 𝑥 2 (1+5𝑥 )
2I=∫−2[1+5𝑥 + 1+5𝑥 ]𝑑𝑥 ⇒2I=∫−2[ ]𝑑𝑥
1+5𝑥
2 2 𝑥3 23 0 8 16 8
2I = ∫−2 𝑥 2 𝑑𝑥 = 2 ∫0 𝑥 2 𝑑𝑥 = 2( 3 )20 =2 ( 3 − 3) = 2 (3 − 0) = So, I = 3
3
11 𝜋/4
𝑠𝑖𝑛𝑥 + 𝑐𝑜𝑠𝑥
𝐸𝑣𝑎𝑙𝑢𝑎𝑡𝑒: ∫ 𝑑𝑥
0 9 + 16𝑠𝑖𝑛 2𝑥
Sol. 𝜋/4
𝑠𝑖𝑛𝑥 + 𝑐𝑜𝑠𝑥
𝐿𝑒𝑡 𝐼 = ∫ 𝑑𝑥
0 9 + 16𝑠𝑖𝑛 2𝑥
Put 𝑠𝑖𝑛𝑥 − 𝑐𝑜𝑠𝑥 = 𝑡 ⟹ (𝑐𝑜𝑠𝑥 + 𝑠𝑖𝑛𝑥)𝑑𝑥 = 𝑑𝑡
𝜋
When 𝑥 = 0, 𝑡 = −1 & 𝑤ℎ𝑒𝑛 𝑥 = 4 , 𝑡 = 0
⟹ (𝑠𝑖𝑛𝑥 − 𝑐𝑜𝑠𝑥)2 = 𝑡 2 ⟹ 1 − 𝑠𝑖𝑛2𝑥 = 𝑡 2 ⟹ 𝑠𝑖𝑛2𝑥 = 1 − 𝑡 2
0
𝑑𝑡
∴𝐼=∫ 2
−1 9 + 16(1 − 𝑡 )
0 0 0
𝑑𝑡 𝑑𝑡 𝑑𝑡
=∫ 2
= ∫ 2
== ∫ 2 2
−1 9 + 16 − 16𝑡 ) −1 25 − 16𝑡 −1 5 − (4𝑡)
1 1 5 + 4𝑡 0 1 1
= [ log { }]−1 = [𝑙𝑜𝑔1 − log ( )]
4 2.5 5 − 4𝑡 40 9
1 1 1 1
= [− log ( )] = 𝑙𝑜𝑔9 = 𝑙𝑜𝑔3
40 9 40 20
12 2 1 1
Evaluate: ∫1 (𝑥 − 2𝑥 2) 𝑒 2𝑥 𝑑𝑥
2 1 1
𝐿𝑒𝑡𝐼 = ∫1 (𝑥 − 2𝑥 2 ) 𝑒 2𝑥 𝑑𝑥,

Put 2x=t ⟹ 2𝑑𝑥 = 𝑑𝑡 , Also when 𝑥 = 1, 𝑡 = 2 𝑎𝑛𝑑 𝑥 = 2, 𝑡 = 4

2 1 1 1 4 2 2 4 1 1
∴ ∫1 (𝑥 − 2𝑥 2 ) 𝑑𝑥 = 2 ∫2 ( 𝑡 − 2) 𝑒𝑡 𝑑𝑥 = ∫2 ( 𝑡 − 2) 𝑒𝑡 𝑑𝑡
𝑡 𝑡

1 1
Now let 𝑓(𝑡) = 𝑡 , then 𝑓′(𝑡) = 𝑡 2

25
4 14
𝐼=∫ (𝑓(𝑡) + 𝑓′(𝑡))𝑒𝑡 𝑑𝑡 = [𝑒 𝑡 𝑓(𝑡)]42 𝑡
⟹ [𝑒 . ]
2 𝑡 2

𝑒4 𝑒2 2
(𝑒 2 − 2)
= − ⟹𝐼=𝑒
4 2 4
13 5
Evaluate: ∫−5|𝑥 + 2| 𝑑𝑥
Sol. 5
Let 𝐼 = ∫−5|𝑥 + 2| 𝑑𝑥
𝑏 𝑐 𝑏
Using the property ∫𝑎 𝑓(𝑥) = ∫𝑎 𝑓(𝑥) + ∫𝑐 𝑓(𝑥) 𝑤ℎ𝑒𝑟𝑒 𝑎 < 𝑐 < 𝑏
−2 5
𝐼=∫ −(𝑥 + 2)𝑑𝑥 + ∫ (𝑥 + 2)𝑑𝑥
−5 −2
2 −2 2 5
𝑥 𝑥
= − [ + 2𝑥] + [ + 2𝑥]
2 −5
2 −2
25 25
= − [2 − 4 − + 10] + [ + 10 − 2 + 4] = 29
2 2
Miscellaneous Exercise
𝜋
1 2 𝑠𝑖𝑛𝑥 − 𝑐𝑜𝑠𝑥
𝐸𝑣𝑎𝑙𝑢𝑎𝑡𝑒: ∫ 𝑑𝑥
0 1 + 𝑠𝑖𝑛𝑥 𝑐𝑜𝑠𝑥
2 3
𝜋/2 𝑠𝑖𝑛2 𝑥
Evaluate: ∫
0 3 3 𝑑𝑥
𝑠𝑖𝑛 2 𝑥+𝑐𝑜𝑠 2 𝑥
𝜋
3 𝑥 𝑡𝑎𝑛𝑥
𝐸𝑣𝑎𝑙𝑢𝑎𝑡𝑒: ∫ 𝑑𝑥
0 sec 𝑥 + tan 𝑥
4 8
𝐸𝑣𝑎𝑙𝑢𝑎𝑡𝑒: ∫ |𝑥 − 5|𝑑𝑥
2
𝜋 𝑥
5 𝐸𝑣𝑎𝑙𝑢𝑎𝑡𝑒: ∫0 𝑑𝑥
𝑎2 𝑐𝑜𝑠2 𝑥+𝑏2 sin2 𝑥

6 𝜋/2
𝑥𝑠𝑖𝑛𝑥𝑐𝑜𝑠𝑥
𝐸𝑣𝑎𝑙𝑢𝑎𝑡𝑒: ∫ 𝑑𝑥
0 sin4 𝑥 + 𝑐𝑜𝑠 4 𝑥
7 𝜋/2
𝐸𝑣𝑎𝑙𝑢𝑎𝑡𝑒: ∫−𝜋/2 𝑠𝑖𝑛2 𝑥 𝑑𝑥
8 𝜋/4
𝐸𝑣𝑎𝑙𝑢𝑎𝑡𝑒: ∫0 𝑡𝑎𝑛𝑥 𝑑𝑥
𝜋
9 2 √1+𝑐𝑜𝑠𝑥
𝐸𝑣𝑎𝑙𝑢𝑎𝑡𝑒: ∫ 𝜋 5 𝑑𝑥 [Hint: Multiply Nr and Dr by √1 − 𝑐𝑜𝑠𝑥 and use
3 (1−𝑐𝑜𝑠𝑥)2
substitution 1 − 𝑐𝑜𝑠𝑥 = 𝑡]
10 𝜋/3
𝑠𝑖𝑛𝑥 + 𝑐𝑜𝑠𝑥
𝐸𝑣𝑎𝑙𝑢𝑎𝑡𝑒: ∫ 𝑑𝑥
𝜋/6 √𝑠𝑖𝑛2𝑥
11 1 𝑙𝑜𝑔𝑥
𝐸𝑣𝑎𝑙𝑢𝑎𝑡𝑒: ∫0 √1−𝑥 2 𝑑𝑥 [Hint: Put 𝑥 = 𝑠𝑖𝑛𝑥]
12 3.5
𝐸𝑣𝑎𝑙𝑢𝑎𝑡𝑒: ∫0.2 [𝑥]𝑑𝑥 [use definition of greatest integer function]
𝜋
13 2𝑠𝑖𝑛𝑥 2
𝐸𝑣𝑎𝑙𝑢𝑎𝑡𝑒: ∫ 𝑠𝑖𝑛𝑥 𝑑𝑥
0 2 + 2𝑐𝑜𝑠𝑥
𝜋
14 sin2 𝑥
2
𝐸𝑣𝑎𝑙𝑢𝑎𝑡𝑒: ∫ 𝑑𝑥
0 𝑠𝑖𝑛𝑥 + 𝑐𝑜𝑠𝑥

26
15 2
𝐸𝑣𝑎𝑙𝑢𝑎𝑡𝑒: ∫ {|𝑥 + 1| + |𝑥| + |𝑥 − 1|}𝑑𝑥
−1
𝜋
16 2 1 − 𝑠𝑖𝑛2𝑥
𝐸𝑣𝑎𝑙𝑢𝑎𝑡𝑒: ∫ 𝑒 2𝑥 ( ) 𝑑𝑥
𝜋 1 − 𝑐𝑜𝑠2𝑥
4
17 3
2
𝐸𝑣𝑎𝑙𝑢𝑎𝑡𝑒: ∫ |𝑥. sin 𝜋𝑥 |𝑑𝑥
−1
18 3
2
𝐸𝑣𝑎𝑙𝑢𝑎𝑡𝑒: ∫ |𝑥. cos 𝜋𝑥 |𝑑𝑥
0
𝜋
19 𝑥 sin 𝑥
𝐸𝑣𝑎𝑙𝑢𝑎𝑡𝑒: ∫ 𝑑𝑥
0 1 + cos2 𝑥
𝜋
20 3 1
𝐸𝑣𝑎𝑙𝑢𝑎𝑡𝑒: ∫ 𝑑𝑥
𝜋 1 + √cot 𝑥
6
21 2
𝐸𝑣𝑎𝑙𝑢𝑎𝑡𝑒: ∫ |𝑥 2 + 2𝑥 − 3|𝑑𝑥
0
22 1
𝐸𝑣𝑎𝑙𝑢𝑎𝑡𝑒: ∫ 𝑥|𝑥|𝑑𝑥
−1
23 1
𝐸𝑣𝑎𝑙𝑢𝑎𝑡𝑒: ∫ [2𝑥]𝑑𝑥
0
𝜋
24 cos2 𝑥
𝐸𝑣𝑎𝑙𝑢𝑎𝑡𝑒: ∫04 cos2 𝑥+4 sin2 𝑥 𝑑𝑥 [ Make proper rational fraction and then divide Nr
and Dr by cos 2 𝑥]
25 1
𝐸𝑣𝑎𝑙𝑢𝑎𝑡𝑒: ∫ |2𝑥 − 1|𝑑𝑥
1
4
𝜋
26
𝐸𝑣𝑎𝑙𝑢𝑎𝑡𝑒: ∫ log (1 + cos 𝑥) 𝑑𝑥
0
𝜋
27 cos 𝑥 𝑏 𝑏
𝐸𝑣𝑎𝑙𝑢𝑎𝑡𝑒: ∫ 2𝜋 1+𝑒 𝑥 𝑑𝑥 [Hint: Apply ∫𝑎 𝑓(𝑥)𝑑𝑥 = ∫𝑎 𝑓(𝑎 + 𝑏 − 𝑥)𝑑𝑥

2
𝜋
28 sin 2𝑥
𝐸𝑣𝑎𝑙𝑢𝑎𝑡𝑒: ∫0 2 𝑑𝑥 [Hint: Divide Nr and Dr by cos4 𝑥 𝑎𝑛𝑑 𝑝𝑢𝑡 tan2 𝑥 = 𝑡]
sin4 𝑥+cos4 𝑥

Answers of Miscellaneous Exercise


1 𝐴𝑛𝑠: 0 8 1 15 Ans:
19 22 Ans: 0
𝐴𝑛𝑠: 𝑙𝑜𝑔2 2
2
2 𝜋 9 3 16 1 𝜋 23 1
𝐴𝑛𝑠: 𝐴𝑛𝑠: Ans:2 𝑒 2 Ans: 2
4 2
𝜋
3 Ans: 2 (𝜋 − 2) 10 Ans:2 sin−1
√3−1 17 Ans:
3𝜋+1 24 2
Ans: 3 tan−1 2 − 12
𝜋
2 𝜋2
𝜋
4 Ans: 9 11 Ans: − 2 𝑙𝑜𝑔2 18 5𝜋−2
Ans: 𝜋2 25 Ans: 16
5

5 Ans: 2𝑎𝑏
𝜋2 12 Ans: 2
9 19 𝜋2
Ans: 4 26 Ans: −𝜋 𝑙𝑜𝑔2
𝜋 𝜋
6 Ans: 16
𝜋2 13 Ans: 4 20 Ans: 12 27 Ans:1
7 𝜋 14 1 21 Ans: 4 28 Ans:
𝜋
𝐴𝑛𝑠: Ans: log |√2 + 1| 2
2 √2

27
APPLICATION OF INTEGRALS
BASIC CONCEPTS:
1. Area of the region bounded by the curve 𝑦 =
𝑓(𝑥) , the 𝑥-axis and the line 𝑥 = 𝑎 , 𝑥 = 𝑏 is
given by
𝑏 𝑏
∫𝑎 𝑦 𝑑𝑥 = ∫𝑎 𝑓(𝑥) 𝑑𝑥

2. If the curve lies below x-axis, then the area of the


region bounded by the curve 𝑦 =
𝑓(𝑥) , the 𝑥-axis and the line 𝑥 =
𝑎 , 𝑥 = 𝑏 is given by

𝑏
|∫𝑎 𝑦 𝑑𝑥|

3. Area of the region bounded by the curve 𝑥=


𝑔(𝑦), the 𝑦-axis and the line 𝑦 = 𝑐 , 𝑦 = 𝑑 is
given by
𝑑 𝑑
∫𝑐 𝑥 𝑑𝑦 = ∫𝑐 𝑔(𝑦) 𝑑𝑦

4 Area of the region bounded by the simple curve 𝑦 = 𝑓(𝑥)𝑎𝑛𝑑 𝑦 = 𝑔(𝑥) .


Required area = Area of ACBDA +Area of BPRQB
𝑐 𝑏
= ∫𝑎 [𝑓(𝑥) − 𝑔(𝑥)]𝑑𝑥 + ∫𝑐 [𝑓(𝑥) − 𝑔(𝑥)] 𝑑𝑥

KEY POINTS :
𝑥 𝑎2 𝑥
1. ∫ √𝑎2 − 𝑥 2 𝑑𝑥 = 2 √𝑎2 − 𝑥 2 + 2 sin−1 𝑎
2. Curve 𝑎𝑥 + 𝑏𝑦 = 𝑐 is a straight line.
3. Curve 𝑦 2 = 4𝑎𝑥 is a parabola with vertex (0,0) and axis is x-axis.
4. Curve 𝑥 2 = 4𝑎𝑦 is a parabola with vertex (0,0) and axis is y-axis.
5. Curve 𝑥 2 + 𝑦 2 =𝑎2 is a circle with center (0,0) and radius = a
6. Curve (𝑥 − 𝑎)2 + (𝑦 − 𝑏)2 =𝑐 2 is a circle with center (𝑎, 𝑏) and radius = c
7. Curve 𝑥 2 + 𝑦 2 + 2𝑔𝑥 + 2𝑓𝑦 + 𝑐 = 0 is a circle with center (-𝑔,- 𝑓)
and radius = √𝑔2 + 𝑓 2 − 𝑐

28
𝑥2 𝑦2
8. Curve + 𝑏2 = 1 , (𝑎 > 𝑏) is a ellipse with center (0,0)
𝑎2

Length of major axis =2𝑎 , Length of minor axis =2 𝑏


9. Circle and ellipse are symmetric in each quadrant.
10. Parabola is symmetric about its axis.
Ex.1 Find the area of the region bounded by the curve 𝑦 2 = 4𝑥 and the line 𝑥 = 4.
Sol. Curve 𝑦 2 = 4𝑥 … (1) is a parabola with vertex (0,0) and line 𝑥 = 4
Draw rough sketch
Shaded region OABO be the required area.
Parabola is symmetric about its axis.
Required area =2 area of OACO
4 4
= 2 ∫0 𝑦 𝑑𝑥 = 2 ∫0 2 𝑥1/2 𝑑𝑥 (use 1)
4
𝑥 3/2 8 64
= 4 ( 3/2 ) = 3 (43/2 − 0) = sq.unit
0 3

𝑥2 𝑦2
Ex.2 Using definite integration, find the area of the region 𝑎2 + 𝑏2 = 1.

𝑥2 𝑦2
Sol. Curve 𝑎2 + 𝑏2 = 1 is a ellipse with center (0,0)

Draw rough sketch, Shaded region be the required area. Ellipse is symmetric in
each quadratic.
Required area = 4 x area of OABO
𝑎 𝑎𝑏
=4∫0 𝑦 𝑑𝑥 = 4 ∫0 √𝑎2 −𝑥 2 𝑑𝑥
𝑎
𝑎
𝑏 𝑥 𝑎2 𝑥
=4 𝑎 (2 √𝑎2 − 𝑥 2 + sin−1 𝑎)
2 0

𝑏 𝑎 𝑎2 𝑎 𝑎2 0
=4 𝑎 (2 √𝑎2 − 𝑎2 + sin−1 𝑎 − 0 − sin−1 𝑎)
2 2

𝑏 𝑎2 𝑎2
=4 𝑎 (0 + sin−1 1 − 0 − sin−1 0)
2 2

𝑏 𝑎2 𝜋
=4 𝑎 ( 2 − 0) = 𝜋𝑎𝑏 𝑠𝑞. 𝑢𝑛𝑖𝑡
2

29
Ex.3 Find the area of the region bounded by the line 𝑦 = 3𝑥 + 2 , 𝑥 − 𝑎𝑥𝑖𝑠 and the ordinates
𝑥 = −1 𝑎𝑛𝑑 𝑥 = 1.
Sol. Draw rough sketch of line 𝑦 = 3𝑥 + 2 ,
𝑥 = −1 𝑎𝑛𝑑 𝑥 = 1
Shaded region be the required area.
Area ABCA lies blow x-axis and area ADEA above x-axis
−2/3 1
Required area = |∫−1 𝑦 𝑑𝑥| + ∫−2/3 𝑦 𝑑𝑥
−2/3 1
= |∫−1 (3𝑥 + 2) 𝑑𝑥| + ∫−2/3(3𝑥 + 2) 𝑑𝑥
−2/3 1
3𝑥 2 3𝑥 2 1 25 13
= |[ + 2𝑥] |+[ + 2𝑥] =6+ = 𝑠𝑞. 𝑢𝑛𝑖𝑡
2 −1 2 −2/3 6 3

Ex.4 Find the area of the region in the first quadrant enclosed by the 𝑥 − 𝑎𝑥𝑖𝑠 , 𝑡ℎ𝑒 𝑙𝑖𝑛𝑒 𝑦 =
𝑥 𝑎𝑛𝑑 𝑡ℎ𝑒 𝑐𝑢𝑟𝑣𝑒 𝑥 2 + 𝑦 2 = 32.

Sol. Given curve 𝑥 2 + 𝑦 2 = 32 ……(i) is a circle with center (0,0), radius=4√2 and the line 𝑦 =
𝑥 …………(ii) passes through (0,0)
Draw rough sketch, Shaded region be the required area.
Solving (i) and (ii)getting 𝑥 = 4 , 𝑦 = 4 the intersection
point of line and circle is B(4,4).
Required area = area of OBMO +area of BMAB
4 4√2
= ∫0 𝑦𝑙𝑖𝑛𝑒 𝑑𝑥 + ∫4 𝑦𝑐𝑖𝑟𝑐𝑙𝑒 𝑑𝑥
4 4√2
= ∫0 𝑥 𝑑𝑥 + ∫4 √32 − 𝑥 2 𝑑𝑥 (from
(i) and (ii)
4 4√2
𝑥2 𝑥 32 𝑥
= [ 2 ] + [2 √32 − 𝑥 2 + sin−1 4√2]
0 2 4

4√2 4 1
=8+[ × 0 + 16 sin−1 1] − [2 √32 − 16 + 16 sin−1 √2]
2
𝜋 𝜋
= 8 + 16 × 2 − 8 − 16 × 4 = 8𝜋 − 4𝜋 = 4𝜋 𝑠𝑞. 𝑢𝑛𝑖𝑡

Ex.5 Find the area bounded by the curve


{(𝑥, 𝑦): 𝑦 ≥ 𝑥 2 𝑎𝑛𝑑 𝑦 = |𝑥|}
Sol. Given curve is 𝑦 = 𝑥 2 …..(i) is parabola and
𝑥, 𝑥 > 0
𝑦 = |𝑥| = { ……….(ii)
−𝑥 , 𝑥 ≤ 0

30
Draw rough sketch, Shaded region be the
required area.
Both the curve are symmetric about Y- axis.
From curve (i) and (ii) 𝑥 = 0 𝑜𝑟 𝑥 = 1𝑜𝑟 𝑥 =
−1
The points of intersection of the curves
O(0,0),A(1,1) and B(-1,1)
Required area = Area of shaded region=2
area of OAPO
1 1
= 2 ∫0 (𝑦𝑙𝑖𝑛𝑒 − 𝑦𝑝𝑎𝑟𝑎𝑏𝑜𝑙𝑎 )𝑑𝑥 = 2 ∫0 𝑥 − 𝑥 2 )𝑑𝑥
1
𝑥2 𝑥3 1 1 1
= 2[2 − ] = 2 [2 − 3] = 3 𝑠𝑞. 𝑢𝑛𝑖𝑡
3 0

𝑥2 𝑦2 𝑥 𝑦
Ex.6 Find the area of the smaller region bounded by the curve + 36 = 1 and the line 2 + 6 =
4
1.
𝑥2 𝑦2
Sol. Curve + 36 = 1 ………..(i)is ellipse
4

Draw rough sketch, Shaded region be the required area


2 2
Required Area = ∫0 𝑦𝑒𝑙𝑙𝑖𝑝𝑠𝑒 𝑑𝑥 − ∫0 𝑦𝑙𝑖𝑛𝑒 𝑑𝑥

2 2
= 3 ∫0 √4 − 𝑥 2 𝑑𝑥 − ∫0 (6 − 3𝑥)𝑑𝑥

2
𝑥 4 𝑥 2 3𝑥 2
= 3 [2 √4 − 𝑥 2 + 2 sin−1 2] − [6𝑥 − ]
0 2 0

=3[0 + 2 sin−1 1 − 0 − 0] − [12 − 6] = 3𝜋 − 6 𝑠𝑞. 𝑢𝑛𝑖𝑡


Ex.7 CASE STUDY BASED : A farmer has a triangular shaped field. His son, a science student
observes the triangular field has three edges and can be drawn on a plain paper with three
lines given by the equations. 𝑦 = 2𝑥 − 2, 𝑦 = 4 − 𝑥 𝑎𝑛𝑑 2𝑦 = (𝑥 − 1)
On the basis of above information, answer the following questions.
(i)Find the corner points of triangular field.
(ii) Using integration, find the total area of triangular field.
Sol. Given equation are
1
𝑦 = 2𝑥 − 2 …….(i), 𝑦 = 4 − 𝑥 … … . (𝑖𝑖), 𝑦 = 2 (𝑥 − 1) … … . (𝑖𝑖𝑖)

31
To determine corner points of triangle field solve
equation in pairs.
From equation (𝑖) 𝑎𝑛𝑑 (𝑖𝑖) 𝑥 = 2 , 𝑦 =
2 𝑐𝑜𝑟𝑛𝑒𝑟 𝑝𝑜𝑖𝑛𝑡 𝑖𝑠 𝐵(2,2)
From equation (𝑖𝑖) 𝑎𝑛𝑑 (𝑖𝑖𝑖) 𝑥 = 3 , 𝑦 =
1 𝑐𝑜𝑟𝑛𝑒𝑟 𝑝𝑜𝑖𝑛𝑡 𝑖𝑠 𝐶 (3,1)
From equation (𝑖) 𝑎𝑛𝑑 (𝑖𝑖𝑖) 𝑥 = 1 , 𝑦 =
0 𝑐𝑜𝑟𝑛𝑒𝑟 𝑝𝑜𝑖𝑛𝑡 𝑖𝑠 𝐴(1,0)
2 3
Now area of triangle ABC=∫1 𝑦𝐴𝐵 𝑑𝑥 + ∫2 𝑦𝐵𝐶 𝑑𝑥 −
3
∫1 𝑦𝐴𝐶 𝑑𝑥
2( 3 31
=∫1 2𝑥 − 2)𝑑𝑥 + ∫2 (4 − 𝑥)𝑑𝑥 − ∫1 2 (𝑥 − 1)𝑑𝑥
2 3 3
𝑥2 𝑥2 1 𝑥2
=2 [ 2 − 𝑥] + [4𝑥 − ] − 2 [ 2 − 𝑥]
1 2 2 1

1 9 1 9 1
=2 [2 − 2 − 2 + 1] + [12 − 2 − 8 + 2] − 2 [2 − 3 − 2 + 1]
3 3
=1+2 − 1 = 2 𝑠𝑞. 𝑢𝑛𝑖𝑡

EXERCISE
Q.1 Find the area of the region bounded by the curve 𝑥 2 = 4𝑦 , 𝑦 − 𝑎𝑥𝑖𝑠 and the latus
rectum.
0
Q.2 Sketch the graph of 𝑦 = |𝑥 + 3| 𝑎𝑛𝑑 𝑒𝑣𝑎𝑙𝑢𝑎𝑡𝑒 ∫−6|𝑥 + 3| 𝑑𝑥.

Q.3 Find the area enclosed by the parabola 4𝑦 = 3𝑥 2 𝑎𝑛𝑑 𝑡ℎ𝑒 𝑙𝑖𝑛𝑒 2𝑦 = 3𝑥 + 12.
Q.4 Using integration, find the area of the region bounded by the triangle whose vertices are
(-1,0), (1,3) and (3,2).
Q.5 Using integration, find the area of the region bounded by the lines :
2𝑥 + 𝑦 = 4 , 3𝑥 − 2𝑦 = 6 𝑎𝑛𝑑 𝑥 − 3𝑦 + 5 = 0.
𝑄.6 Find the area of the region {(𝑥, 𝑦): 𝑥 2 +𝑦 2 ≤ 4, 𝑥 + 𝑦 ≥ 2}
Q.7 Find the area bounded by the curve 𝑦 = 𝑐𝑜𝑠𝑥 𝑏𝑒𝑡𝑤𝑒𝑒𝑛 𝑥 = 0 and 𝑥 = 2𝜋 .
Q8. Find the area enclosed by the curve 𝑥 2 + 𝑦 2 − 2𝑥 = 0 𝑤𝑖𝑡ℎ 𝑡ℎ𝑒 𝑥 − 𝑎𝑥𝑖𝑠.
Q9. Find the area bounded by the curve 𝑦 2 = 4𝑥 𝑎𝑛𝑑 𝑦 = 2𝑥.
Q10. Find the area of the region in the first quadrant enclosed by the 𝑥 −
𝑎𝑥𝑖𝑠 , 𝑡ℎ𝑒 𝑙𝑖𝑛𝑒 √3 𝑦 = 𝑥 𝑎𝑛𝑑 𝑡ℎ𝑒 𝑐𝑢𝑟𝑣𝑒 𝑥 2 + 𝑦 2 = 4.

ANSWERS:
1. 4/3 2. 9 3. 27 4. 4 5. 7/2
6. 𝜋 − 2 7. 4 8. 𝜋/2 9. 1/3 10. 𝜋/3

32
DIFFERENTIAL EQUATIONS

Definition: - An equation that involves an independent variable, dependent variable and


differential coefficients of dependent variable with respect to independent
variable is called differential equation.

2
𝑑𝑦 𝑑3𝑦 𝑑2𝑦 𝑑𝑦
𝐸𝑥𝑎𝑚𝑝𝑙𝑒𝑠: (1) 𝑥 +𝑦 =0 (2) + 2 ( ) − +𝑦 =0
𝑑𝑥 𝑑𝑥 3 𝑑𝑥 2 𝑑𝑥

𝑑𝑦 𝑑𝑦
(3) 𝑥 + sin ( ) = 0 (4) 𝑦 ′′ + 2𝑦 ′ + cos 𝑦 = 0
𝑑𝑥 𝑑𝑥

Order and Degree of the Differential Equation

Order of a Differential Equation - order of the highest order derivative in the differential
equation.

Degree of a Differential Equation - it is the exponent (power) of the highest order


derivative, after the equation is freed from the negative and fractional power of the
derivatives.

Note:- Degree is not defined if the given differential equation is not a polynomial
equation in its derivatives.
Students can learn as: If differential equation involves following types of terms, then we say
that degree of differential equation is not defined-

𝐬𝐢𝐧 𝒚′ , 𝐜𝐨𝐬 𝒚′′ , 𝒆𝒚′ , 𝒍𝒐𝒈 𝒚′ 𝒆𝒕𝒄.


Solved Questions
Find the order and degree of the following Differential Equation -
2
𝑑3 𝑦 𝑑2 𝑦 𝑑𝑦
1. 3
+ 2( 2
) − +𝑦=0
𝑑𝑥 𝑑𝑥 𝑑𝑥

𝑏𝑒𝑐𝑎𝑢𝑠𝑒 ℎ𝑖𝑔ℎ𝑒𝑠𝑡 𝑜𝑟𝑑𝑒𝑟 𝑑𝑒𝑟𝑖𝑣𝑎𝑡𝑖𝑣𝑒 is 𝑑3 𝑦/𝑑𝑥 3 , therefore order=3, 𝑎𝑛𝑑 𝑏𝑒𝑐𝑎𝑢𝑠𝑒 𝑝𝑜𝑤𝑒𝑟
𝑜𝑓 ℎ𝑖𝑔ℎ𝑒𝑠𝑡 𝑜𝑟𝑑𝑒𝑟 𝑑𝑒𝑟𝑖𝑣𝑎𝑡𝑖𝑣𝑒 𝑖𝑠 1, 𝑡ℎ𝑒𝑟𝑒𝑓𝑜𝑟𝑒 𝑑𝑒𝑔𝑟𝑒𝑒 = 1.
𝑑𝑦
2. 𝑥 𝑑𝑥 + 𝑦 = 0 𝑂𝑟𝑑𝑒𝑟 = 1, 𝐷𝑒𝑔𝑟𝑒𝑒 = 1
𝑑𝑦 𝑑𝑦
3. + sin (𝑑𝑥 ) = 0 𝑂𝑟𝑑𝑒𝑟 = 1, 𝐷𝑒𝑔𝑟𝑒𝑒 = 𝑛𝑜𝑡 𝑑𝑒𝑓𝑖𝑛𝑒𝑑
𝑑𝑥

4. 𝑦′′ + 2𝑦′ + 𝑠𝑖𝑛𝑦 = 0 Order=2, Degree=1


5. (𝑦′′′)2 + (𝑦′′)3 + (𝑦′)4 + 𝑦 5 = 0 𝑂𝑟𝑑𝑒𝑟 = 3, 𝐷𝑒𝑔𝑟𝑒𝑒 = 2
2/3
𝑑3 𝑦
6. (𝑑𝑥 3 ) = 𝑥
2
𝑑3 𝑦
Sol: Taking whole cube both side (𝑑𝑥 3 ) = 𝑥 3

33
Now we can see order= 3, degree = 2

Solution of Differential Equations-


A relation between the variables, such that this relation and derivative obtained from it,
satisfies the given differential equation, is called solution of differential equation.

General and Particular Solution of a Differential Equation

The solution which contains arbitrary constants (equal to order of the differential
equation) is called General Solution.

The solution free from arbitrary constants i.e. solution obtained from general
solution by giving particular values to the arbitrary constants is called Particular
Solution.

To find the solution of given Differential Equation

We have to find the solutions of three types of differential equations in our syllabus, we
use the method to solve them accordingly-

(1) Differential Equation with Variables Separable.


(2) Homogeneous Differential Equation
(3) Linear Differential Equation (First order and first degree only)

[1] Differential Equation with Variables Separable-

In this type of Differential Equation, we can separate the variables at LHS and RHS and
then integrate both sides to get the solution.

Solved Questions:
1. sec 2 𝑥 tan 𝑦 𝑑𝑥 + sec 2 𝑦 tan 𝑥 𝑑𝑦 = 0

sec2 𝑥 sec2 𝑦
Sol: sec 2 𝑥 tan 𝑦 𝑑𝑥 = − sec 2 𝑦 tan 𝑥 𝑑𝑦 ⇒ 𝑑𝑥 = − 𝑑𝑦
tan 𝑥 tan 𝑦

[Note that, in this step, variables x and y are completely separated. Remember, dx and dy
should not be in denominator. After separating variables, integrate both sides.]

sec 2 𝑥 sec 2 𝑦 𝑓 ′ (𝑥)


⇒∫ 𝑑𝑥 = − ∫ 𝑑𝑦 𝑛𝑜𝑤 𝑢𝑠𝑖𝑛𝑔 ∫ 𝑑𝑥 = log 𝑓(𝑥) + 𝐶
tan 𝑥 tan 𝑦 𝑓(𝑥)

⇒ log(tan 𝑥) = − log(tan 𝑦 ) + 𝐶 ⇒ log(tan 𝑥) + log(tan 𝑦 ) = 𝐶

⇒ log(tan 𝑥 tan 𝑦) = 𝐶

Here, note that, we can also take 𝑙𝑜𝑔𝐶 as arbitrary constant in place of C,

Then log(tan 𝑥 tan 𝑦) = log 𝐶 ⇒ tan 𝑥 tan 𝑦 = 𝐶

34
2. Find the particular solution satisfying the given condition-
𝑑𝑦
= 𝑦 tan 𝑥 , 𝑦 = 1, 𝑤ℎ𝑒𝑛 𝑥 = 0
𝑑𝑥
𝑑𝑦 𝑑𝑦
Sol: = tan 𝑥 𝑑𝑥 ⇒ ∫ = ∫ tan 𝑥 𝑑𝑥
𝑦 𝑦

log 𝑦 = − log cos 𝑥 + 𝐶 … … … … … … … . (1)

𝑈𝑠𝑖𝑛𝑔 𝑡ℎ𝑒 𝑔𝑖𝑣𝑒𝑛 𝑐𝑜𝑛𝑑𝑖𝑡𝑖𝑜𝑛 𝑦 = 1, 𝑥=0 ⇒ log 1 = − log cos 0 + 𝐶

0 = − log 1 + 𝐶 ⇒ 𝐶=0 (𝑁𝑜𝑡𝑒 𝑡ℎ𝑎𝑡 log 1 = 0)

𝑃𝑢𝑡𝑡𝑖𝑛𝑔 𝑣𝑎𝑙𝑢𝑒 𝑜𝑓 𝐶 𝑖𝑛 (1), log 𝑦 = −log(cos 𝑥) ⇒ log 𝑦 = log(cos 𝑥)−1

⇒ log 𝑦 = log sec 𝑥

⇒ 𝑦 = sec 𝑥
𝑑𝑦
3. For the Differential Equation 𝑥𝑦 𝑑𝑥 = (x + 2)(y + 2), find the solution curve

passing through the point (1, –1).


Sol: [Curve passes through (1, –1) means, after getting general solution, we put x=1, y=–1 and
find the value of arbitrary constant.]
𝑦 𝑥+2
𝑥𝑦 𝑑𝑦 = (x + 2)(y + 2)𝑑𝑥 ⇒∫ 𝑑𝑦 = ∫ 𝑑𝑥
𝑦+2 𝑥
𝑦+2−2 𝑥+2 2 2
⇒∫ 𝑑𝑦 = ∫ 𝑑𝑥 ⇒ ∫ (1 − ) 𝑑𝑦 = ∫ (1 + ) 𝑑𝑥
𝑦+2 𝑥 𝑦+2 𝑥

⇒ 𝑦 − 2 log|𝑦 + 2| = 𝑥 + 2 log|𝑥| + 𝐶 …………………..(1)

Put x=1, y= –1, ⇒ −1 − 2 log 1 = 1 + 2 log 1 + C


⇒ −1 − 0 = 1 + 0 + C ⇒ C = −2 𝑝𝑢𝑡 𝑖𝑛 (1)
𝑦 − 2 log|𝑦 + 2| = 𝑥 + 2 log|𝑥| − 2

4. The volume of spherical balloon being inflated changes at a constant rate. If initially its radius
is 3 units and after 3 seconds it is 6 units. Find the radius of balloon after 𝑡 seconds.

Sol: Given that, Rate of change in volume w.r.t. time = constant


𝑑𝑉 𝑑 4 3 4 𝑑𝑟
⇒ =𝑘 ⇒ ( 𝜋𝑟 ) = 𝑘 ⇒ 𝜋 . 3𝑟 2 . =𝑘
𝑑𝑡 𝑑𝑡 3 3 𝑑𝑡

⇒ 4𝜋𝑟 2 𝑑𝑟 = 𝑘 𝑑𝑡 ⇒ 4𝜋 ∫ 𝑟 2 𝑑𝑟 = 𝑘 ∫ 𝑑𝑡

𝑟3
⇒ 4𝜋 = 𝑘𝑡 + 𝐶 … … … … … … … … . (1)
3
Given conditions are (a) at t=0, r=3 [it gives constant C]

35
And (b) at t=3, r=6 [it gives constant k]

Using condition (a) in (1),


33
4𝜋 = 0+𝐶 ⇒ 𝐶 = 36𝜋, 𝑝𝑢𝑡 𝑡ℎ𝑖𝑠 𝑣𝑎𝑙𝑢𝑒 𝑜𝑓 𝐶 𝑖𝑛 (1)
3
𝑟3
⇒ 4𝜋 = 𝑘𝑡 + 36𝜋 … … … … … … … … . (2)
3
Using condition (b) in (2),
63
⇒ 4𝜋 = 3𝑘 + 36𝜋 𝑖𝑡 𝑔𝑖𝑣𝑒𝑠 𝑘 = 84𝜋, 𝑝𝑢𝑡 𝑡ℎ𝑖𝑠 𝑣𝑎𝑙𝑢𝑒 𝑜𝑓 𝑘 𝑖𝑛 (2)
3
𝑟3 𝑟3
⇒ 4𝜋 = 84𝜋 𝑡 + 36𝜋 ⇒ = 21𝑡 + 9 (𝑐𝑎𝑛𝑐𝑒𝑙𝑙𝑖𝑛𝑔 4𝜋)
3 3
⇒ 𝑟 3 = 63𝑡 + 27 𝑟 = (63𝑡 + 27)1/3

5- Solve the Differential Equation 3𝑒 𝑥 tan 𝑦 𝑑𝑥 + (1 − 𝑒 𝑥 ) sec 2 𝑦 𝑑𝑦 = 0

3𝑒 𝑥 sec2 𝑦
Sol.: 3𝑒 𝑥 tan 𝑦 𝑑𝑥 = (𝑒 𝑥 − 1) sec 2 𝑦 𝑑𝑦 ⇒ ∫ 𝑒 𝑥 −1 𝑑𝑥 = ∫ 𝑑𝑦
tan 𝑦

𝑥
𝑓 ′ (𝑥)
⇒ 3 log(𝑒 − 1) = log tan 𝑦 + log 𝑐 … . 𝑢𝑠𝑖𝑛𝑔 ∫ 𝑑𝑥 = log 𝑓(𝑥) + 𝐶
𝑓(𝑥)
⇒ log(𝑒 𝑥 − 1)3 = log(𝑐 tan 𝑦)
⇒ (𝑒 𝑥 − 1)3 = 𝑐 tan 𝑦

𝑑𝑦
6- Solve the Differential Equation = 𝑒 𝑥−𝑦 + 𝑥 2 𝑒 −𝑦
𝑑𝑥
𝑑𝑦
Sol: = 𝑒 𝑥−𝑦 + 𝑥 2 𝑒 −𝑦 = 𝑒 𝑥 . 𝑒 −𝑦
+ 𝑥 2 𝑒 −𝑦 = 𝑒 −𝑦 (𝑒 𝑥 + 𝑥 2 )
𝑑𝑥
𝑑𝑦 𝑥3
∫ −𝑦 = ∫(𝑒 𝑥 + 𝑥 2 ) 𝑑𝑥 ⇒ 𝑦
∫ 𝑒 𝑑𝑦 = ∫(𝑒 + 𝑥 𝑥 2)
𝑑𝑥 𝑦
⇒ 𝑒 =𝑒 + +𝑐 𝑥
𝑒 3

[2] Homogeneous Differential Equation


𝑑𝑦
A differential equation = 𝐹(𝑥, 𝑦) is said to be homogeneous if 𝐹(𝑥, 𝑦) is a homogeneous
𝑑𝑥
function of degree zero. It means if in 𝐹(𝑥, 𝑦), we replace 𝑥 𝑏𝑦 𝜆𝑥 𝑎𝑛𝑑 𝑦 𝑏𝑦 𝜆𝑦, then all λ
will be cancelled and we get the same function. For example-

𝑑𝑦 𝑥 2 + 𝑦 2
= is a differential equation, if we replace x by λx and y by λy,
𝑑𝑥 𝑥 2 + 𝑥𝑦

𝑑𝑦 𝜆2 𝑥 2 +𝜆2 𝑦 2 𝜆2 (𝑥 2 +𝑦 2 ) 𝑥 2 +𝑦 2
= 𝜆2 𝑥 2 +𝜆𝑥.𝜆𝑦 = = 𝜆0 , ∴ it is homogeneous Differential Equation.
𝑑𝑥 𝜆2 (𝑥 2 +𝑥.𝑦) 𝑥 2 +𝑥𝑦

𝑑𝑦 𝑦
This type of homogeneous Differential Equation can be expressed as = 𝑓 (𝑥 )
𝑑𝑥

𝑦2
𝑑𝑦 1+ 2 𝑦
In above example, dividing numerator & denominator by x2, = 𝑥
𝑦 = 𝑓 (𝑥 )
𝑑𝑥 1+
𝑥

36
Method to solve Homogeneous Differential Equation
𝒅𝒚 𝒚
Form of Differential Equation is = 𝒇 (𝒙 ) ………………………………..(1)
𝒅𝒙

𝑃𝑢𝑡, 𝑦 = 𝑣𝑥 𝑎𝑛𝑑 𝑑𝑖𝑓𝑓𝑒𝑟𝑒𝑛𝑡𝑖𝑎𝑡𝑒 𝑏𝑜𝑡ℎ 𝑠𝑖𝑑𝑒𝑠 𝑤. 𝑟. 𝑡. 𝑥

𝑑𝑦 𝑑𝑣 𝑑𝑦
=𝑣+𝑥 𝑇ℎ𝑒𝑛, 𝑃𝑢𝑡 𝑏𝑜𝑡ℎ 𝑣𝑎𝑙𝑢𝑒𝑠 𝑜𝑓 𝑦 𝑎𝑛𝑑 𝑖𝑛 (1), 𝑎𝑛𝑑 𝑠𝑜𝑙𝑣𝑒 𝑓𝑢𝑟𝑡ℎ𝑒𝑟.
𝑑𝑥 𝑑𝑥 𝑑𝑥

𝑁𝑜𝑤 𝑡ℎ𝑒 𝑣𝑎𝑟𝑖𝑎𝑏𝑙𝑒𝑠 𝑣 𝑎𝑛𝑑 𝑥 𝑐𝑎𝑛 𝑏𝑒 𝑠𝑒𝑝𝑎𝑟𝑎𝑡𝑒𝑑. Therefore, separate the variables and then
integrate both sides.

Solved Questions

Solve the following Differential Equations-


𝑥+𝑦
1. 𝑦 ′ = 𝑥
𝑦 𝑦
Sol: Given Differential Equation can be written as 𝑦 ′ = 1 + 𝑥 , 𝑤ℎ𝑖𝑐ℎ 𝑖𝑠 𝑓𝑢𝑛𝑐𝑡𝑖𝑜𝑛 𝑜𝑓 (𝑥 )
Therefore, it is a homogeneous Differential Equation
𝑑𝑦
𝑃𝑢𝑡, 𝑦 = 𝑣𝑥 ⇒
𝑑𝑥
𝑑𝑣
=𝑣+𝑥 𝑝𝑢𝑡 𝑖𝑛 𝑔𝑖𝑣𝑒𝑛 Differential Equation
𝑑𝑥
𝑑𝑣 𝑑𝑣 𝑑𝑥
⇒ 𝑣 + 𝑥 𝑑𝑥 = 1 + 𝑣 ⇒ 𝑥 𝑑𝑥 = 1 ⇒ ∫ 𝑑𝑣 = ∫ 𝑥

𝑦
⇒ 𝑣 = log|𝑥| + 𝑐 ⇒ = log|𝑥| + 𝑐 ⇒ 𝑦= 𝑥 log 𝑥 + 𝑐𝑥 …. Ans.
𝑥

2. Find the particular solution satisfying the given condition

𝑑𝑦 𝑦 𝑦
− + 𝑐𝑜𝑠𝑒𝑐 = 0, 𝑦 = 0, 𝑤ℎ𝑒𝑛 𝑥 = 1
𝑑𝑥 𝑥 𝑥
𝑑𝑦 𝑦 𝑦 𝑑𝑦 𝑑𝑣
Sol: = − 𝑐𝑜𝑠𝑒𝑐 𝑃𝑢𝑡, 𝑦 = 𝑣𝑥 ⇒ = 𝑣 + 𝑥 𝑑𝑥 𝑝𝑢𝑡 𝑖𝑛 𝑔𝑖𝑣𝑒𝑛 Diff. Equation
𝑑𝑥 𝑥 𝑥 𝑑𝑥

𝑑𝑣 𝑑𝑣
𝑣+𝑥 = 𝑣 − 𝑐𝑜𝑠𝑒𝑐 𝑣 ⇒ 𝑥 = −𝑐𝑜𝑠𝑒𝑐 𝑣
𝑑𝑥 𝑑𝑥
𝑑𝑣 𝑑𝑥 𝑑𝑥
⇒ = ⇒ − ∫ sin 𝑣 𝑑𝑣 = ∫
−𝑐𝑜𝑠𝑒𝑐 𝑣 𝑥 𝑥
𝑦
⇒ cos 𝑣 = log|𝑥| + 𝑐 ⇒ cos
= log|𝑥| + 𝑐 … … … … … (1)
𝑥
Using the given condition 𝑦 = 0, 𝑤ℎ𝑒𝑛 𝑥 = 1
cos 0 = log 1 + 𝑐 ⇒ 𝑐 = 1 𝑝𝑢𝑡 𝑡ℎ𝑖𝑠 𝑣𝑎𝑙𝑢𝑒 𝑜𝑓 𝑐 𝑖𝑛 (1), 𝑤𝑒 𝑔𝑒𝑡

𝑦
cos = log|𝑥| + 1
𝑥

37
𝑦 𝑦
OR 𝐴𝑠 log 𝑒 = 1, ∴ 𝑤𝑒 𝑐𝑎𝑛 𝑤𝑟𝑖𝑡𝑒 cos = log|𝑥| + log 𝑒 ⇒ cos = log|𝑒𝑥|
𝑥 𝑥

𝜋
3. Find the equation of the curve passing through (1, 4 ), if the slope of the tangent to the
𝑦 𝑦
curve at any point (x, y) is − cos2 𝑥 .
𝑥

𝑑𝑦 𝑦 𝑦 𝑑𝑦 𝑑𝑣
Sol: = − cos 2 𝑥 𝑃𝑢𝑡, 𝑦 = 𝑣𝑥 ⇒ = 𝑣 + 𝑥 𝑑𝑥 𝑝𝑢𝑡 𝑖𝑛 𝑔𝑖𝑣𝑒𝑛 Diff. Equation
𝑑𝑥 𝑥 𝑑𝑥

𝑑𝑣 𝑑𝑣 𝑑𝑥 𝑑𝑥
𝑣+𝑥 = 𝑣 − cos 2 𝑣 ⇒ = − ⇒ ∫ sec 2 𝑣 𝑑𝑣 = − ∫
𝑑𝑥 cos2 𝑣 𝑥 𝑥
𝑦
⇒ tan 𝑣 = − log|𝑥| + 𝑐 ⇒ tan = − log|𝑥| + 𝑐
𝑥
𝜋 𝜋 𝑦
𝑁𝑜𝑤 𝑢𝑠𝑖𝑛𝑔 𝑥 = 1, 𝑦 = 4 , tan 4 = − log 1 + 𝑐 ⇒ 𝑐 = 1 ∴ tan 𝑥 = − log|𝑥| + 1

[3] Linear Differential Equation


𝒅𝒚
Form of linear Differential Equation is + 𝑷𝒚 = 𝑸
𝒅𝒙
Where P and Q are the functions of x or constants only.

Method to solve Linear Differential Equation


𝑑𝑦
Convert the given Differential Equation in the form + 𝑃𝑦 = 𝑄 and write P and Q separately
𝑑𝑥
which are functions of x or constants.
Now find Integrating factor (I.F.) as 𝑰. 𝑭. = 𝒆∫ 𝑷𝒅𝒙 ,
Note:- In this integration , do not write the constant of integration.
Now write the solution of the given Differential Equation as given below and then solve

𝒚. (𝑰. 𝑭. ) = ∫ 𝑸. (𝑰. 𝑭. )𝒅𝒙 + 𝒄


Note:- Before solving the questions, note the following concepts:
(here the base of log is e.)
−1 1
(𝑎) 𝑒 log 𝑥 = 𝑥 (𝑏) 𝑒 − log 𝑥 = 𝑒 log 𝑥 = 𝑥 −1 =
𝑥
2
(c) 𝑒 2 log 𝑥 = 𝑒 log 𝑥 = 𝑥 2 (𝑑) 𝐼𝑛 𝑔𝑒𝑛𝑒𝑟𝑎𝑙 𝑒 log 𝑓(𝑥) = 𝑓(𝑥)
Solved Questions
Find the general solution of the following Differential Equation
𝑑𝑦 𝑦
1. + 𝑥 = 𝑥2
𝑑𝑥
𝒅𝒚 1
Sol : Compare with form 𝒅𝒙 + 𝑷𝒚 = 𝑸, we get P = 𝑥 𝑎𝑛𝑑 𝑄 = 𝑥 2
𝟏
𝐼. 𝐹. = 𝒆∫ 𝑷𝒅𝒙 = 𝒆∫𝒙 𝒅𝒙 = 𝒆𝒍𝒐𝒈 𝒙 = 𝑥
Therefore, solution of Differential Equation will be
𝑥4
𝑦. (𝑥) = ∫(𝑥 2 ) . 𝑥 𝑑𝑥 + 𝑐 ⇒ 𝑥𝑦 = ∫ 𝑥 3 𝑑𝑥 ⇒ 𝑥𝑦 = +𝑐
4

38
𝑑𝑦 𝜋
2. cos2 𝑥 + 𝑦 = tan 𝑥 𝑤ℎ𝑒𝑟𝑒 0 ≤ 𝑥 ≤
𝑑𝑥 2
𝑑𝑦
Sol: cos2 𝑥 + 𝑦 = tan 𝑥
𝑑𝑥
𝑑𝑦 𝑦 tan 𝑥 𝑑𝑦
Dividing both sides by cos2 𝑥 , + cos2 𝑥 = cos2 𝑥 ⇒ + (sec 2 𝑥)𝑦 = tan 𝑥 . sec 2 𝑥
𝑑𝑥 𝑑𝑥

𝑑𝑦
𝐶𝑜𝑚𝑝𝑎𝑟𝑖𝑛𝑔 𝑤𝑖𝑡ℎ + 𝑃𝑦 = 𝑄, 𝑤𝑒 𝑔𝑒𝑡 𝑃 = sec 2 𝑥 , 𝑄 = tan 𝑥 . sec 2 𝑥
𝑑𝑥
2𝑥
∴ I. F. = e∫ Pdx = 𝑒 ∫ sec 𝑑𝑥
= 𝑒 tan 𝑥
Now, solution of Differential Equation is
𝑦(𝑒 tan 𝑥 ) = ∫ tan 𝑥 . sec 2 𝑥 𝑒 tan 𝑥 𝑑𝑥 + 𝑐
𝑇𝑜 𝑖𝑛𝑡𝑒𝑔𝑟𝑎𝑡𝑒 𝑅𝐻𝑆, 𝑝𝑢𝑡 tan 𝑥 = 𝑡 ⇒ sec 2 𝑥 𝑑𝑥 = 𝑑𝑡
∴ 𝑦(𝑒 tan 𝑥 ) = ∫ 𝑡. 𝑒 𝑡 𝑑𝑡 + 𝑐 𝑛𝑜𝑤 𝑖𝑛𝑡𝑒𝑔𝑟𝑎𝑡𝑖𝑛𝑔 𝑏𝑦 𝑝𝑎𝑟𝑡𝑠,

𝑦(𝑒 tan 𝑥 ) = 𝑡. 𝑒 𝑡 − ∫ 1. 𝑒 𝑡 𝑑𝑡 + 𝑐 = 𝑡. 𝑒 𝑡 − 𝑒 𝑡 + 𝑐

⇒ 𝑦(𝑒 tan 𝑥 ) = 𝑒 𝑡 (𝑡 − 1) + 𝑐 = 𝑒 tan 𝑥 (tan 𝑥 − 1) + 𝑐


⇒ 𝑦 = tan 𝑥 − 1 + 𝑐. 𝑒 − tan 𝑥
3. Find the particular solution of the Differential Equation satisfying the given condition
𝑑𝑦 1
(1 + 𝑥 2 ) + 2𝑥𝑦 = 𝑦 = 0, 𝑤ℎ𝑒𝑛 𝑥 = 1
𝑑𝑥 1+𝑥 2
𝑑𝑦 1
Sol: (1 + 𝑥 2 ) 𝑑𝑥 + 2𝑥𝑦 = 1+𝑥 2

Dividing by (1+x2),
𝑑𝑦 2𝑥 1 2𝑥 1
+ 1+𝑥 2 = (1+𝑥 2 )2
∴ 𝑃= , 𝑄=
𝑑𝑥 1+𝑥 2 (1+𝑥 2 )2
2𝑥
∫ 𝑑𝑥 2)
𝐼. 𝐹. = 𝑒 ∫ 𝑃 𝑑𝑥 = 𝑒 1+𝑥2 = 𝑒 log(1+𝑥 = 1 + 𝑥2
2𝑥 𝑓 ′ (𝑥)
[𝑁𝑜𝑡𝑒 𝑡ℎ𝑎𝑡 𝑖𝑛 ∫ 𝑑𝑥 , 𝑤𝑒 𝑢𝑠𝑒 ∫ 𝑑𝑥 = log 𝑓(𝑥) + 𝐶]
1 + 𝑥2 𝑓(𝑥)
1
∴ 𝑆𝑜𝑙𝑢𝑡𝑖𝑜𝑛 𝑜𝑓 𝐷. 𝐸. 𝑤𝑖𝑙𝑙 𝑏𝑒 𝑦 (1 + 𝑥 2 ) = ∫ (1 + 𝑥 2 )𝑑𝑥 + 𝑐
(1 + 𝑥 2 )2
1
⇒ 𝑦 (1 + 𝑥 2 ) = ∫ 𝑑𝑥 + 𝑐 ⇒ 𝑦 (1 + 𝑥 2 ) = tan−1 𝑥 + 𝑐 … … (1)
(1 + 𝑥 2 )
𝑁𝑜𝑤 𝑢𝑠𝑒 𝑡ℎ𝑒 𝑔𝑖𝑣𝑒𝑛 𝑐𝑜𝑛𝑑𝑖𝑡𝑖𝑜𝑛 𝑦 = 0, 𝑥 = 1
𝜋 𝜋
⇒ 𝑐 = −4 𝑝𝑢𝑡 𝑣𝑎𝑙𝑢𝑒 𝑜𝑓 𝑐 𝑖𝑛 (1) 𝑤𝑒 𝑔𝑒𝑡, 𝑦 (1 + 𝑥 2 ) = tan−1 𝑥 − 4

𝑒 −2√𝑥 𝑦 𝑑𝑥
4. 𝑆𝑜𝑙𝑣𝑒 𝑡ℎ𝑒 Differential Equation ( − ) =1
√𝑥 √𝑥 𝑑𝑦

𝑒 −2√𝑥 𝑦 𝑑𝑥
Sol: ( − ) =1
√𝑥 √𝑥 𝑑𝑦

39
𝑑𝑦 𝑒 −2√𝑥 𝑦 𝑑𝑦 𝑦 𝑒 −2√𝑥
⇒ = ( − ) ⇒ + =
𝑑𝑥 √𝑥 √ 𝑥 𝑑𝑥 √ 𝑥 √𝑥

𝑑𝑦 1 𝑒 −2√𝑥
𝐶𝑜𝑚𝑝𝑎𝑟𝑒 𝑤𝑖𝑡ℎ + 𝑃𝑦 = 𝑄 , 𝑤𝑒 𝑔𝑒𝑡 𝑃 = , 𝑄=
𝑑𝑥 √ 𝑥 √𝑥
1
∫ 𝑑𝑥
𝐼. 𝐹. = 𝑒 ∫ 𝑃 𝑑𝑥 = 𝑒 √𝑥 = 𝑒 2√𝑥
𝑒 −2√𝑥 1
∴ 𝑠𝑜𝑙𝑢𝑡𝑖𝑜𝑛 𝑖𝑠 𝑦 𝑒 2√𝑥 = ∫ . 𝑒 2√𝑥 𝑑𝑥 + 𝑐 ⇒ 𝑦 𝑒 2√𝑥 = ∫ . 𝑑𝑥 + 𝑐
√𝑥 √𝑥
⇒ 𝑦 𝑒 2√𝑥 = 2√𝑥 + 𝑐
5. 𝑆𝑜𝑙𝑣𝑒 𝑡ℎ𝑒 Differential Equation 𝑑𝑦 = cos 𝑥 (2 − 𝑦 𝑐𝑜𝑠𝑒𝑐 𝑥)𝑑𝑥
𝜋
𝑔𝑖𝑣𝑒𝑛 𝑡ℎ𝑎𝑡 𝑦 = 2 𝑤ℎ𝑒𝑛 𝑥 =
2
Sol: 𝑑𝑦 = cos 𝑥 (2 − 𝑦 𝑐𝑜𝑠𝑒𝑐 𝑥)𝑑𝑥
𝑑𝑦 𝑑𝑦
= 2 cos 𝑥 − 𝑦 cos 𝑥 . 𝑐𝑜𝑠𝑒𝑐 𝑥 ⇒ = 2 cos 𝑥 − 𝑦 cot 𝑥
𝑑𝑥 𝑑𝑥
𝑑𝑦
⇒ 𝑑𝑥 + 𝑦 cot 𝑥 = 2 cos 𝑥 ⇒ 𝑃 = cot 𝑥, 𝑄 = 2 cos 𝑥

𝐼. 𝐹. = 𝑒 ∫ 𝑃 𝑑𝑥 = 𝑒 ∫ cot 𝑥 𝑑𝑥 = 𝑒 log sin 𝑥 = sin 𝑥


∴ 𝑠𝑜𝑙𝑢𝑡𝑖𝑜𝑛 𝑖𝑠 𝑦 sin 𝑥 = ∫ 2 cos 𝑥 . sin 𝑥 𝑑𝑥 + 𝑐
cos 2𝑥
⇒ 𝑦 sin 𝑥 = ∫ sin 2𝑥 𝑑𝑥 + 𝑐 ⇒ 𝑦 sin 𝑥 = − +𝑐
2
𝜋 1 3 cos 2𝑥 3
𝐺𝑖𝑣𝑒𝑛 𝑐𝑜𝑛𝑑𝑖𝑡𝑖𝑜𝑛 𝑖𝑠 𝑦 = 2 𝑤ℎ𝑒𝑛 𝑥 = ⇒𝑐 = 2−2=2 ∴ 𝑦 sin 𝑥 = − +2
2 2

6. Solve the differential equation


𝑑𝑦
(𝑥 2 + 1) + 2𝑥𝑦 = √𝑥 2 + 4
𝑑𝑥

𝑑𝑦 2𝑥 √𝑥 2 +4
Sol: divide by (𝑥 2 + 1) + 𝑥 2+1 𝑦 =
𝑑𝑥 𝑥 2 +1

𝑑𝑦 2𝑥 √𝑥 2 +4
𝐶𝑜𝑚𝑝𝑎𝑟𝑒 𝑤𝑖𝑡ℎ + 𝑃𝑦 = 𝑄, 𝑃 = 𝑥 2+1 , 𝑄=
𝑑𝑥 𝑥 2 +1
2𝑥
∫ 2 𝑑𝑥 2 +1)
𝐼. 𝐹. = 𝑒 ∫ 𝑃 𝑑𝑥 = 𝑒 𝑥 +1 = 𝑒 log(𝑥 = 𝑥2 + 1
∴ 𝑆𝑜𝑙𝑢𝑡𝑖𝑜𝑛 𝑜𝑓 𝐷𝑖𝑓𝑓. 𝐸𝑞𝑛. 𝑤𝑖𝑙𝑙 𝑏𝑒 𝑦 (𝐼. 𝐹. ) = ∫ 𝑄. (𝐼. 𝐹. )𝑑𝑥 + 𝐶

⇒ 𝑦 (𝑥 2 + 1) = ∫ √𝑥 2 + 4 𝑑𝑥 + 𝐶

1
⇒ 𝑦(𝑥 2 + 1) = 𝑥 √𝑥 2 + 4 + 2 log |𝑥 + √𝑥 2 + 4| + 𝐶
2
Miscellaneous Exercise: -
1. Find the order and degree of the differential equation
3
𝑑2 𝑦 𝑑𝑦 2 𝑑𝑦
(𝑑𝑥 2 ) + (𝑑𝑥 ) + sin (𝑑𝑥 ) = 1

40
𝑑𝑦 3
2. Find the order and degree of the differential equation 𝑥 + 𝑑𝑦 = 𝑦2
𝑑𝑥
𝑑𝑥

3. 𝑆𝑜𝑙𝑣𝑒 the differential equation 𝑥(1 + 𝑦 2 )𝑑𝑥 − 𝑦(1 + 𝑥 2 )𝑑𝑦 = 0


4. 𝑆𝑜𝑙𝑣𝑒 the differential equation 3𝑒 𝑥 tan 𝑦 𝑑𝑥 + (1 − 𝑒 𝑥 ) sec 2 𝑦 𝑑𝑦 = 0
𝑑𝑦
5. 𝑆𝑜𝑙𝑣𝑒 the differential equation = 1 + 𝑥 + 𝑦 + 𝑥𝑦
𝑑𝑥
𝑑𝑦 1
6. Write the integrating factor of the differential equation − 1+𝑥 . 𝑦 = (1 + 𝑥)𝑒 𝑥
𝑑𝑥
𝑑𝑦
7. Write the integrating factor of the differential equation 𝑥 log 𝑥 + 𝑦 = 2 log 𝑥
𝑑𝑥

8. In a bank, principal increases continuously at the rate of 5% per year. In how many years Rs.
1000 double itself?
9. 𝑆𝑜𝑙𝑣𝑒 the differential equation (𝑦 2 + 1) 𝑑𝑥 + (𝑥 2 + 1) 𝑑𝑦 = 0
𝑑𝑦
10. 𝑆𝑜𝑙𝑣𝑒 the differential equation = 𝑦 sin 2𝑥 , 𝑖𝑡 𝑖𝑠 𝑔𝑖𝑣𝑒𝑛 𝑡ℎ𝑎𝑡 𝑦(0) = 1
𝑑𝑥
𝑑𝑦 𝑦
11. 𝑆𝑜𝑙𝑣𝑒 the differential equation 𝑥 𝑑𝑥 = 𝑦 − 𝑥 tan 𝑥
12 . 𝑆𝑜𝑙𝑣𝑒 the differential equation 𝑥 𝑑𝑦 − 𝑦 𝑑𝑥 = √𝑥 2 − 𝑦 2 𝑑𝑥
13. Find the particular solution of the differential equation cos 𝑥 𝑑𝑦 = sin 𝑥 (cos 𝑥 − 2𝑦)𝑑𝑥,
𝜋
𝑔𝑖𝑣𝑒𝑛 𝑡ℎ𝑎𝑡 𝑦 = 0, 𝑤ℎ𝑒𝑛 𝑥 = 3 [ Hint. I.F. = sec 2 𝑥]
𝑑𝑦
14. 𝑆𝑜𝑙𝑣𝑒 the differential equation − (cot 𝑥). 𝑦 = 2 cos 𝑥
𝑑𝑥
𝑑𝑦
15. 𝑆𝑜𝑙𝑣𝑒 the differential equation 2𝑥 2 𝑑𝑥
− 2𝑥𝑦 + 𝑦 2 = 0
16. 𝑆𝑜𝑙𝑣𝑒 the differential equation 𝑥 𝑑𝑦 + (𝑦 − 𝑥 3 )𝑑𝑥 = 0 [ Hint. I.F. = x]
𝑑𝑦 𝑥+ 𝑦 cos 𝑥
17. Find the particular solution of the differential equation 𝑑𝑥 = − 1+sin 𝑥 , given that y=1 if
x=0. [Hint. I.F. = 1+ sin x]
𝑑𝑦
18. 𝑆𝑜𝑙𝑣𝑒 the differential equation 𝑑𝑥 + 𝑦 = cos 𝑥 − sin 𝑥
Answers

1. Order=2, degree= not defined 2. Order=1, degree= 2


3. 1 + 𝑥 2 = 𝑐(1 + 𝑦 2 ) 4. tan 𝑦 = 𝑐 (𝑒 𝑥 − 1)3
𝑥2 1
5. log|1 + 𝑦| = 𝑥 + + 𝑐 6.
2 1+𝑥

7. log 𝑥 8. 20 log 2 𝑦𝑒𝑎𝑟𝑠


cos 2𝑥 1
9. tan−1 𝑥 + tan−1 𝑦 = 𝑐 10. Log 𝑦 = − +2
2
𝑦 𝑐 𝑦
11. sin 𝑥 = 𝑥 12. Sin−1 𝑥 = log 𝑥 + 𝑐
𝑦
13. 𝑦 sec 2 𝑥 = sec 𝑥 − 2 14. = 2 log sin 𝑥 + 𝑐
sin 𝑥
2𝑥 𝑥3 𝑐
15. log 𝑥 + 𝑐 = 16. 𝑦 = +𝑥
𝑦 4

𝑥2
17. 𝑦 (1 + sin 𝑥) = − +1 18. 𝑦 = cos 𝑥 + 𝑐 𝑒 −𝑥
2

41
VECTORS
Introduction
❖ Scalar is a quantity that has only magnitude like length, mass, time, temperature, work,
etc.
❖ Vector is a quantity that has magnitude as well as direction like displacement, velocity,
force, weight, etc.
❖ A directed line segment AB is a vector denoted as 𝐴𝐵
⃗⃗⃗⃗⃗
❖ Zero vector or null vector is a vector whose magnitude is zero and direction is undefined,
⃗⃗⃗⃗⃗ = ⃗0
i.e. whose initial and terminal points are coincident.𝐴𝐴
𝑎⃗
❖ Unit vector (𝒂
̂ ): a vector whose magnitude is unity, 𝑎̂ = |𝑎⃗|
❖ Equal vectors: if two vectors 𝑎 and 𝑏⃗ have same direction and equal magnitude, i.e. 𝑎=𝑏⃗
❖ Co-initial Vector: two or more vectors having the same initial point are called co-initial
vectors.
❖ Collinear Vectors: two or more vectors are said to be collinear if they are parallel to the
same line, irrespective of their magnitudes and directions.
❖ Negative of a vector: a vector whose magnitude is the same as that of a given vector, but
direction is opposite that of it, is called negative of the given vector.
❖ Position Vector: vector 𝑂𝑃 ⃗⃗⃗⃗⃗ having origin point O(0, 0, 0) as initial point and point P(x, y,
z) as terminal point is called the position vector of point P with respect to origin.
Magnitude of ⃗⃗⃗⃗⃗
𝑂𝑃 = | ⃗⃗⃗⃗⃗⃗
𝑂𝑃| = √𝑥2 + 𝑦2 + 𝑧2

❖ Direction Cosines: as in figure, angle 𝛼, 𝛽, 𝛾 made by


the vector 𝑟 with the positive directions of x, y and z-axes
respectively, are called its direction angles and the cosine
values of these angles i.e. 𝑙 = 𝑐𝑜𝑠𝛼, 𝑚 = 𝑐𝑜𝑠𝛽 and n=𝑐𝑜𝑠𝛾
are called direction cosines of vector 𝑟

❖ Direction Ratios : the coordinate of point P(as in above figure) may also be expressed as
(lr, mr, nr). The numbers lr, mr, nr are proportional to the direction cosines are called as
direction ratios of vector 𝑟 and denoted as a, b, c.
NOTE: 𝑙 2 + 𝑚2 + 𝑛2 = 1 but 𝑎2 + 𝑏 2 + 𝑐 2 ≠ 1 in general.

❖ Addition of Vectors: in general, if we have two vectors 𝐴𝐵 ⃗⃗⃗⃗⃗ and 𝐵𝐶


⃗⃗⃗⃗⃗ then to add them,
they are positioned so that the initial point of one coincides with the terminal point of the other.
(as in figure)
As ⃗⃗⃗⃗⃗
𝐴𝐶 = ⃗⃗⃗⃗⃗ ⃗⃗⃗⃗⃗ since ⃗⃗⃗⃗⃗
𝐴𝐵 + 𝐵𝐶 𝐴𝐶 = - ⃗⃗⃗⃗⃗
𝐶𝐴

So, we have, 𝐴𝐵 ⃗⃗⃗⃗⃗ + 𝐵𝐶


⃗⃗⃗⃗⃗ + 𝐶𝐴
⃗⃗⃗⃗⃗ = 𝐴𝐴
⃗⃗⃗⃗⃗ = ⃗0, this means that when
the sides of a triangle are taken in order, it leads to zero.
Parallelogram law of vector addition- if we have two vectors 𝑎 and
𝑏⃗ represented by the two adjacent sides of a parallelogram in magnitude and direction, then
42
their sum 𝑎+ 𝑏⃗ is represented in magnitude and direction by the diagonal of the parallelogram
through their common point.

❖ Properties of vector addition:


(i)Commutative property 𝑎+ 𝑏⃗ = 𝑏⃗+ 𝑎
(ii) Associative property :( 𝑎+ 𝑏⃗) + 𝑐 = 𝑎 + ( 𝑏⃗ + 𝑐 )

❖ Components of a vector: if the position vector of point P(x, y, z) is


𝑂𝑃= x𝑖̂ + 𝑦𝑗̂ + 𝑧𝑘̂ . This form of any vector is called its component form. Here x, y and z are
⃗⃗⃗⃗⃗
called as the scalar components of 𝑟 and x𝑖̂, 𝑦𝑗̂ 𝑎𝑛𝑑 𝑧𝑘̂ are called vector components of 𝑟.

❖ Vector joining two points: if P(𝑥1 , 𝑦1 , 𝑧1 ) and Q(𝑥2 , 𝑦2 , 𝑧2 ) are any two points, then the
vector joining P and Q is the vector 𝑃𝑄 ⃗⃗⃗⃗⃗ = (𝑥2 − 𝑥1 )𝑖̂ + (𝑦2 − 𝑦1 )𝑗̂ + (𝑧2 − 𝑧1 )𝑘̂ and the
magnitude of ⃗⃗⃗⃗⃗ ⃗⃗⃗⃗⃗ | = √(𝑥2 − 𝑥1 )2 + (𝑦2 − 𝑦1 )2 + (𝑧2 − 𝑧1 )2
𝑃𝑄 = |𝑃𝑄
❖ Section Formula: Let P and Q are two points represented by the position vectors ⃗⃗⃗⃗⃗
𝑂𝑃 and
⃗⃗⃗⃗⃗⃗ ⃗⃗⃗⃗⃗ ⃗⃗⃗⃗⃗⃗ ⃗⃗⃗⃗⃗
⃗⃗⃗⃗⃗ = 𝑚𝑂𝑄+𝑛𝑂𝑃 (ii) Externally 𝑂𝑅
⃗⃗⃗⃗⃗⃗ and point R divides PQ in m : n (i) Internally 𝑂𝑅
𝑂𝑄 ⃗⃗⃗⃗⃗ = 𝑚𝑂𝑄−𝑛𝑂𝑃
𝑚+𝑛 𝑚−𝑛
LEVEL -1 (2 Marks)
Question.1 Find a vector in the direction of vector 𝑎 = 𝑖̂ − 2𝑗̂ + 3𝑘̂ that has a magnitude of 14
units.
𝑎⃗ 1
Solution: the unit vector in the direction of the given vector 𝑎 is 𝑎̂ = |𝑎⃗| = (𝑖̂ − 2𝑗̂ + 3𝑘̂)
√14

⃗ | = √(1)2 + (−2)2 + (3)2


Since |𝑎 = √14, therefore the vector having magnitude equal to 14
1
and in the direction of 𝑎 is = 14𝑎̂ = 14. (𝑖̂ − 2𝑗̂ + 3𝑘̂) = √14(𝑖̂ − 2𝑗̂ + 3𝑘̂) Ans.
√14

Question.2 Find the unit vector in the direction of the sum of the vectors 𝑎 = (3𝑖̂ + 2𝑗̂ − 3𝑘̂) and
𝑏⃗ = (𝑖̂ + 4𝑗̂ + 5𝑘̂)

Answer: the sum of the given vector is 𝑎 + 𝑏⃗ = (3𝑖̂ + 2𝑗̂ − 3𝑘̂)+(𝑖̂ + 4𝑗̂ + 5𝑘̂) =(4𝑖̂ + 6𝑗̂ + 2𝑘̂)
=𝑐 (𝑙𝑒𝑡)

2 2 2 𝑐 ̂)
(4𝑖̂+6𝑗̂ +2𝑘 ̂)
(2𝑖̂+3𝑗̂ +𝑘
|𝑐̂ | = √(4) + (6) + (2) = √56 = 2√14 , 𝑐̂ = = = Ans.
|𝑐 | 2√14 √14

Question:3 Find the position vector of a point R which divides the line joining two points P and
Q whose position vectors are 2𝑖̂ − 4𝑗̂ + 𝑘̂ and 𝑖̂ − 3𝑗̂ + 2𝑘̂ respectively, in the ratio 2 : 1 (i)
internally. (ii) Externally

𝑂𝑃 =(2𝑖̂ − 4𝑗̂ + 𝑘̂)𝑎𝑛𝑑 𝑂𝑄


Solution: Here 𝑎 = ⃗⃗⃗⃗⃗ ⃗⃗⃗⃗⃗⃗ = 𝑏⃗ = (𝑖̂ − 3𝑗̂ + 2𝑘̂)

(i) when divides internally


⃗⃗⃗⃗⃗⃗ ⃗⃗⃗⃗⃗ ̂ ̂ ̂
⃗⃗⃗⃗⃗ = 𝑚𝑂𝑄+𝑛𝑂𝑃 = 1(2𝑖̂−4𝑗̂ +𝑘)+2(𝑖̂−3𝑗̂ +2𝑘)= 4𝑖̂−10𝑗̂+5𝑘)
𝑂𝑅 𝑚+𝑛 2+1 3

(ii) when divides externally

43
⃗⃗⃗⃗⃗⃗ ⃗⃗⃗⃗⃗ ̂ ̂
⃗⃗⃗⃗⃗ = 𝑚𝑂𝑄−𝑛𝑂𝑃 = 1(2𝑖̂−4𝑗̂ +𝑘)−2(𝑖̂−3𝑗̂ +2𝑘) = 2𝑗̂ − 3𝑘̂ Ans.
𝑂𝑅 𝑚−𝑛 2−1

Scalar (or dot) product of two vectors


❖ The scalar product of two nonzero vectors 𝑎 and 𝑏⃗ , denoted by 𝑎. 𝑏⃗ is defined as
𝑎. 𝑏⃗ = | 𝑎| | 𝑏⃗ |𝑐𝑜𝑠𝜃, where 𝜃 is the angle between 𝑎 and 𝑏⃗, 0≤ 𝜃 ≤ 𝜋, if either 𝑎 = ⃗0 or 𝑏⃗ = ⃗0,
then 𝜃 is not defined, and in this case we define 𝑎. 𝑏⃗=0.

❖ 𝑎. 𝑏⃗ = 0, if and only if 𝑎 and 𝑏⃗ are perpendicular to each other, i.e. 𝑎. 𝑏⃗ = 0 ⇔ 𝑎 ⊥ 𝑏⃗


❖ If 𝜃 = 0, then 𝑎. 𝑏⃗ = | 𝑎||𝑏⃗|, in particular 𝑎. 𝑎 = |𝑎|2
❖ If 𝜃 = 𝜋, then 𝑎. 𝑏⃗ = −| 𝑎||𝑏⃗|, |, in particular 𝑎. 𝑎 = −|𝑎|2
❖ For mutually perpendicular unit vectors 𝑖̂, 𝑗̂, 𝑘̂ : 𝑖̂. 𝑖̂ = 𝑗̂. 𝑗̂ = 𝑘̂. 𝑘̂ 1 𝑎𝑛𝑑 𝑖̂. 𝑗̂ = 𝑗̂. 𝑘̂ = 𝑖̂. 𝑘̂ = 0

𝑎⃗ .𝑏
❖ The angle 𝜃 between two nonzero vectors 𝑎 and 𝑏⃗ is given by 𝑐𝑜𝑠𝜃 = (|𝑎⃗|| 𝑏⃗|) or 𝜃 =

⃗ .𝑏
𝑎
𝑐𝑜𝑠−1 ( ⃗|
)
|𝑎
⃗ || 𝑏

❖ The scalar product is commutative i.e. 𝑎 . 𝑏⃗ = 𝑏⃗. 𝑎


❖ Distributive property 𝑎.(𝑏⃗ + 𝑐) = 𝑎 . 𝑏⃗ + 𝑎 . 𝑐
❖ Let 𝑎 and 𝑏⃗ be any two vectors, and 𝜆 be any scalar. Then (𝜆 𝑎). 𝑏⃗ = 𝜆(𝑎 . 𝑏⃗) = 𝑎. (𝜆𝑏⃗).
❖ Scalar product in terms of components: Let two vectors are 𝑎 = 𝑎1 𝑖̂ + 𝑎2 𝑗̂ + 𝑎3 𝑘̂ and 𝑎 =
𝑏1 𝑖̂ + 𝑏2 𝑗̂ + 𝑏3 𝑘̂ then 𝑎 . 𝑏⃗ = (𝑎1 𝑖̂ + 𝑎2 𝑗̂ + 𝑎3 𝑘̂). (𝑏1 𝑖̂ + 𝑏2 𝑗̂ + 𝑏3 𝑘̂)
⇒𝑎 ⃗ .𝑏⃗ = 𝑎1 𝑏1 + 𝑎2 𝑏2 + 𝑎3 𝑏3

❖ Projection of a vector on a line (or vector): let a vector 𝐴𝐵 ⃗⃗⃗⃗⃗ makes an angle 𝜃 with a
given directed line l(say), in the anticlockwise direction. Then the projection of ⃗⃗⃗⃗⃗
𝐴𝐵 on line l is
⃗⃗⃗⃗⃗ |cos𝜃, and the direction of 𝑝 being the same (or opposite)
a vector 𝑝 (say) with magnitude |𝐴𝐵
to that line l, depending upon whether cos𝜃 is positive or negative.
The vector 𝑝 is called the projection vector and its magnitude |𝑝| is simply called as the
projection of the vector ⃗⃗⃗⃗⃗
𝐴𝐵 on the directed line l.
Observations: (i) if 𝑝̂ is the unit vector along a line l, then the projection of a vector ⃗𝒂 on the
line l is given by 𝒂
⃗ . 𝑝̂ .
⃗𝒃 ⃗
⃗ .𝒃
𝒂
⃗ on other vector ⃗𝒃 is given by 𝒂
(ii) Projection of a vector 𝒂 ⃗ . 𝑏̂ 𝑜𝑟 𝒂
⃗. ⃗|
𝑜𝑟 ⃗|
|𝒃 |𝒃

⃗⃗⃗⃗⃗⃗ will be itself and if 𝜃 = 𝜋 then the projection


(iii) if 𝜃 = 0, then the projection vector of 𝑨𝑩
vector of ⃗⃗⃗⃗⃗⃗
𝑨𝑩 will be ⃗⃗⃗⃗⃗⃗
𝑩𝑨.
𝜋 3𝜋
(iv) if 𝜃 = 2 𝑜𝑟 2
, then the projection vector of ⃗⃗⃗⃗⃗⃗
𝑨𝑩 will be zero vector.

NOTE: if a vector 𝒂⃗ = 𝑎1 𝑖̂ + 𝑎2 𝑗̂ + 𝑎3 𝑘̂ then, 𝑎1 , 𝑎2 , 𝑎3 , are precisely the projections of 𝒂


⃗ along
x-axis, y-axis and z-axis, respectively.

Level - I (2 Marks)
Question :1. Find the angle between the vectors 𝑎 = 𝑖̂ + 𝑗̂ − 𝑘̂ and 𝑏⃗ = 𝑖̂ − 𝑗̂ + 𝑘̂.

44

⃗ .𝑏
𝑎
Solution: the angle 𝜃 between two vectors given by 𝜃 = 𝑐𝑜𝑠−1 ( ⃗|
)
|𝑎
⃗ || 𝑏

Since 𝑎 . 𝑏⃗ = (𝑖̂ + 𝑗̂ − 𝑘̂). (𝑖̂ − 𝑗̂ + 𝑘̂ ) = 1 − 1 − 1 = −1


−1 −1
nd |𝑎| = √1 + 1 + 1 = √3 𝑎𝑛𝑑 | 𝑏⃗| = √1 + 1 + 1 = √3 so, 𝜃 = 𝑐𝑜𝑠−1 (√3√3) = 𝑐𝑜𝑠−1 ( 3 ) Ans.

Question:2 Find the projection of the vector 𝑎 = 2𝑖̂ + 𝑗̂ − 3𝑘̂ on the vector 𝑏⃗ = 𝑖̂ − 2𝑗̂ + 𝑘̂.

Solution: the projection of the vector 𝑎 on the vector 𝑏⃗ is given by


𝒂 ⃗
⃗ .𝒃 ̂).(𝒊̂−𝟐𝒋̂+𝒌
(𝟐𝒊̂+𝒋̂−𝟑𝒌 ̂) 2−2−3 −3 3
⃗|
= = = 2√21 , therefore, projection is 2√21 Ans.
|𝒃 √4+1+9√1+4+1 √14√6

Question:3 if 𝑎 is a unit vector and (𝑥 − 𝑎). (𝑥 + 𝑎) = 8, then find |𝑥|.


Solution: Since 𝑎 is a unit vector, therefore |𝑎| = 1,
also , (𝑥 − 𝑎). (𝑥 + 𝑎) = 8,
Or 𝑥. 𝑥 + 𝑥. 𝑎 − 𝑎. 𝑥 − 𝑎. 𝑎 = 8, or |𝑥|2 − 1 = 8 i.e. |𝑥|2 = 9 , so, |𝑥| = 3 Ans.

Question:4 If 𝑎 = 2𝑖̂ + 2𝑗̂ + 2𝑘̂ , 𝑏⃗ = −𝑖̂ + 2𝑗̂ + 𝑘̂ and 𝑐 = 3𝑖̂ + 𝑗̂ are such that 𝑎 + 𝜆𝑏⃗ is
perpendicular to 𝑐, then find the value of 𝜆.

Solution: Since 𝑎 + 𝜆𝑏⃗ ⊥ 𝑐 therefore, (𝑎 + 𝜆𝑏⃗).𝑐 = 0


̂) + 𝜆(−𝑖̂ + 2𝑗̂ + 𝑘
⇒ {(2𝑖̂ + 2𝑗̂ + 2𝑘 ̂)}. (3𝑖̂ + 𝑗̂ ) = 0

⇒ (2 − 𝜆).3 + (2 + 2𝜆).1 + (2 + 𝜆).0 = 0 ⇒ 𝜆 = 8 Ans.

Question:5 If the vertices A, B, C of a triangle ABC are (1, 2, 3), (-1, 0, 0) and (0, 1, 2)
respectively, then find angle ABC

Solution: Here angle ABC is the angle between the vectors ⃗⃗⃗⃗⃗ ⃗⃗⃗⃗⃗
𝐵𝐴 and𝐵𝐶

Now ⃗⃗⃗⃗⃗
𝐵𝐴= ⃗⃗⃗⃗⃗ ⃗⃗⃗⃗⃗ = (1 + 1)𝑖̂ + (2 − 0)𝑗̂ + (3 − 0)𝑘̂ = 2𝑖̂ + 2𝑗̂ + 3𝑘̂
𝑂𝐴 − 𝑂𝐵
⃗⃗⃗⃗⃗ = 𝑂𝐶
And 𝐵𝐶 ⃗⃗⃗⃗⃗ = (0 + 1)𝑖̂ + (1 − 0)𝑗̂ + (2 − 0)𝑘̂ = 𝑖̂ + 𝑗̂ + 2𝑘̂
⃗⃗⃗⃗⃗ − 𝑂𝐵
⃗⃗⃗⃗⃗
𝐵𝐴.𝐵𝐶 ⃗⃗⃗⃗⃗ ̂ ).(𝑖̂+𝑗̂ +2𝑘
(2𝑖̂+2𝑗̂ +3𝑘 ̂) 2+2+6
Now angle ABC = 𝑐𝑜𝑠 −1 (|𝐵𝐴
⃗⃗⃗⃗⃗ ||𝐵𝐶
⃗⃗⃗⃗⃗ |
) = 𝑐𝑜𝑠 −1 (√22 ) = 𝑐𝑜𝑠 −1 ( )=
+22 +32 √12 +12 +22 √17√6
10
𝑐𝑜𝑠 −1 ( )
√102

Level - II (3 Marks)

Question:1 Let 𝑎 and 𝑏⃗ two unit vectors and 𝜃 is the angle between them. Then 𝑎 + 𝑏⃗ is a unit
vector. Find an angle.
2
Solution: Here |𝑎|= |𝑏⃗|=1 and |𝑎 + 𝑏⃗| = 1 ⇒ |𝑎 ⃗ + ⃗𝑏| = 1
⃗ + ⃗𝑏). (𝑎
⇒ (𝑎 ⃗ + ⃗𝑏) = 1 ⇒ 𝑎 ⃗ +𝑏⃗. 𝑎 + 𝑏⃗. 𝑏⃗ = 1
⃗ .𝑎+𝑎.𝑏
2 ⃗ ) + |𝑏⃗|2 = 1
⇒ |𝑎 ⃗ .𝑏
⃗ | + 2(𝑎

⃗ |𝑐𝑜𝑠𝜃 + 1 = 1 since |𝑎| = |𝑏⃗| = 1


⃗ ||𝑏
⇒ 1 + 2|𝑎

45
⇒ 2𝑐𝑜𝑠𝜃 = −1 ⇒ 𝑐𝑜𝑠𝜃 = −1/2 ⇒ 𝜃 = 2𝜋/3 Ans.
𝜃 1
Question:2 If 𝑎 and 𝑏⃗ are two unit vectors inclined at an angle 𝜃, then prove that sin2= 2 |𝑎 −
𝑏⃗|
Solution: Here |𝑎|= |𝑏⃗|=1
|𝑎 − 𝑏⃗|2 = (𝑎 − 𝑏⃗). (𝑎 − 𝑏⃗) = 𝑎.𝑎-𝑎.𝑏⃗ -𝑏⃗. 𝑎 + 𝑏⃗. 𝑏⃗

= |𝑎|2 − 2(𝑎.𝑏⃗) + |𝑏⃗|2 = 1+1 - 2 |𝑎||𝑏⃗|𝑐𝑜𝑠𝜃


= 2 - 2𝑐𝑜𝑠𝜃 = 2(1 - 𝑐𝑜𝑠𝜃)
𝜃 𝜃
= 2x2𝑠𝑖𝑛2 2 =4𝑠𝑖𝑛2 2
𝜃 𝜃 1
So, |𝑎 − 𝑏⃗|2 = 4𝑠𝑖𝑛2 2 ⇒sin2= 2 |𝑎 − 𝑏⃗ | Hence Proved.

Question:3 Let 𝑎, 𝑏⃗ and 𝑐 are three vectors such that | 𝑎 |=1, |𝑏⃗| =2 and |𝑐 | = 3. If the projection
of 𝑏⃗ along 𝑎 is equal to the projection of 𝑐 along 𝑎; and 𝑏⃗ is perpendicular to 𝑐 then find |3 𝑎-
2𝑏⃗+2𝑐 |. [CBSE 2019]
⃗ .𝑎⃗
𝑏 𝑐 .𝑎⃗
Solution: Given |𝑎⃗| = |𝑎⃗| ⇒ ⃗𝑏 . 𝑎 ⃗ ……(i)
⃗ = 𝑐⃗ . 𝑎

𝑏⃗ ⊥ 𝑐 ⇒ ⃗𝑏. ⃗𝑐 = 0…….(ii) |3 𝑎 − 2𝑏⃗ + 2𝑐|2 = (3 𝑎 − 2𝑏⃗ + 2𝑐). (3 𝑎 − 2𝑏⃗ + 2𝑐 )

= 9|𝑎|2 + 2|𝑏⃗|2 + 4|𝑐|2 − 12𝑎 . 𝑏⃗ − 8𝑏⃗ . 𝑐 + 12𝑎 . 𝑐


= 9(1)2 + 4(2)2 + 4(3)2 [ using (i) and (ii) ] = 61

Therefore |3 𝑎 − 2𝑏⃗ + 2𝑐 |2 = 61, so |3 𝑎 − 2𝑏⃗ + 2𝑐 | = √61 Ans.

Question:4 If 𝑎, 𝑏⃗ and 𝑐 are unit vectors such that 𝑎+ 𝑏⃗ + 𝑐 = ⃗0, then write the value of 𝑎. 𝑏⃗ +
𝑏⃗. 𝑐 + 𝑐. 𝑎. [CBSE 2016]

Solution: Given 𝑎+ 𝑏⃗ + 𝑐 = ⃗0

|𝑎 + 𝑏⃗ + 𝑐|2 = 0 ⇒ (𝑎+ 𝑏⃗ + 𝑐). (𝑎 + 𝑏⃗ + 𝑐 ) = 0 ⇒ |𝑎|2 + |𝑏⃗|2 + |𝑐 |2 + 2𝑎 . 𝑏⃗ + 2𝑏⃗ . 𝑐 +


2𝑎 . 𝑐 = 0

1 + 1 + 1 +2 (𝑎. 𝑏⃗ + 𝑏⃗ . 𝑐 + 𝑐. 𝑎 ) = 0 ⇒ 𝑎 . 𝑏⃗ + 𝑏⃗. 𝑐 + 𝑐 . 𝑎 = -3/2


Question:5 If with reference to the right handed system of mutually perpendicular unit
vectors 𝑖̂, 𝑗̂ and 𝑘̂, 𝛼 =3𝑖̂-𝑗̂ and 𝛽 = 2𝑖̂ + 𝑗̂ − 3𝑘̂, then express 𝛽 in the form 𝛽 = 𝛽
⃗⃗⃗⃗1 + ⃗⃗⃗⃗
𝛽2 where
⃗⃗⃗⃗1 is parallel to 𝛼 and ⃗⃗⃗⃗
𝛽 𝛽2 is perpendicular to 𝛼 .
⃗⃗⃗⃗1 = 𝜆𝛼
Solution: let𝛽 ⃗⃗⃗⃗1 = 𝜆( 3𝑖̂-𝑗̂)
⃗ where 𝜆 is a scalar, so 𝛽

⃗⃗⃗⃗ ⃗⃗⃗⃗1 = (2𝑖̂ + 𝑗̂ − 3𝑘̂)-𝜆( 3𝑖̂-𝑗̂) = (2-3𝜆)𝑖̂ + (1 + 𝜆)𝑗̂ -3𝑘̂


𝛽2 = 𝛽 − 𝛽

Since ⃗⃗⃗⃗
𝛽2 is to be perpendicular to 𝛼 So we should have 𝛼 . ⃗⃗⃗⃗
𝛽2 = 0
1
( 3𝑖̂-𝑗̂).( (2-3𝜆)𝑖̂ + (1 + 𝜆)𝑗̂ -3𝑘̂)=0, therefore, 6-9 𝜆 − 1 − 𝜆 = 0, so, 𝜆 = 2,

46
⃗⃗⃗⃗1 = 1/2( 3𝑖̂-𝑗̂) and ⃗⃗⃗⃗ 1 1 1 3
Therefore 𝛽 𝛽2= (2-3. 2)𝑖̂ + (1 + 2)𝑗̂ -3𝑘̂ = 2 𝑖̂ + 2 𝑗̂ − 3𝑘
̂ Ans.

Question:6 Let 𝑎 = 1𝑖̂ + 4𝑗̂ + 2𝑘̂ 𝑏⃗ = 3𝑖̂ − 2𝑗̂ + 7𝑘̂ and 𝑐 = 2𝑖̂ − 𝑗̂ + 4𝑘̂. Find a vector 𝑑 which
is perpendicular to both 𝑎 and 𝑏⃗ and 𝑐 . 𝑑 = 15.

Solution: let vector 𝑑 = 𝑥𝑖̂ + 𝑦𝑗̂ + 𝑧𝑘̂

Since, 𝑑 is perpendicular to 𝑎 and 𝑏⃗. So, 𝑑 . 𝑎 = 0 and , 𝑑 . 𝑏⃗ = 0

Therefore, 𝑑 . 𝑎 = 0, 𝑔𝑖𝑣𝑒𝑠, 𝑥 + 4𝑦 + 2𝑧 = 0 … . (𝑖)

𝑑 . 𝑏⃗ = 0 𝑔𝑖𝑣𝑒𝑠 3𝑥 − 2𝑦 + 7𝑧 = 0 … . (𝑖𝑖)

And 𝑑. 𝑐 = 15 𝑔𝑖𝑣𝑒𝑠, 2𝑥 − 𝑦 + 4𝑧 = 15 … (𝑖𝑖𝑖)


From (i) 𝑥 = 4𝑦 + 2𝑧, put in (ii) and (iii), we get 12𝑦 + 13𝑧 = 0 … (𝑖𝑣)
and 7𝑦 + 8𝑧 = 15. . (𝑣), so, from (iv) and (v) we get x = 160/3, y = -5/3, z= 70/3
1
So, 𝑑 = 160/3𝑖̂ − 5/3𝑗̂ + 70/3𝑘̂ = 3 ( 160𝑖̂ − 5𝑗̂ + 70𝑘̂) Ans.

VECTOR PRODUCT OR CROSS PRODUCT


❖ The vector product of two nonzero vectors 𝑎 𝑎𝑛𝑑 𝑏⃗ , is denoted by 𝑎 × 𝑏⃗ and defined as
𝑎 × 𝑏⃗ = | 𝑎 ||𝑏⃗|𝑠𝑖𝑛𝜃𝑛̂
Where 𝜃 is the angle between 𝑎 𝑎𝑛𝑑 𝑏⃗ and 𝑛̂ is a unit vector perpendicular to both 𝑎 𝑎𝑛𝑑 𝑏⃗

❖ Let 𝑎 𝑎𝑛𝑑 𝑏⃗ be two non zero vectors. Then 𝑎 × 𝑏⃗ = ⃗0 if and only if 𝑎 𝑎𝑛𝑑 𝑏⃗ are parallel (or
collinear) to each other, i.e.
𝑎 × 𝑏⃗ = ⃗0 ⇔ 𝑎 ||𝑏⃗ in particular 𝑎 × 𝑎 = ⃗0 𝑠𝑖𝑛𝑐𝑒 𝜃 = 0, 𝑠𝑜 𝑠𝑖𝑛𝜃 = 0

❖ If 𝜃 = 𝜋/2, then 𝑎 × 𝑏⃗ = | 𝑎 ||𝑏⃗|, since 𝑠𝑖𝑛𝜃 =1


❖ For mutually perpendicular unit vectors 𝑖̂, 𝑗̂𝑎𝑛𝑑 𝑘̂ we have
𝑖̂ × 𝑖̂ = 𝑗̂ × 𝑗̂ = 𝑘̂ × 𝑘̂ = ⃗0

𝑖̂ × 𝑗̂ = 𝑘̂, 𝑗̂ × 𝑘̂ = 𝑖̂, 𝑘̂ × 𝑖̂ = 𝑗̂
|𝑎⃗ ×𝑏⃗|
❖ In vector product, the angle between two vectors 𝑎 𝑎𝑛𝑑 𝑏⃗ may be given as 𝑠𝑖𝑛𝜃 = ⃗ |
|𝑎⃗ ||𝑏

❖ Vector product is not commutative i.e. 𝑎 × 𝑏⃗ = −𝑏⃗ × 𝑎, So, 𝑗̂ × 𝑖̂ = −𝑘̂, 𝑘̂ × 𝑗̂ = −𝑖̂,


𝑖̂ × 𝑘̂ = − 𝑗̂
1
❖ If 𝑎 𝑎𝑛𝑑 𝑏⃗ represent the adjacent sides of a triangle then its area is given as |𝑎 × 𝑏⃗|. 2
❖ If 𝑎 𝑎𝑛𝑑 𝑏⃗ represent the adjacent sides of a parallelogram then its area is given as |𝑎 × 𝑏⃗|.
1
And if 𝑎 𝑎𝑛𝑑 𝑏⃗ represent the diagonals of a parallelogram then area is |𝑎 × 𝑏⃗|. 2
❖ Distributive property: 𝑎 × (𝑏⃗ + 𝑐 ) = 𝑎 × 𝑏⃗ + 𝑎 × 𝑐
❖ Let 𝑎 𝑎𝑛𝑑 𝑏⃗ are two vectors given in component form as 𝑎 = 𝑎1 𝑖̂ + 𝑎2 𝑗̂ + 𝑎3 𝑘̂ and 𝑏⃗ =
𝑏1 𝑖̂ + 𝑏2 𝑗̂ + 𝑏3 𝑘̂, then their cross product may be given as

47
𝑖̂ 𝑗̂ 𝑘̂
𝑎 × 𝑏⃗ = |𝑎1 𝑎2 𝑎3 | = 𝑖̂(𝑎2 𝑏3 − 𝑎3 𝑏2 ) − 𝑗̂(𝑎1 𝑏3 − 𝑎3 𝑏1 ) + 𝑘̂(𝑎1 𝑏2 − 𝑎2 𝑏1 )
𝑏1 𝑏2 𝑏3

LEVEL-I (2 Marks)
Question:1 Find the area of a triangle having the points A(1, 1, 1), B(1, 2, 3) and C(2, 3, 1) as
its vertices.
1
𝐴𝐵 = 𝑗̂ + 2𝑘̂ and ⃗⃗⃗⃗⃗
Answer: we have ⃗⃗⃗⃗⃗ ⃗⃗⃗⃗⃗ 𝑋𝐴𝐶
𝐴𝐶 = 𝑖̂ + 2𝑗̂. The area of the given triangle is 2 |𝐴𝐵 ⃗⃗⃗⃗⃗ |

𝑖̂ 𝑗̂ 𝑘̂
Now ⃗⃗⃗⃗⃗
𝐴𝐶 × ⃗⃗⃗⃗⃗
𝐴𝐶 = |0 1 2| = 𝑖̂(0 − 4) − 𝑗̂(0 − 2) + 𝑘̂ (0 − 1) =-4 𝑖̂ + 2𝑗̂ − 𝑘̂
1 2 0
1 1
⃗⃗⃗⃗⃗ × ⃗⃗⃗⃗⃗
Area of triangle = ½ |𝐴𝐶 𝐴𝐶 | =2 √(−4)2 + (2)2 + (−1)2 = 2 √21 Ans.

Question:2 Find the area of the parallelogram whose adjacent sides are determined by the
vectors 𝑎 = 𝑖̂ − 𝑗̂ + 3𝑘̂ and 𝑏⃗ = 2𝑖̂ − 7𝑗̂ + 𝑘̂.

𝑖̂ 𝑗̂ 𝑘̂
Solution: Here 𝑎 × 𝑏⃗ =|1 −1 3|= = 𝑖̂(−1 + 21) − 𝑗̂(1 − 6) + 𝑘̂(−7 + 2) =20𝑖̂ + 5𝑗̂ − 5𝑘̂
2 −7 1
Area of parallelogram = |𝑎 × 𝑏⃗|=√(20)2 + (5)2 + (−5)2 =√450 =15√2 Ans.

Question:3 Find the area of the parallelogram whose diagonals are determined by the vectors
𝑎 = 2𝑖̂ − 𝑗̂ + 3𝑘̂ and 𝑏⃗ = 𝑖̂ − 2𝑗̂ + 3𝑘̂ .

𝑖̂ 𝑗̂ 𝑘̂
Solution: Here 𝑎 × 𝑏⃗ =|2 −1 3|= = 𝑖̂(−3 + 6) − 𝑗̂(6 − 3) + 𝑘̂(−4 + 1) =3𝑖̂ − 3𝑗̂ − 3𝑘̂
1 −2 3
Area of parallelogram = 1/2 |𝑎 × 𝑏⃗|=√(3)2 + (−3)2 + (−3)2 =√27 =3√3 Ans.

Question:4 Find α and β if (2𝑖̂ + 6𝑗̂ + 27𝑘̂) × (𝑖̂ + 𝛼𝑗̂ + 𝛽𝑘̂) = ⃗0

Solution: (2𝑖̂ + 6𝑗̂ + 27𝑘̂) × (𝑖̂ + 𝛼𝑗̂ + 𝛽𝑘̂) = ⃗0

𝑖̂ 𝑗̂ 𝑘̂
̂ ̂
(2𝑖̂ + 6𝑗̂ + 27𝑘) × (𝑖̂ + 𝛼𝑗̂ + 𝛽𝑘 ) = |2 6 27| = ⃗0
1 𝛼 β

Therefore, 𝑖̂(6β − 27α) − 𝑗̂(2𝛽 − 27) + 𝑘̂(2α − 6)= 0𝑖̂ + 0𝑗̂ + 0𝑘̂
Since, in two equal vectors their components are also equal.
27
Therefore, 6β − 27α = 0,2𝛽 − 27 = 0 𝑎𝑛𝑑 2α − 6 = 0 So, we get 𝛽 = 𝑎𝑛𝑑 α = 3 Ans.
2

√2
Question: 5 Let the vectors 𝑎 and 𝑏⃗ are such that | 𝑎| = 3 and | 𝑏⃗| = 3 , if 𝑎 × 𝑏⃗ is a unit vector,
then find angle between 𝑎 and 𝑏⃗.

48
Solution: Since 𝑎 × 𝑏⃗ is a unit vector, so |𝑎
⃗⃗⃗ × 𝑏⃗| = 1

√2
|| 𝑎 ||𝑏⃗||𝑠𝑖𝑛𝜃|=1 Therefore, 3. 𝑠𝑖𝑛𝜃 = 1 So, 𝑠𝑖𝑛𝜃 = 1/√2 So, angle 𝜃 = 𝜋/4 Ans.
3

MISCELLANEOUS QUESTIONS
LEVEL-I (2 Marks)
Q.1 For given vectors 𝑎 = 2𝑖̂ − 𝑗̂ + 2𝑘̂ and 𝑏⃗ = −𝑖̂ + 𝑗̂ − 𝑘̂ find the unit vector in the direction
of the vector 𝑎 + 𝑏⃗.

Q.2 If 𝑎 = 5𝑖̂ − 𝑗̂ − 3𝑘̂ and 𝑏⃗ = 𝑖̂ + 3𝑗̂ − 5𝑘̂ , then show that the vectors 𝑎 + 𝑏⃗ and 𝑎 − 𝑏⃗ are
perpendicular.

Q.3 Find| 𝑎 − 𝑏⃗|, if two vectors 𝑎 𝑎𝑛𝑑 𝑏⃗ are such that |𝑎| = 2, |𝑏⃗|=3 and 𝑎. 𝑏⃗ = 4.

Q.4 Find |𝑎| and |𝑏⃗|, if (𝑎 + 𝑏⃗).(𝑎 − 𝑏⃗) = 8 and |𝑎| = 8 |𝑏⃗|.

Q.5Find the magnitude of two vectors 𝑎 and 𝑏⃗ , having the same magnitude and such that the
angle between them is 600 and their scalar product is ½.

Q.6 Show that |𝑎|𝑏⃗ + |𝑏⃗|𝑎 is perpendicular to |𝑎|𝑏⃗ − |𝑏⃗|𝑎 for any two nonzero vectors 𝑎 and
𝑏⃗ .

Q.7 Find a unit vector perpendicular to each of the vector (𝑎 + 𝑏⃗) and (𝑎 − 𝑏⃗), where 𝑎 = 𝑖̂ +
̂ 𝑏⃗=𝑖̂ + 2𝑗̂ + 3𝑘̂.
𝑗̂ + 𝑘,
Q.8 Show that the direction cosines of a vector equally inclined to the axes OX, OY and OZ are
1 1 1
, , .
√3 √3 √3

Q.9 If 𝑎 = 𝑖̂ + 𝑗̂ − 2𝑘̂, 𝑏⃗=3𝑖̂ − 2𝑗̂ + 3𝑘̂ and 𝑐 = 2𝑖̂ + 𝑗̂ + 2𝑘̂ , find a unit vector parallel to the
vector 2𝑎 + 3 𝑏⃗- 4𝑐 .
Q.10 Find a vector of magnitude 5 units and parallel to the resultant of the vectors 𝑎 = 2𝑖̂ +
3𝑗̂ − 𝑘̂ , 𝑏⃗=𝑖̂ − 2𝑗̂ + 𝑘̂

LEVEL -II (3 Marks)


Question:1 Three vectors 𝑎, 𝑏⃗ and 𝑐 satisfy the condition 𝑎 + 𝑏⃗ + 𝑐 = ⃗0. Evaluate the quantity
𝜇 = ⃗𝑎. ⃗𝑏 + ⃗𝑏. 𝑐
⃗ +𝑐 ⃗ |= 3 and |𝑐 |= 4.
⃗ . If |𝑎|=2, |𝑏
⃗.𝑎

Question:2 If 𝑎 , 𝑏⃗ and 𝑐 are mutually perpendicular vectors of equal magnitudes, show that
the vectors 𝑎 + 𝑏⃗ + 𝑐 is equally inclined to 𝑎, 𝑏⃗ and 𝑐 .

Question:3 if 𝑎𝑋𝑏⃗ = 𝑐 𝑋𝑑 and 𝑎𝑋𝑐 = 𝑏⃗𝑋𝑑 then show that 𝑎 − 𝑑 is parallel to 𝑏⃗ − 𝑐, where 𝑎 ≠
𝑑 and 𝑏⃗ ≠ 𝑐 [CBSE 2016]

Hint: 𝑎 𝑋𝑏⃗ − 𝑎𝑋𝑐 = 𝑐 𝑋𝑑 − 𝑏⃗𝑋𝑑


⃗⃗⃗ − 𝑐 ) = (𝑐 − 𝑏⃗)𝑋𝑑 or 𝑎𝑋(𝑏
𝑎𝑋(𝑏 ⃗⃗⃗ − 𝑐 ) − (𝑐 − 𝑏⃗)𝑋𝑑 = ⃗0 or 𝑎𝑋(𝑏
⃗⃗⃗ − 𝑐 ) + 𝑑𝑋(𝑐 − 𝑏⃗) = ⃗0

49
⃗⃗⃗
(𝑏 − 𝑐 )𝑋(𝑎 − 𝑑) = ⃗0 , therefore, 𝑎 − 𝑑 is parallel to 𝑏⃗ − 𝑐

Question:4 The two adjacent sides of a parallelogram are 2𝑖̂ − 4𝑗̂ − 5𝑘̂ and 2𝑖̂ + 2𝑗̂ + 3𝑘̂. Find
the two unit vectors parallel to its diagonals. Using the diagonal vectors, find the area of the
parallelogram.

Hint: First find diagonals by𝑑 ⃗⃗⃗⃗1 = 𝑎 + 𝑏⃗ and ⃗⃗⃗⃗


𝑑2 = 𝑎 − 𝑏⃗ then find their unit vectors and hence
1
⃗⃗⃗⃗1 𝑋𝑑
find area by 2 |𝑑 ⃗⃗⃗⃗2 |.

Question:5 Show that the points A(1, -2, -8), B(5, 0, -2) and C(11, 3, 7) are collinear and
find the ratio in which B divides AC.

Question:6 If 𝑎 = 𝑖̂ + 2𝑗̂ + 𝑘̂, 𝑏⃗ = 2𝑖̂ + 𝑗̂ and 𝑐 = 3𝑖̂ − 4𝑗̂ − 5𝑘̂, then find a unit vector
perpendicular to both of the vectors 𝑎 − 𝑏⃗ and 𝑐 − 𝑏⃗. (Hint: first find 𝑎 − 𝑏⃗ = 𝑝(𝑙𝑒𝑡) and 𝑐 −
𝑝×𝑞⃗
𝑏⃗ = 𝑞 , hence find 𝑛̂ = , where 𝑛̂ is a unit vector perpendicular to both of the vectors 𝑎 −
|𝑝×𝑞⃗|
𝑏⃗ and 𝑐 − 𝑏⃗. [CBSE 2015]

Question:7 If 𝑎 and 𝑏⃗ are two unit vectors inclined at an angle 𝜃, then prove that cos(2)=
𝜃

1
|𝑎 + ⃗ | and tan(𝜃)= |𝑎⃗−𝑏⃗|
𝑏
2 2 |𝑎⃗+𝑏⃗|

Question:8 A vector 𝑟 has magnitude 14 and direction ratios 2, 3, -6. Find the direction
cosines and components of 𝑟, given that 𝑟 makes an acute angle with x-axis. {Hint: Find 𝑙 =
𝑎 𝑏 𝑐 ̂)
14(2𝑖̂+3𝑗̂ −6𝑘
,𝑚=
√𝑎2 +𝑏 2 +𝑐 2
,𝑛 = 𝑡ℎ𝑒𝑛 𝑓𝑖𝑛𝑑 𝑟 =
√𝑎2 +𝑏 2 +𝑐 2 √𝑎2 +𝑏 2 +𝑐 2
= 4𝑖̂ + 6𝑗̂ − 8𝑘̂
√22 +32 +62

ANSWERS
LEVEL-I
𝟏 𝟏 ̂
1. 𝒊̂ + ̂ 3. √𝟓 4. 𝟏𝟔√𝟐, 𝟐√𝟐 5. |𝑎| = 𝟏, |𝑏⃗ | = 1 7. −𝟏 𝒊̂ +
𝒌
𝟐
𝒋̂ −
𝟏
̂
𝒌 9.
2𝑖̂+𝑗̂ +2𝑘
10.
√𝟐 √𝟐 𝟑√𝟕 𝟑√𝟕 √𝟔 √𝟔 √𝟔 √19
𝟑√𝟏𝟎 √𝟏𝟎
𝒊̂ + 𝒋̂
𝟐 𝟐

LEVEL-II
̂
1. -29/2 4. 𝑑̂1 = 4𝑖̂−2𝑗̂ −2𝑘̂, 𝑑
̂1 = 3𝑗̂ +4𝑘̂ , area= 2√101 5. 2 : 3 6.
−𝒋̂+𝒌
8.
√24 5 𝟐
𝑙 =2/7, m=3/7, n=-6/7, Components 4𝑖̂, 6𝑗̂, 8𝑘̂

50
THREE DIMENSIONAL GEOMETRY
BASIC CONCEPTS:-
1. DISTANCE FORMULA:- Distance between two given points P(x1, y1,z1) and
Q(x2,y2,z2) is
PQ= √(x2 − x1 )2 + (𝑦2 − 𝑦1 )2 + (𝑧2 − 𝑧1 )2

Q:- Find the distance between the points P ( -2,4,1) and Q ( 1,2,-5)

Soln:- PQ = √(1 + 2)2 + (2 − 4)2 + (−5 − 1)2 = 7 unit

2. SECTION FORMULAE:-
IF P(x1, y1,z1) and Q(x2,y2,z2) be two points and R (x, y, z) be a point on the line
segment joining P and Q .
i) The point R divide PQ in m1:m2 Internally Then co-ordinates of point R are
𝑚1 𝑥2 +𝑚2 𝑥1 𝑚1 𝑦2 +𝑚2 𝑥1 𝑚1 𝑧2 +𝑚2 𝑧1
( , , ) .
𝑚1 +𝑚2 𝑚1 +𝑚2 𝑚1 +𝑚2
ii) The point R divide PQ in m:n Externally Then co-ordinates of point R are
𝑚1 𝑥2 −𝑚2 𝑥1 𝑚1 𝑦2 −𝑦𝑥1 𝑚1 𝑧2 −𝑚2 𝑧1
( , , )
𝑚1 −𝑚2 𝑚1 −𝑚2 𝑚1 −𝑚2

Q:- Find the coordinates of the point which divides the join of P( 2,-1,4) and Q (4,3,2) in the
ratio 2:3 internally.

P R Q
Soln:- Let R(x, y, z) be the required point then
𝑚1 𝑥2 +𝑚2 𝑥1 2 x 4+3 x2 14
x= = = ( 2,-1,4) (x, y, z) (4,3,2)
𝑚1 +𝑚2 2+3 5

𝑚1 𝑦2 +𝑚2 𝑦1 2 x3+3 x (−1) 3


y= = =5
𝑚1 +𝑚2 2+3

𝑚1 𝑧2 +𝑚2 𝑧1 2x 2+3 x4 16
z= = =
𝑚1 +𝑚2 2+3 5

14 3 16
Required point is ( 5 , 5 , )
5

3. Direction Cosines and Direction ratios of a line :-


A) If a line makes angle 𝛼 , 𝛽, 𝑎𝑛𝑑 𝛾 with x, y, z axis respectively then its direction
cosines are given by
l = Cos𝛼, m = Cos 𝛽 and n= Cos 𝛾 such that l2 + m2 +n2 = 1 .
B) The real numbers which are proportional to direction cosines of the line, are known
l m n
as direction ratios of that line, If a, b, c are three real no, then = =
a b c
C) If a, b, c are direction ratio of any line then
a b c
l = ± √a2 , m= ± √a2 , n = ± √a2
+b2 +c2 +b2 +c2 +b2 +c2
4. Angle between two lines

51
if 𝑙1 , 𝑚1 , 𝑛1 and 𝑙2 , 𝑚2 , 𝑛2 are direction cosines of two lines then Cos𝜃 = 𝑙1 𝑙2 + 𝑚1 𝑚2 +
𝑛1 𝑛2
If 𝑎1 , 𝑏1 , 𝑐1 𝑎𝑛𝑑 , 𝑎2 , 𝑏2 , 𝑐2 are direction ratios of two lines then angle between them
a1 a2 +b1 b2 +c1 c2
Cos𝜃 = .
√a21 +b21 +c21 √a22 +b22 +c22

• If two lines are perpendicular then a1 a2 + b1 b2 + c1 c2 = 0 .


𝑎 𝑏 𝑐
• If two lines are parallel then 𝑎1 = 𝑏1 = 𝑐1
2 2 2

• The direction ratios of a line passing through two points P(x1, y1,z1) and
Q(x2,y2,z2) is
(x2- x1, y2 - y1, z2- z1 ).

Q:- Find the direction ratios of a line passing through the points (2,3,4) and (-5,2,7) .

Soln:- Direction ratios of the line are = [ ( -5-2), (2-3) , (7-4) ] = ( -7,-1,3) .

5. Equation of line Passing through one point and parallel to given vector
Vector form :- Equation of a line passing through l passing through a point A with
position vector 𝑎 and parallel to a given vector 𝑏⃗ is 𝑟 = 𝑎 + 𝜆 𝑏⃗ .

𝑄: − Find the equation of the line which passes


through the point (3, 4, 5) and is parallel to the vector 2𝑖̂ + 2𝑗̂ − 3 𝑘̂ .
Soln:- Equation of line is 𝑟 = 𝑎 + 𝜆 𝑏⃗
𝑎 =3𝑖̂ + 4𝑗̂ + 5 𝑘̂ and 𝑏⃗ = 2𝑖̂ + 2𝑗̂ − 3𝑘̂.
𝑟 = 3𝑖̂ + 4𝑗̂ + 5 𝑘̂+ 𝜆 (2𝑖̂ + 2𝑗̂ − 3𝑘̂. )
Cartesian Form :- The cartesian equation of a line passing through a point A (x1, y1,z1)
𝑥−𝑥 𝑦−𝑦 𝑧−𝑧
and having direction ratios (a,b,c) is 𝑎 1 = 𝑏 1 = 𝑐 1 .
Note :- Check every time that the equation should be in standard form before getting
passing point and direction ratios.
i.e the coefficient of x,y,z must be +1, otherwise we have to make it .
2𝑥 − 3 𝑦+2 3𝑧 − 5
= =
4 2 1
3 5
𝑥− 𝑦−(−2) 𝑧−
2 3
= = 1
2 2
3
3 5 1
Passing point of line is (2,-2, ) and dr’s ( 2,2, 3 )
3
Q:- Find the equation of a line passing through a point ( 3,-2,5) and having direction
ratios (4,2,1) .

52
𝑥−3 𝑦+2 𝑧−5
Soln :- The equation of required line is = = .
4 2 1
Conversion of vector form to cartesian
Let equation of line be 𝑟 = 𝑥1 𝑖̂ + 𝑥2 𝑗̂ + 𝑥3 𝑘̂ + 𝜆 (𝑏1 𝑖̂ + 𝑏2 + 𝑏3 𝑘̂ )
Then 𝑥1 𝑖̂ + 𝑥2 𝑗̂ + 𝑥3 𝑘̂ ( 𝑥1 , 𝑥2 , 𝑥3 ) Passing point
̂
(𝑏1 𝑖̂ + 𝑏2 + 𝑏3 𝑘 ) < 𝑏1 , 𝑏2 , 𝑏3 > direction ratios
𝑥−𝑥 𝑦−𝑥 𝑧−𝑥
Equation of line in cartesian form is 𝑏 1 = 𝑏 2 = 𝑏 3 .
1 2 3
Conversion of cartesian to vector form
𝑥−𝑥1 𝑦−𝑦1 𝑧−𝑧1
Let the equation of the line be = = then
𝑎 𝑏 𝑐
Passing point ( 𝑥1 , 𝑦1 , 𝑧1 ) 𝑎 = 𝑥1 𝑖̂ + 𝑦1 𝑗̂ + 𝑧1 𝑘̂ co-ordinates
direction ratios <a,b,c> 𝑏⃗ = a𝑖̂ + 𝑏𝑗̂ + 𝑐𝑘̂ .
Equation of line is 𝑟 = 𝑎 + 𝜆 𝑏⃗
𝑟 = 𝑥1 𝑖̂ + 𝑦1 𝑗̂ + 𝑧1 𝑘̂ + 𝜆 ( a𝑖̂ + 𝑏𝑗̂ + 𝑐𝑘̂ )
𝑥−𝑥1 𝑦−𝑦 𝑧−𝑧
• The co-ordinates of any point on the line = 𝑏1= 𝑐1
𝑎
𝑥−𝑥1 𝑦−𝑦1 𝑧−𝑧1
Let = = =𝜆
𝑎 𝑏 𝑐
x= 𝑥1 + 𝑎𝜆 , y = 𝑦1 + 𝑏𝜆 z = 𝑧1 + 𝑐𝜆 (Parametric form of line )
𝑥+2 𝑦+1 𝑧−3
Q:- Find the point on the line 3 = 2 = 2 at a distance of 3√2 from the point (1,2,3)
.
𝑥+2 𝑦+1 𝑧−3
Soln:- The co-ordinates of any point on the line = = =𝜆
3 2 2
X = 3 𝜆 − 2 y = 2 𝜆 − 1 , z = 2 𝜆 + 3 ………………….(1)
Let the co-ordinates of the desired points are ( 3 𝜆 − 2 , 2 𝜆 − 1 , 2 𝜆 + 3 )
The distance between this point and (1,2,3) is 3√2 .
√(3 𝜆 − 2 − 1)2 + (2 𝜆 − 1 − 2)2 + (2 𝜆 + 3 − 3)2 = 3√2
⟹ 9( 𝜆 − 1)2 + (2 𝜆 − 3)2 + 4𝜆2 = 18
30
⟹ 17𝜆2 – 30 𝜆 = 0 ⟹ 𝜆 = 0, 17
𝑆𝑢𝑏𝑠𝑡𝑖𝑡𝑢𝑡𝑒 𝑡ℎ𝑒 𝑣𝑎𝑙𝑢𝑒 𝑜𝑓 𝜆 in (1)
56 43 111
𝑅𝑒𝑞𝑢𝑖𝑟𝑒𝑑 𝑝𝑜𝑖𝑛𝑡𝑠 will be (−2, , −1,3) and (17 , 17 , )
17
𝑥−1 𝑦−2 𝑧−3 𝑥−4 𝑦−1 𝑧
𝑄: − Show that the line = = and = = intersect. Find their point of
2 3 4 5 2 1
intersection.

𝑥−2 𝑦−2 𝑧−3


𝑆𝑜𝑙𝑛 ∶− The co-ordinates of any point on the line = = = 𝜆 (let)
2 3 4
x= 2 𝜆 + 1 , y= 3 𝜆 + 2 z= 4 𝜆 + 3
Co-ordinates of the general point on the line be ( 2 𝜆 + 1 , 3 𝜆 + 2 , 4 𝜆 + 3)
𝑥−4 𝑦−1 𝑧
The co-ordinates of any point on the line 5 = 2 = 1 = 𝜇 (let)
X = 5 𝜇 + 4 , 𝑦 = 2𝜇 + 1, 𝑧 = 𝜇
Co-ordinates of the general point on the line are ( 5 𝜇 + 4 , 2𝜇 + 1, 𝜇 )
If both the lines intersect they must have a common point for some value of 𝜆 and 𝜇
(2 𝜆 + 1 , 3 𝜆 + 2 , 4 𝜆 + 3) = (5 𝜇 + 4 , 2𝜇 + 1, 𝜇 )

53
2 𝜆 + 1 = 5 𝜇 + 4 ; 3 𝜆 + 2 = 2𝜇 + 1 ; 4 𝜆 + 3 = 𝜇
2 𝜆 - 5 𝜇 = 3 ; 3𝜆-2 𝜇 = -1 ; 4 𝜆 - 𝜇 =-3
Getting 𝜆 = -1 and 𝜇 = -1 (solving first two equations) and it satisfy the third one .
Hence both the line will intersect and Point of Intersection is (-1,-1,-1).
Equation of line passing through two points

Vector equation of line passing through the points with position vector 𝑎 & 𝑏⃗ is 𝑟 =
𝑎 + 𝜆( 𝑏⃗ - 𝑎 )
𝐶𝑎𝑟𝑡𝑒𝑠𝑖𝑎𝑛 𝑒𝑞𝑢𝑎𝑡𝑖𝑜𝑛 𝑜𝑓 𝑎 𝑠𝑡𝑟𝑎𝑖𝑔ℎ𝑡 𝑙𝑖𝑛𝑒 𝑝𝑎𝑠𝑠𝑖𝑛𝑔 𝑡ℎ𝑟𝑜𝑢𝑔ℎ 𝑡𝑤𝑜 𝑔𝑖𝑣𝑒𝑛 points A(x1, y1,z1)
𝑥−𝑥 𝑦−𝑦 𝑧−𝑧
and B(x2,y2,z2) is given by 𝑥 − 𝑥1 = 𝑦 − 𝑦1 = 𝑧 −𝑧1 .
2 1 2 1 2 1
𝑄:- Find the equation of a line through the points A ( 2,0,3) and B ( 1,5,6).
𝑆𝑜𝑙𝑛:- Cartesian form of the line
𝑥−2 𝑦 𝑧−3 𝑥−2 𝑦 𝑧−3
= = ⟹ = = .
1−2 5−0 6−3 −1 5 3
Vector equation of the line 𝑟 = 𝑎 + 𝜆( 𝑏⃗ - 𝑎 )
𝑎 =2𝑖̂ + 0𝑗̂ + 3 𝑘̂ and 𝑏⃗ = 𝑖̂ + 5𝑗̂ + 6𝑘̂.
𝑏⃗ - 𝑎 =- 𝑖̂ + 5𝑗̂ + 3𝑘̂.
𝐸𝑞𝑢𝑎𝑡𝑖𝑜𝑛 𝑜𝑓 𝑡ℎ𝑒 𝑙𝑖𝑛𝑒 𝑟 = 2𝑖̂ + 0𝑗̂ + 3 𝑘̂ + 𝜆(− 𝑖̂ + 5𝑗̂ + 3𝑘̂ )

SHORTEST DISTANCE BETWEEN TWO LINES :-


SKEW (neither parallel nor intersecting)
LINE
S
PARALLEL
Shortest Distance:- The shortest distance between the lines which is perpendicular to both
the line .
1. Shortest distance between skew lines :- Let 𝑙1 and 𝑙2 be
two skew lines given by
𝑟 = 𝑎1 + 𝜆 ⃗b1 and 𝑟 = 𝑎2 + 𝜇b ⃗ 2 respectively
then
⃗ 1 Xb
(𝑎⃗2 −𝑎⃗1 ).(b ⃗ 2)
𝑆. 𝐷 = | ⃗ 1 Xb
⃗ 2|
|
|b
𝑺𝒕𝒆𝒑𝒔 for calculation
A) Identify 𝑎1 , 𝑎2 , ⃗b1 and ⃗b2
B) Evaluate 𝑎2 - 𝑎1

54
C) Evaluate ⃗b1 X⃗⃗b2
D) Evaluate |b⃗ 1 Xb
⃗ 2|
E) Replace appropriate values and get the solution.
Q:- Find the shortest distance between the lines 𝑟 = 𝑖̂ + 2𝑗̂ + 𝑘̂ + 𝜆( 𝑖̂ − 𝑗̂ + 𝑘̂ )
⃗⃗𝑟 = 2𝑖̂ − 𝑗̂ − 𝑘̂ + 𝜇 ( 2𝑖̂ + 𝑗̂ + 2𝑘̂ ).
𝑆𝑜𝑙:- A) 𝑎1 = 𝑖̂ + 2𝑗̂ + 𝑘̂ 𝑎2 = 2𝑖̂ − 𝑗̂ − 𝑘̂
⃗b1 = 𝑖̂ − 𝑗̂ + 𝑘̂ ⃗b2 = 2𝑖̂ + 𝑗̂ + 2𝑘̂
B) 𝑎2 - 𝑎1 = 𝑖̂ − 3𝑗̂ − 2 𝑘̂
𝑖̂ 𝑗̂ 𝑘̂
C) ⃗
b1 X b2 = |1 −1 1| = -3𝑖̂ +3 𝑘̂
⃗⃗
2 1 2
D) ⃗ ⃗
|b1 Xb2 | = √(−3)2 + (3)2 = 3√2
⃗ 1 Xb
(𝑎⃗2 −𝑎⃗1 ).(b ⃗ 2)
E) 𝑆. 𝐷 = | ⃗ 1 Xb
⃗ 2|
|
|b
̂ ).(−3𝑖̂ +3 𝑘
(𝑖̂−3𝑗̂ −2 𝑘 ̂)
= | |
3√2
−3−6 9 3√2
= | 3√2 | = 3√2 = unit
2

𝑥−1 𝑦+1 𝑧 𝑥+1 𝑦−2 𝑧−2


Q:- Find the shortest distance between the lines = = and = =
2 3 1 5 1 0
𝑥−1 𝑦−(−1) 𝑧 𝑥−(−1) 𝑦−2 𝑧−2
Sol:- Equation of lines = = and = =
2 3 1 5 1 0
A) 𝑎1 = 𝑖̂ − 𝑗̂ 𝑎2 = −𝑖̂ + 2𝑗̂ + 2𝑘̂
⃗b1 = 2𝑖̂ + 3𝑗̂ + 𝑘̂ ⃗b2 = 5𝑖̂ + 𝑗̂
B) 𝑎2 - 𝑎1 = -2𝑖̂ + 3𝑗̂ + 2 𝑘̂
𝑖̂ 𝑗̂ 𝑘̂
C) ⃗b1 X⃗⃗b2 = |2 3 1| = -𝑖̂ + 5𝑗̂ − 13𝑘̂
5 1 0
D) ⃗ ⃗
|b1 Xb2 | = √(−1)2 + (5)2 + (−13)2 = √195
⃗ 1 Xb
(𝑎⃗2 −𝑎⃗1 ).(b ⃗ 2)
E) 𝑆. 𝐷 = | ⃗ ⃗
|
|b1 Xb2 |
̂ ).( −𝑖̂+5𝑗̂ −13𝑘̂ )
(−2𝑖̂+3𝑗̂ +2 𝑘
=| |
√195
2+15−26 9
=| | = unit
√195 √195

2. Shortest distance between Parallel lines:-


Let 𝑙1 and 𝑙2 be two Parallel lines given by
𝑟 = 𝑎1 + 𝜆𝑏⃗ and 𝑟 = 𝑎2 + 𝜇𝑏⃗
respectively then
⃗)
(𝑎⃗2 −𝑎⃗1 )𝑋 𝑏
𝑆. 𝐷 = | ⃗|
|
|𝑏
𝑺𝒕𝒆𝒑𝒔 for calculation
A) Identify 𝑎1 , 𝑎2 , and 𝑏⃗
B) Evaluate 𝑎2 - 𝑎1
C) Evaluate (𝑎2 − 𝑎1 )𝑋 𝑏⃗)
55
D) Evaluate |𝑏⃗|
E) Replace appropriate values and get the solution.
Q:-Find the shortest distance between the 𝑙𝑖𝑛𝑒𝑠 𝑟 = 𝑖̂ + 2𝑗̂ + 3𝑘̂ + (2 𝑖̂ + 3𝑗̂ + 4𝑘̂ )
⃗⃗𝑟 = 2𝑖̂ + 4𝑗̂ + 5𝑘̂ + 𝜇 ( 2𝑖̂ + 3𝑗̂ + 4𝑘̂ ).
𝑆𝑜𝑙: − :- A) 𝑎1 = 𝑖̂ + 2𝑗̂ + 3𝑘̂ , 𝑎2 = 2𝑖̂ + 4𝑗̂ + 5𝑘̂ & 𝑏⃗ = 2 𝑖̂ + 3𝑗̂ + 4𝑘̂
B) 𝑎2 - 𝑎1 = 𝑖̂ + 2𝑗̂ + 2 𝑘̂
𝑖̂ 𝑗̂ 𝑘̂
C) (𝑎2 - 𝑎1 ) X 𝑏⃗ = |1 2 2| = 2𝑖̂ − 𝑘̂
2 3 4
D) | 𝑏⃗| = √(2) + (3)2 + (4)2 = √29
2

⃗)
(𝑎⃗2 −𝑎⃗1 )𝑋 𝑏
E) 𝑆. 𝐷 = | ⃗ |
|𝑏
|
̂
2𝑖̂−𝑘
= | |
√29
√(2)2 +(−1)2 √5
=| | = unit .
√29 √29

The Plane
Definition:- A plane is surface such that if any two point are taken on it , the line
segment joining them lies completely on the surface .
Standard form of plane is given by ax + by + cz + d = 0
Where <a,b,c> represent direction ratios of normal of the plane .
Equation of plane
1. Passing through one point A (x1, y1,z1) is a(x-𝑥1 ) + b( y-𝑦1 ) +c(z-𝑧1 ) =0 where 𝑎, 𝑏, 𝑐 are
direction ratios of normal to the plane
2. Passing through three points A(x1, y1,z1) , B(x2,y2,z2) & C( 𝑥3 , 𝑦3 , 𝑧3 ) is given by
𝑥 − 𝐱 𝟏 𝑦 − 𝑦1 𝑧 − 𝑧1
| 2 − 𝑥1 𝑦2 − 𝑦1 𝑧2 − 𝑧1 | = 0
𝑥
𝑥3 − 𝑥1 𝑦3 − 𝑦1 𝑧3 − 𝑧2

Q:- Find the equation of the plane passing through the points (2,2,-1) B(3,4,2) and C ( 7,0,6)

Soln:- The equation of plane passing through the points (2,2,-1) B(3,4,2) and C ( 7,0,6) .
𝑥−2 𝑦−2 𝑧+1
Is given by | 1 2 3 |=0
5 −2 7
⟹ (x-2)(14+6) - (y-2)(7-15) + (z+1) (-2-10) = 0 ⟹ 20x-40 +8y-16 -12z-12 =0
⟹ 20x +8y -12z =68 ⟹ 5x + 2y -3z= 17
Alternative method:-
The general equation of a plane passing through (2,2,-1) is
a (x-2) + b (y-2) + c(z +1) = 0 …………………………(1)
It will pass through the points B(3,4,2) and C ( 7,0,6) then
a ( 3-2) + b(4-2) + c( 2+1) =0 ⟹ a + 2b + 3c = 0 …………….(2)
and a ( 7-2) + b ( 0-2) + c( 6 + 1 ) = 0 ⟹ 5a -2b +7c = 0 …………… (3)
Solving (2) and (3) by cross multiplication

56
a b c
= 15−7 = −2−10 a b c a b
14+6

1 2 3 1 2

5 -2 7 5 -2
a b c
= 8 = −12 = 𝜆 (say) ⟹ a= 20 𝜆 , b = 8 𝜆 , c = -12 𝜆
20
Replace them in equation (1)
20 𝜆 (x-2) + 8 𝜆 (y-2) - 12 𝜆 (z +1) = 0 or 5x +2y -3z =17

Co-Planarity :- Four points are said to be co-planar if they lie on the same plane .
For proving this get the equation of plane passing through three points and verify with
the fourth one.
Q:- Show that the four points A(0,-1,-1) ( -4,4,4) (4,5,1) and ( 3,9,4) are co-planar
Solution:- The equation of plane through the points A(0,-1,-1) ( -4,4,4) (4,5,1) is given by
𝑥−0 𝑦+1 𝑧+1
| −4 5 5 |=0
4 6 2
⟹ x(10-30) - (y+1)(-8-20) + (z+1)(-24-20) = 0
⟹ -20x +28y +28 -44z -44 = 0 ⟹ 5x -7y + 11z +4 =0
𝑛𝑜𝑤 𝑖𝑓 place the point ( 3,9,4) then ( 5 x3) –(7 x 9) + (11 x4) +4 = 0
⟹15-63 + 44 +4 =0 which satisfy the equation of plane .
𝐻𝑒𝑛𝑐𝑒 𝑎𝑙𝑙 𝑡ℎ𝑒 𝑝𝑜𝑖𝑛𝑡𝑠 𝑎𝑟𝑒 𝐶𝑜 − 𝑝𝑙𝑎𝑛𝑛𝑒𝑟.
𝑰𝒏𝒕𝒆𝒓𝒄𝒆𝒑𝒕 form of plane :- The equation of a plane intercepting lengths a,b,c
x y z
with x-axis , y –axis and z-axis is given by + + = 1.
a b c
Q:- Write the equation of plane whose intercepts on the
coordinate axes are -4, 2 and 3 .
Solution:- The equation of plane will be
x y z
+ 2 + 3 = 1. ⟹ -3x+6y +4z = 12
−4

𝑽𝑬𝑪𝑻𝑶𝑹 𝑬𝑸𝑼𝑨𝑻𝑰𝑶𝑵 𝑶𝑭 𝑨 𝑷𝑳𝑨𝑵𝑬


1) The vector equation of a plane passing through a
point having position vector 𝑎 and normal to vector 𝑛⃗
is (𝑟 − 𝑎) . 𝑛⃗ = 0
⟹ 𝑟. 𝑛⃗ = 𝑎 . 𝑛⃗ =d (let)
Q:- Find the vector equation of a plane passing through a
point having a position vector 2𝑖̂ + 3𝑗̂ − 4𝑘̂ and
perpendicular to the vector 2𝑖̂ − 𝑗̂ + 2𝑘̂ .
Solution:- 𝑎 = 2𝑖̂ + 3𝑗̂ − 4𝑘̂ 𝑛⃗ = 2𝑖̂ − 𝑗̂ + 2𝑘̂
The equation of plane is given by ⃗⃗𝑟. 𝑛⃗ = 𝑎 . 𝑛⃗
⟹ ⃗⃗𝑟. ( 2𝑖̂ − 𝑗̂ + 2𝑘 ) = (2𝑖̂ + 3𝑗̂ − 4𝑘̂)( 2𝑖̂ − 𝑗̂ + 2𝑘̂)
̂

57
⟹ ⃗⃗𝑟. ( 2𝑖̂ − 𝑗̂ + 2𝑘̂ ) = -7 .
Reduction of equation of plane from vector form to Cartesian form
Let Equation of plane be 𝑟 . 𝑛⃗ = d then substitute 𝑟 =x 𝑖̂ + 𝑦𝑗̂ + 𝑧𝑘̂ and perform dot
product .
e.g. ⃗⃗𝑟. ( 2𝑖̂ − 𝑗̂ + 2𝑘̂ ) = -7
(x 𝑖̂ + 𝑦𝑗̂ + 𝑧𝑘̂ ). ( 2𝑖̂ − 𝑗̂ + 2𝑘̂ ) = -7 or 2x –y +2z = -7
Reduction of equation of plane from cartesian form to vector form
Let equation of plane be ax + by +cz +d = 0 then its vector equation
will be
⃗⃗𝑟. ( 𝑎𝑖̂ + 𝑏𝑗̂ + 𝑐𝑘̂ ) +d = 0
(a, b, c) 𝑎𝑖̂ + 𝑏𝑗̂ + 𝑐𝑘̂ then replace it in place of 𝑛⃗ .
Q:- Write the equation of plane into vector form : 3x-2y + 5z = 9
Solution:- Equation of plane 3x-2y + 5z – 9=0
⟹ ⃗⃗𝑟. ( 3𝑖̂ − 2𝑗̂ + 5𝑘̂ ) -9 = 0
𝑄: − The foot of perpendicular drawn from the origin to the plane is P( 4,-2,-5). Find the
equation of the plane. O
𝑆𝑜𝑙𝑢𝑡𝑖𝑜𝑛 :-

Let Equation of plane is 𝑟. 𝑛⃗ = 𝑎 . 𝑛⃗


𝑎 = 4𝑖̂ − 2𝑗̂ − 5𝑘̂ 𝑛⃗ = 𝑂𝑃
⃗⃗⃗⃗⃗ = 4𝑖̂ − 2𝑗̂ − 5𝑘̂ P
The equation of plane is 𝑟. (4𝑖̂ − 2𝑗̂ − 5𝑘̂) = (4𝑖̂ − 2𝑗̂ − 5𝑘̂). ( 4𝑖̂ − 2𝑗̂ − 5𝑘̂ )
𝑟. (4𝑖̂ − 2𝑗̂ − 5𝑘̂) = 45
Equation of plane in normal form :- The vector
equation of a plane normal to unit vector 𝑛̂ and at
a distance d from the origin is 𝑟. 𝑛̂ = 𝑑
Q:- Find the Vector equation of the plane which is at a
distance of 6 units from origin and has 2,-1,2 as the
direction ratios of a normal to it .
Solution :- Let the equation of the plane be 𝑟. 𝑛̂ = 𝑑
𝑛⃗ = 2𝑖̂ − 𝑗̂ + 2𝑘̂ d= 6

𝑛 ̂
2𝑖̂−𝑗̂ + 2𝑘
𝑛̂ = |𝑛⃗| = 3
̂
2𝑖̂−𝑗̂ + 2𝑘
𝑅𝑒𝑞𝑢𝑖𝑟𝑒𝑑 𝑒𝑞𝑢𝑎𝑡𝑖𝑜𝑛 𝑜𝑓 𝑝𝑙𝑎𝑛𝑒 𝑖𝑠 𝑟. ( )=6
3
2) Vector equation of plane passing through three points :- Let A,B,C be
three points on a plane having their position vectors 𝑎 , 𝑏⃗ and 𝑐 respectively. Then
Equation of a plane passing three points is given by
( 𝑟 − 𝑎) . ( ⃗⃗⃗⃗⃗⃗
𝐴𝐵 𝑋 ⃗⃗⃗⃗⃗
𝐴𝐶 ) = 0 Q:-
Find the vector equation of the plane passing through the point A( 1,1,1), B ( 1,-1,1)
and C ( -7,-3,-5) .
Solution :- 𝑎 = 𝑖̂ + 𝑗̂ + 𝑘̂ ⃗⃗⃗⃗⃗⃗
𝐴𝐵 = = -2𝑗̂ 𝐴𝐶 =-8𝑖̂ − 2𝑗̂ − 6 𝑘̂
⃗⃗⃗⃗⃗

58
𝑖̂ 𝑗̂ 𝑘̂
𝐴𝐵 𝑋 𝐴𝐶 = | 0 −2 0 | = 12𝑖̂ - 16𝑘̂
⃗⃗⃗⃗⃗⃗ ⃗⃗⃗⃗⃗
−8 −2 −6
The required equation of plane passes through point A( 1,1,1) whose
position vector is
( 𝑟 − 𝑎) . ( ⃗⃗⃗⃗⃗⃗ ⃗⃗⃗⃗⃗ ) = 0
𝐴𝐵 𝑋 𝐴𝐶
[ 𝑟 − (𝑖̂ + 𝑗̂ + 𝑘̂ ) ] .( 12𝑖̂ - 16𝑘̂ ) = 0
𝑟 .( 12𝑖̂ - 16𝑘̂ ) + 4 = 0
𝑟 . ( 3𝑖̂ − 4𝑘̂ ) + 1 = 0
Equation of plane passing through the intersection of given two planes :-
Let equation of two planes be
𝜋1 ∶ 𝑎1 𝑥 + 𝑏1 𝑦 + 𝑐1 𝑧 + 𝑑1 = 0 and 𝜋2 ∶ 𝑎2 𝑥 +
𝑏2 𝑦 + 𝑐2 𝑧 + 𝑑2 = 0 then equation of plane passing
through intersection of given two planes is given by
𝜋1 + 𝜆𝜋2 = 0
i.e 𝑎1 𝑥 + 𝑏1 𝑦 + 𝑐1 𝑧 + 𝑑1 + 𝜆 ( 𝑎2 𝑥 + 𝑏2 𝑦 + 𝑐2 𝑧 +
𝑑2 ) = 0
The value of 𝜆 will be calculated based on additional information provided by individual
problems .
Q:- Find the equation of a plane containing the line of the plane x + y + z -6 = 0 and 2x +
3y + 4z + 5 = 0 and passing through the point (1,1,1) .

Solution :- let 𝜋1 : x + y + z -6 = 0 and 𝜋2 ∶ 2x + 3y + 4z + 5 = 0 then equation of plane


passing through the intersection of given two planes is 𝜋1 + 𝜆𝜋2 = 0
x + y + z -6 + 𝜆( 2x + 3y + 4z + 5) = 0 ……………………….(1)
Equation (1) passing through (1,1,1) then
1 + 1 + 1 – 6 + 𝜆 ( 2 + 3 + 4+ 5 ) = 0
3
⟹ 𝜆 = 14
3
Replace the value of 𝜆 in equation(1) i.e x + y + z -6 +14 ( 2x + 3y + 4z + 5) = 0
⟹ 20x + 23y + 26z -69 = 0 .
Note:- if Equation of Plane is given in vector form then convert it into Cartesian form and
then use this Process .

Distance of A point from a plane :- The length of the perpendicular from point P(x1,
y1,z1) to the plane
ax + by + cz + d = 0 is given by
|ax1 +by1 +cz1 +d|
√a2 +b2 +c2
Q:- Find the distance of the point (2,1,1) from the plane x -
2y +4z = 9 .

59
Solution :- The distance of the point (2,1,1,) from the plane x -2y +4z = 9 is given by
|ax1 +by1 +cz1 +d|
√a2 +b2 +c2
|2−2 ×1+4 ×1| 4
Required distance = = unit
√12 +(−2)2 +4 2 √21

Q:-2 A variable plane which remains at a constant distance 3p from the origin cut the co-
1 1 1 1
ordinates axes A,B,C . Show that the locus of the centroid of the triangle ABC is 𝑥 2 + 𝑦 2 + 𝑧 2 = 𝑝2 .
x y z
Solution:- Let the equation of plane be + b + c = 1. ……………….(1)
a
The plane meets at point A(a,0,0) B(0,b,0) and C(0,0,c).
Let ( 𝛼 , 𝛽, 𝛾) be the centre of triangle ABC,Then
a+0+0 a
𝛼 = 3 = 3 ⟹ a = 3 𝛼 ----------(2)
0+b+0 b
𝛽 = = 3 ⟹ a = 3 𝛽 ……………(3)
3
0+0+c c
𝛾= =3 ⟹c=3𝛾 …………….(4)
3
The plane (1) is at a distance 3p from the origin .
3p = Length of perpendicular from (0,0,0) to the plane (1)
0 0 0
| + + −1| 1
3p = 𝑎 𝑏 𝑐
2 2 2
⟹ 3p = 2 2 2
√( 1 ) +(1 ) +(1) √( 1 ) +(1) +(1)
𝑎 𝑏 𝑐 𝑎 𝑏 𝑐
1 1 2 1 2 1 2
⟹ = ( ) + ( ) + ( ) ------------------(5)
9𝑝2 𝑎 𝑏 𝑐
1 1 2 1 2 1 2
Replace the value of a,b,c by using (2),(3),(4) = (3 𝛼) + (3 𝛽) + (3 𝛾)
9𝑝2
1 1 1 1 1 1 1 1
= + + then locus of ( 𝛼 , 𝛽, 𝛾) is + + 𝑧 2 = 𝑝2 .
𝑝2 𝛼2 𝛽2 𝛾2 𝑥2 𝑦2
𝑥−𝑥 𝑦−𝑦 𝑧−𝑧1 𝑥−𝑥 𝑦−𝑦 𝑧−𝑧
Co-planarity of two lines :- If the line 𝑎 1 = 𝑏 1 = 𝑐 and 𝑎 2 = 𝑏 2 = 𝑐 2
1 1 1 2 2 2
𝑥2 − 𝐱 𝟏 𝑦2 − 𝑦1 𝑧2 − 𝑧1
Are Co-planner ,then | 𝑎1 𝑏1 𝑐1 | = 0 and
𝑎2 𝑏2 𝑐2
𝑥 − 𝐱 𝟏 𝑦 − 𝑦1 𝑧 − 𝑧1
Equation of plane containing those lines will be | 𝑎1 𝑏1 𝑐1 | = 0
𝑎2 𝑏2 𝑐2
𝑥 − 𝐱 𝟐 𝑦 − 𝑦2 𝑧 − 𝑧2
or . | 𝑎1 𝑏1 𝑐1 | =0
𝑎2 𝑏2 𝑐2
𝑥+1 𝑦+3 𝑧+5 𝑥−2 𝑦−4 𝑧−6
Q:- Prove that the lines 3
=
5
=
7
and
1
=
4
=
7
are Coplanar .also find the
plane containing these two lines .
𝑥+1 𝑦+3 𝑧+5 𝑥−2 𝑦−4 𝑧−6
Solution :- Given Lines are 3 = = and = =
5 7 1 4 7
(x1, y1,z1) = ( -1,-3,-5 ) and (x2,y2,z2) = (2,4,6,)
𝑥2 − 𝐱 𝟏 𝑦2 − 𝑦1 𝑧2 − 𝑧1
Lines will be co-planar if | 𝑎1 𝑏1 𝑐1 | = 0
𝑎2 𝑏2 𝑐2

60
𝑥2 − 𝐱 𝟏 𝑦2 − 𝑦1 𝑧2 − 𝑧1 3 7 11
| 𝑎1 𝑏1 𝑐1 | = |3 5 7 | =
𝑎2 𝑏2 𝑐2 1 4 7
=21 -98 + 77 = 0 Lines are co-planar.
𝑥+1 𝑦+3 𝑧+5
Equation of required plane is be | 3 5 7 | =0 ⟹ x- 2y +z = 0
1 4 7
𝑥−2 𝑦−4 𝑧−6
Or | 3 5 7 | = 0 ⟹ x- 2y +z = 0
1 4 7
Level I (2 MARK )
1. Write the direction cosines of the line equally inclined to the three co-ordinate axes.
2. Reduce the equation of the plane 3x + 4y – z + 7 = 0 in the normal form and hence find its
distance from origin.
3. Find the distance between the parallel planes : ⃗⃗𝑟 . ( 2𝑖̂ − 𝑗̂ + 3 ̂𝑘 ) = 4 𝑎𝑛𝑑 𝑟 . ( 6𝑖̂ − 3 𝑗̂ +
9 𝑘̂ ) + 13 = 0
2x −1 4 − y z + 1
4. Cartesian equation of line AB is = = . Write the direction ratio’s of a line
2 7 2
parallel to AB.
5. Find the direction cosines of a line passing through the points (– 1, 0, 2) and (3, 4, 6).
6. Find the equation of the plane passing through the point (1,7,0) and perpendicular to the
vector (4𝑖̂ − 𝑗̂ + 5 𝑘̂ ).
7. Find the length of the perpendicular from origin to the plane x-2y+2z-9=0.
8. The foot of the perpendicular drawn from the origin to a plane is (2, 5, 7). Find the
equation of the plane.
9. Find the equation of the plane through (2, 3,4 ) and parallel to the plane 5x – 6y + 7z = 3.
10. Find the intercepts cut off by the plane 2x + y – z = 5 on the respective axes.
11. If the points (1, 1, p) and (–3, 0, 1) be equidistant from the plane 𝑟. ( 3𝑖̂ + 4 𝑗̂ − 12 𝑘̂) +
13 = 0 then find the value of p.
12. If a line makes angle 𝛼, 𝛽, 𝛾 with positive directions of x- axis, y-axis and z-axis
respectively, then find the value of sin 2  + sin 2  + sin 2  .
x − 2 y −1 z + 5
13. Find the value of 𝜆 if line = = , is ⊥ to the plane 3x – y – 2z = 7.
6  −4
14. If the equation of a line is x = ay + b, z = cy + d , then find the passing point and direction
ratios of the line .
15. Find the vector equation of a plane whose Cartesian equation is 2x+3y-4z+7=0.
16. Find the distance of a point (2, 5, -3) from the plane 𝑟. ( 6𝑖̂ − 3 𝑗̂ + 2 𝑘̂) = 4
17. A line makes an angle π/4 with each of x-axis and y-axis. Find the angle between this line
and the z-axis.

61
Answers of level I
1 10 5 1
a = 2 b= 5 c=
−1
cos −1 ( ) 5
1 √3
2 𝑟 .(
̂ +7
3𝑖̂+4𝑗̂ − 𝑘
)=0 11. p=3
7
√26
3 25
unit 12 2
3√14
4 𝑖̂
𝑟 = (2 + 4𝑗̂ -𝑘̂ ) + 𝜆 ( 𝑖̂ − 7𝑗̂ + 2 𝑘̂ ) 13 𝜆 = -2

5 DIRECTION COSINE = (
1
,
1
,
1
) 14 Direction ratio= < a,1,c>
√3 √3 √3
passing point =( b,0,d)
6 4x-y + 5z +3 = 0 15 𝑟 . (2𝑖̂ + 3𝑗̂ − 4𝑘̂ ) + 7 = 0
7 3 unit 16 Perpendicular distance =
13
7
8 2x+5y +7z-78=0 17 900
9 5x-6y +7z-20=0
LEVEL II (3 / 4 MARKS)
1. Find the shortest distance between the lines l1 and l2 given by the following :

x −3 y −5 z −7 x +1 y +1 z +1
l1 : = = , l2 = = =
1 −2 1 7 −6 1
2. Find the foot of the perpendicular drawn from the point P(1,1,2) to the plane 2x + y – z = 5.
3. Find the equation of plane which contains the line of intersection of planes 𝑟⃗⃗ . ( 𝑖̂ + 𝑗̂ +
̂𝑘 ) = 1 𝑎𝑛𝑑 𝑟 . ( 2𝑖̂ + 3 𝑗̂ − 𝑘̂ ) + 4 = 0 and parallel to X – axis.

4. Find the equation of plane(s) passing through the intersection of planes x + 3y + 6 = 0 and
3x – y - 4z = 0. and whose perpendicular distance from origin is unity.
5. Find the vector equation of the plane passing through the intersection of two planes 𝑟⃗⃗ . ( 𝑖̂ +
3 𝑗̂ − ̂𝑘 ) = 5 𝑎𝑛𝑑 𝑟 . ( 2𝑖̂ – 𝑗̂ + 𝑘̂ ) = 3 and the point ( 2 ,1, 3).
6. Find the equation of the plane passing through the point (–1, –1, 2) and perpendicular to
each of the following planes: 2 x + 3 y − 3 z = 2 and 5 x − 4 y + z = 6 .
7. Find the distance of the point (1, –2, 1 ) from the plane x-y+z=5 measured along a line parallel to
x y z
2
= 3
= −1
.

8. Find the distance of the point (– 1, – 5, – 10) from the point of intersection of the line 𝑟⃗⃗ =
(2𝑖̂ − 𝑗̂ + 2 ̂𝑘) + 𝜆( 3𝑖̂ + 4𝑗̂ + 2 ̂𝑘 ) 𝑎𝑛𝑑 and the plane 𝑟 . ( 𝑖̂ − 𝑗̂ + 𝑘̂ ) = 5 .
9. Find the image of point P(3,2,1) with respect to the plane 2x – y + z +1 = 0..
x − 4 y + 3 z +1 x − 1 y + 1 z + 10
10. Prove that the lines = = and = = intersect each other
1 −4 7 2 −3 8
and find the point of intersection.
11. Find the equation of a line passing through the point (2,1,3) and perpendicular to the line
x −1 y − 2 z − 3 x y z
= = and = = .
1 2 3 −3 2 5
62
12. Find the coordinates of the point where the line passing through the points A(3, 4, 1) and
B(5, 1, 6) crosses the XY-plane.
13. Find the coordinates of the point where the line through (3, -4, -5) and ( 2, -3, 1) crosses
the planes 2𝑥 + 𝑦 + 𝑧 = 7.
14. Find the vector equation of the line passing through (1, 2, 3) and parallel to the planes
𝑟. (𝑖̂ − 𝑗̂ + 2𝑘̂) = 5 and 𝑟. (3𝑖̂ + 𝑗̂ + 𝑘̂) = 6
𝑥+3 𝑦−1 𝑧−5 𝑥+1 𝑦−2 𝑧−5
15.Show that the lines −3 = 1 = , = = are coplanar. Also find the equation of
5 −1 2 5
the plane containing the lines.
x y −1 z − 2
16. Find the image of the point (1, 6, 3) on the line = = .
1 2 3
Answers of Level II
1 √116 9 (-1,4,-1)
2. 7 5 4
Foot of perpendicular =(3 , 3 , 3) 10 Point of intersection (5,-7,6)
3. ⃗⃗𝑟 . ( − 𝑗̂ + 3 ̂𝑘 ) = 5 11 𝑥−2 𝑦−1 𝑧−3
= =
2 −7 4
4. 2x+y -2z +3 =0 , x-2y-2z – 3 = 0 12 13 23
( 5 , 5 , 0)
5 𝑟⃗⃗ . ( 3 𝑖̂ + 2̂𝑗 ) − 8 = 0 13 (1,-2,7)
6. 9x+17y +23z-20=0 14 ⃗⃗𝑟 = (𝑖̂ + 2𝑗̂ + 3 ̂𝑘) + 𝜆(− 3𝑖̂ + 5𝑗̂ + 4 ̂𝑘 )
7. √14
unit 15 x-2y +z=0 ,
2
8 13 unit 16 Image point (1,0,7)

63
PROBABILITY
SOME DEFINITIONS:
(1) Event: A subset of the sample space associated with a random experiment is called an event
or a case.
e.g. In tossing a coin, getting either head or tail is an event
(2) Equally Likely Events: The given events are said to be equally likely if none of them is
expected to occur in preference to the other.
e.g. In throwing an unbiased die, all the six faces are equally likely to come.

(3) Mutually Exclusive Events: A set of events is said to be mutually exclusive, if the happening
of one excludes the happening of the other, i.e. if A and B are mutually exclusive,
then (A ∩ B) = Φ
e.g. In throwing a die, all the 6 faces numbered 1 to 6 are mutually exclusive, since if any one of
these faces comes, then the possibility of others in the same trial is ruled out.

(4) Exhaustive Events: Two or more events associated with a random experiment are
exhaustive If their union is the sample space i.e. events E1 , E2 , E3 ............. En associated with a
random
experiment with sample space S are exhaustive if 𝐸1 ∪ E2 ∪ E3 … . .∪ En = S. In a single throw of
die let we consider the events 𝐸1 = {1,2} 𝐸2 = {2,3} 𝑎𝑛𝑑 𝐸3 ={3,4,5,6}. For this we observe
That 𝐸1 ∪ E2 ∪ E3 = {1,2,3,4,5,6} = S It shows that E1 , E2 and E3 are exhaustive events
(5) Probability of an Event: If there are n elementary events associated with a random
experiment and m of them are favourable to an event E, then the probability of occurrence of E
Favourable number of elementary events 𝑚
Is denoted by P(E) and is defined as P(E)= =
𝑇𝑜𝑡𝑎𝑙 𝑛𝑢𝑚𝑏𝑒𝑟 𝑜𝑓 𝑒𝑙𝑒𝑚𝑒𝑛𝑡𝑎𝑟𝑦 𝑒𝑣𝑒𝑛𝑡𝑠 𝑛

Note:
(i) 0 ≤ P(A) ≤ 1
(ii) Probability of an impossible event is zero.
(iii) Probability of certain event (possible event) is 1 that is P(S)=1
(iv) P(A)+P(𝐴, )=1= P(S) ⟹ P(𝐴, )=1-P(A)
(iv) P(A ∪ A’) = P(S)
(v) P(A ∩ A’) = P(Φ)
(vi) P(A’)’ = P(A)
(vii) P(A ∪ B) = P(A) + P(B) – P(A ∩ B)
(viii) P (A U B U C) = P(A) + P(B) + P(C) - P(A ∩ B) - P(A ∩ C) - P(B ∩ C) + P(A ∩ B ∩ C)
̅̅̅̅̅̅̅̅̅
(ix) P(𝐴̅ ∩ 𝐵̅ )=P(𝐴 ∪ 𝐵) =1- P(A∪ 𝐵)=P(𝐴̅)P(𝐵̅)

6 Conditional Probability: Let E and F be two events associated with the same sample space of a
random experiment. Then, probability of occurrence of event E, when the event F has already
occurred, is called a conditional probability of event E over F and is denoted by P(E/F).
𝐸 𝑃(𝐸∩𝐹)
𝑃 (𝐹 ) = 𝑃(𝐹) Where P(F)≠ 0
Similarly, conditional probability of event F over E is given as
𝐹 𝑃(𝐸∩𝐹)
𝑃 (𝐸) = 𝑃(𝐸) Where P(E)≠ 0
7 Properties of Conditional Probability: If E and E are two events of sample space S and G is an
event of S which has already occurred such that P(G) ≠ 0, then
(i) P[(E ∪ F)/G] = P(E/G) + P(F/G) – P[(F ∩ F)/G], P(G) ≠ 0

64
(ii) P[(E ∪ F)/G] = P(E/G) + P(F/G), if E and F are disjoint events.
(iii) P(F’/G) = 1 – P(F/G)
(iv) P(S/E) = P(E/E) = 1
𝑃(𝐸̅ ∩𝐹) 𝑃(𝐹)−𝑃(𝐸∩𝐹)
̅ /𝐹) =
(v)P(𝐸 =
𝑃(𝐹) 𝑃(𝐹)
𝑃(𝐸̅ ∩𝐹̅ ) 1−𝑃(𝐸∪𝐹)
(vi) P(𝐸̅ /𝐹̅ ) = =
𝑃(𝐹̅ ) 1−𝑃(𝐹)

PROBLEMS
Q1 If P(E) = 0.35, P(F) = 0.45 and P(EUF) = 0.65, find P(E/F) and P(F/E).
Sol: 𝑃(𝐸 ∪ 𝐹) = 𝑃(𝐸) + 𝑃(𝐹) − 𝑃(𝐸 ∩ 𝐹) ⇒ 0.65 = 0.35 + 0.45 − 𝑃(𝐸 ∩ 𝐹) ⇒
𝑃(𝐸 ∩ 𝐹) = 0.8 − 0.65 = 0.15
E 𝑃(𝐸 ∩ 𝐹) 0.15 1 F 𝑃(𝐸 ∩ 𝐹) 0.15 3
∴ P( ) = = = P( ) = = =
F 𝑃(𝐹) 0.45 3 E 𝑃(𝐸) 0.35 7
Q2 Susan took two tests. The probability of her passing both tests is 0.6. The probability of her
passing the first test is 0.8. What is the probability of her passing the second test given that she
has passed the first test?
Sol:
𝑆𝑒𝑐𝑜𝑛𝑑 𝑃(𝑠𝑒𝑐𝑜𝑛𝑑∩𝐹𝑖𝑟𝑠𝑡) .6
𝑃 ( 𝐹𝑖𝑟𝑠𝑡 ) = = . .8 = .75
𝑃(𝐹𝑖𝑟𝑠𝑡)
Q3 What is the probability that the total of two dice will be greater than 9, given that the first die
is a 5?
Solution:
Let A = first die is 5 Let B = total of two dice is greater than 9
1
P(A) = 6 Possible outcomes for A ∩ B ={(5, 5), (5, 6)}
2 1
P(A ∩B) = 36 = 18
𝐵 𝑃(𝐵∩𝐴) 1 1 1
𝑃 (𝐴 ) = =18 ÷ 6 = 3
𝑃(𝐴)
Q4 In a school there are 1000 students, out of which 430 are girls. It is known that out of 430, 1
0% of the girls study in class XII. What is the probability that a student chosen randomly studies i
n class XII given that the chosen student is a girl?
Ans. Let E: student chosen randomly studies in class XII,
F: randomly chosen student is girl.
430 43
𝑃(𝐹) = 1000 = .43 𝑃(𝐸 ∩ 𝐹) = 1000 = .043

𝑃(𝐸∩𝐹) 043
P(E/F)= =. .43 = .1
𝑃(𝐹)
Q5. A die thrown three times. Events A and B are defined as below.
A : 4 on the third throw B : 6 on the first and 5 on the second throw.
Find the probability of A given that B has already occurred.
Ans. Total sample space = 216

B={ (6,5,1), (6,5,2), (6,5,3), (6,5,4), (6,5,5), (6,5,6)}


6 1
A∩ 𝐵 = {(6,5,4)} P(B)=216 P(A∩ 𝐵) = 216

65
𝑃(𝐴∩𝐵) 1 6 1
P(A/B)= = ÷ =
𝑃(𝐵) 216 216 6

8 Multiplication Theorem: If E and F are two events associated with a sample space S, then the
probability of simultaneous occurrence of the events E and F is
P(E ∩ F) = P(E) . P(F/E), where P(F) ≠ 0
or
P(E ∩ F) = P(F) . P(F/F), where P(F) ≠ 0
This result is known as multiplication rule of probability.
Multiplication Theorem for More than Two Events: If F, F and G are three events of sample space,
then
𝐹 𝐺
𝑃(𝐸 ∩ 𝐹 ∩ 𝐺) = 𝑃(𝐸)𝑃 (𝐸) 𝑃(𝐸∩𝐹)

9 Independent Events: Two events E and F are said to be independent, if probability of


occurrence or non-occurrence of one of the events is not affected by that of the other. For any
two independent events E and F, we have the relation
(i) P(E/F)=P(E) and P(F/E)= P(F)
(ii) Two events E and F are said to be independent, if P(E ∩ F) = P(F) . P(F)
(iii) P(𝐴̅ ∩ 𝐵)= P(B)-P(A∩ 𝐵) where 𝐴̅ and B are independent events.
(iv)Three events E, F and G are said to be mutually independent, if
(a) P(E ∩ F) = P(E) . P(F)
(b) P(F ∩ G) = P(F) . P(G)
(c) P(E ∩ G) = P(E) . P(G)
(d)P(E ∩ F ∩ G) = P(E) . P(F) . P(G)
If atleast one of the above is not true for three given events, then we say that the events are not
independent.
Note: Independent and mutually exclusive events do not have the same meaning.

Q1. If P(A’) = 2/5 and P(B) = 1/5, find P(A∩B) if A and B are independent events.
2 3
Sol: P(A)=1- P(A’) ⇒ 𝑃(𝐴) = 1 − 5 ⇒ 𝑃(𝐴) = 5 For independents events
3 1 3
P(A∩B) = P(A) P(B) ⇒ P(A∩B) = 5 × 5 = 25
Q2. Given that the events A and B are such that P(A) = ½, P(AUB) = 3/5. Find P(B) if the
events A and B are (i) mutually exclusive (ii) independent.
Sol: [ Two events are called mutually exclusive if 𝐴 ∩ 𝐵 = ∅ 𝑜𝑟, 𝑃(𝐴 ∩ 𝐵) = 0]
3 1 3 1 1
(i) 𝑃(𝐴 ∪ 𝐵) = 𝑃(𝐴) + 𝑃(𝐵) − 𝑃(𝐴 ∩ 𝐵) ⇒ 5 = 2 + 𝑃(𝐵) − 0 ⇒ 𝑃(𝐵) = 5 − 2 = 10
(ii) 𝑃(𝐴 ∪ 𝐵) = 𝑃(𝐴) + 𝑃(𝐵) − 𝑃(𝐴 ∩ 𝐵) ⇒ 𝑃(𝐴 ∪ 𝐵) = 𝑃(𝐴) + 𝑃(𝐵) − 𝑃(𝐴). 𝑃(𝐵)
3 1 1 3 1 1 1 1 1
⇒ = + 𝑃(𝐵) − . 𝑃(𝐵) ⇒ − = . 𝑃(𝐵) ⇒ = . 𝑃(𝐵) ⇒ 𝑃(𝐵) =
5 2 2 5 2 2 10 2 5
Q3. If A and B are two independent events, then the probability of occurrence of at least
one of A and B is given by 1 – P ( ) P ( ).
Ans. P (at least one of A and B) = P (A∪B)
= P (A) + P (B) – P (A∩B) = P (A) + P (B) – P (A). P (B) [∵ 𝐴 𝑎𝑛𝑑 𝐵 𝑎𝑟𝑒 𝑖𝑛𝑑𝑒𝑝𝑒𝑛𝑑𝑒𝑛𝑡 ]
= P (A) + P (B)( 1–P(A)) = P (A) + P (B) P ( )
= 1 – P (𝐴 ) + P (B) P ( )
,
= 1 – P (𝐴, ) (1–P(B))
= 1 – P ( ) P (( ))
Q4. A Box of oranges is inspected by examining three randomly selected oranges drawn
without replacement. If all the three oranges are good, the box is approved for sale, other

66
wise, it is rejected. Find the probability that a box containing 15 oranges out of which 12
are good and 3 are bad ones will be approved for sale.
Ans.
Required probability=p(1st is good)p(2nd is good)p(3rd is good)
12 11 10 44
= × × =
15 14 13 91
Q5. A fair coin and an unbiased die are tossed. Let A be the event head appear on the coin
and B be the event 3 on the die.
Check weather A and B are independent events or not.

Ans.
A : Head appear on the coin
B : 3 appear on the dice
A={(H,1), (H,2), (H,3), (H,4),(H,5)(H,6)}
B= {(H,3), (T,3)} Then A∩ 𝐵 = {(𝐻, 3)}
1 6 2 1
P(A∩ 𝐵) = 12 P(A)= 12 and P (B)=12 = 6
1 1 1
P(A)X P(B)= 2 𝑋 6 =12 = P (A∩ 𝐵)
Hence A and B are independent
Q6. Probability of solving specific problem independently by A and B
𝟏 𝟏
are 𝟐and 𝟑 respectively of both try to solve the problem independently, find the probability
that
(i) the problem is solved
(ii) Exactly one of them solves the problem.
Ans. E1 : A solves the problem E2 : B solves the problem
P (E1) = ½ and P (E2) = 1/3
(i) P (the problem is solved)
= 1 – P (the problem is not solved)= 1- P(𝐸1 , ) P(𝐸2 , )
1 1
= 1-[(1- -2) (1 − 3)] =1-1/3 = 2/3
(ii) P (Exactly one of them solves the problem)= P(𝐸1 )P(𝐸2 , ) + 𝑃(𝐸2 )P(𝐸1 , )
=P(𝐸1 )(1 − 𝑃(𝐸2 )) + 𝑃(𝐸2 )(1 − 𝑃(𝐸1 )
1 1 1 1 1
= 2 (1 − 3) + 3 (1 − 2) = 2

Baye’s Theorem and Probability Distributions


Partition of Sample Space: A set of events E1, E2,…,En is said to represent a partition of the sample
space S, if it satisfies the following conditions:
(i) Ei ∩ Ej = Φ; i ≠ j; i, j = 1, 2, …….. n
(ii) E1 ∪ E2 ∪ …… ∪ En = S
(iii) P(Ei) > 0, ∀ i = 1, 2,…, n

Theorem of Total Probability: Let events E1, E2, …, En form a partition of the sample space S of an
𝐴
experiment. If A is any event associated with sample space S, then P(A)=P(𝐸1 )𝑃 ( )+
𝐸1
𝐴 𝐴 𝐴
𝑃(𝐸2 ) ( ) + ⋯ … … … … + 𝑃(𝐸𝑛 )𝑃 ( ) = ∑𝑛𝑗=1 𝑃(𝐸𝑗 )𝑃( )
𝐸 2 𝐸 𝑛 𝐸 𝐽

67
Baye’s Theorem: If E1, E2,…,En are n non-empty events which constitute a partition of sample
space S, i.e. (a) E1, E2,…, En are pairwise disjoint (b)E1 ∪ E2 ∪ ……. ∪ En = S and(c) P(Ei) > 0, for all i
= 1, 2, ….. n Also, let A be any non-zero event, the probability then
𝐴
𝑃(𝐸𝑖 )𝑃(𝐸 )
𝑖
P(𝐸𝑖 /𝐴) = 𝑛 𝐴 for all i=1,2,3…n
∑𝑖=1 𝑃(𝐸𝑖 )𝑃( )
𝐸 𝑖

Q1. Given three identical boxes I, II and III each containing two coins. In box-I both coins are gold
coins, in box-II, both are silver coins and in the box-III, there is one gold and one silver coin. A
person chooses a box at random and takes out a coin. If the coin is of gold, what is the probability
that the other coin in the box is also of gold.
Ans. let E1, E2 and E3 be the events that boxes I, II and III are chosen.
P (E1) = P (E2) = P (E3) = 1/3
let A be the event the coin drawn is of gold.
2
P(A/𝐸1 ) = 2 = 1 P(A/𝐸2 ) = 0 𝑎𝑛𝑑 P(A/𝐸3 ) = 1/2

1
3
×1
=1 1 1 1 = 2/3
3
×1+0×3+2×3

Q2. Suppose that the reliability of a HIV test is specified as follows of people having HIV, 90% of
the test detect the disease but 10% go undetected of people free of HIV, 99% of the test are
Judged HIV – tive but 1% are diagnosed as showing HIV +ive. From a large population of which
only 0.1% have HIV, one person is selected at random, given the HIV test, and the pathologist
reports him/her is HIV +tive what is the probability that the person actually has HIV.
Ans. let E denote the event that the person selected is actually having HIV and A the event that
the person’s HIV test is diagnosed as + tive.
let not having HIV.
.1
P(E)= .1% = 100= .001 P(𝐸 , )= 1- P(E)= .999
90
P(A/E)= 90% = = .90 P(A/𝐸 , )=1% = .01
100
𝐴
𝑃(𝐸)𝑃(𝐸)
P(E/A)= 𝐴 𝐴
𝑃(𝐸)𝑃(𝐸)+𝑃(𝐸 , )𝑃(𝐸,)
.001×.90 90 90 10
= = = = =.083
.001×.90+.999×.01 90+999 1089 121
Q3. In a factory which manufactures bolts, machines. A, B and C manufacture respectively 25%,
35% and 40% of the bolts. Of their output 5,4 and 2 percent are respectively defective bolts. A
bolt is drawn at random from the product and is found to be defective. What is the probability
that it is manufactured by the machine B.
Ans. let B1 = bolt is manufactures by A
B2 = bolt is manufactured by B B3 = bolt is manufactured by C
let E bolt is defective
P (B1) = 25% = 0.25 P (B2) = 0.35 P (B3) = 0.40

68
P ( E|B1) = 5% = 0.05 P (E|B2) = 0.04 P (E|B3) = 0.02
𝑃(𝐵2 )𝑃(𝐸/𝐵 2)
P(𝐵2/E)= 𝐸 𝐸
𝑃(𝐵1 )𝑃(𝐵 )+𝑃(𝐵2 )𝑃(𝐵 )+𝑃(𝐵3 )𝑃(𝐸/𝐵3 )
1 2

.35×.04
= = 140/(140+125+80)= 140/345= 28/69
.35×.04+.25×.05+.40×.02

Q4. A man is known to speak truth 3 out of 4 times. He throws a die and reports that it is a six.
Find the probability that it is actually a six.
Ans. let E be the event that the man reports that six occurs in the throwing of the dice and let
S1 be the event that six occurs and S2 be the event six does not occur.

1 3
𝑃(𝑆1 )𝑃(𝐸/𝑆1 ) × 3
P(𝑆1/E)= = 6 4
1 3 5 1 =
𝑃(𝑆1 )𝑃(𝐸/𝑆1 )+𝑃(𝑆2 )𝑃(𝐸/𝑆2 ) × + × 8
6 4 6 4
Q5. In answering a question on a multiple choice test a student either knows the answer or
3
guesses Let 4be the probability that he knows the answer and ¼ be the probability he guesses.
1
Assuming that a student who guesses at the answer will be correct with probability 4. What is the
probability that the student knows the answer given that he answered it correctly.
Ans. E1 : the student knows the answer
E2 : the student guesses the answer
A the answer is correct
P (A/E1) = 1, P (A/E2) = ¼
3
𝑃(𝐸1 )𝑃(𝐴/𝐸1 ) 1× 12
P(E1/A)= = 4
3 1 1 =
𝑃(𝐸1 )𝑃(𝐴/𝐸1 )+𝑃(𝐸2 )𝑃(𝐴/𝐸2 ) 1× + × 13
4 4 4

Q6. A card from a pack of 52 cards is lost. From the remaining cards of the pack, two cards are
drawn and are found to be both diamonds. Find the probability of the lost card being a diamond.
Ans. E1 : lost card is diamond
E2 : lost card is not diamond
let A : two cards drawn from the remaining pack are diamonds.

13 1 39 3
P(𝐸1 )= 52 = 4 P(𝐸2 )= 52 = 4
12𝐶2 12×11 13𝐶2 13×12
P (A/E1) = 51 = P (A/E2) = 51 =51×50
𝐶2 51×50 𝐶2

1×12×11
𝑃(𝐸1 )𝑃(𝐴/𝐸1 )
P(𝐸1 /A)= = 4×51×50
1×12×11 3×13×12
𝑃(𝐸1 )𝑃(𝐴/𝐸1 )+𝑃(𝐸2 )𝑃(𝐴/𝐸2 ) +
4×51×50 4×51×50
= 132/(132+468)= 132/600 = 11/50

Q7. Suppose a girl throws a die. If she gets a 5 or 6, she tosses a coin three times and notes the
number of heads. If she gets 1, 2, 3, 4, she tosses a coin once and notes whether a head or tail is
obtained. If she obtained exactly one had, what is the probability that she threw 1, 2, 3 or 4 with the
die?

69
Ans. E1 : 1, 2, 3, 4 is shown on dice
E2 : 5 or 6 is shown on dice
4 2 2 1
P (E1) = = and P(𝐸2 )= =
6 3 6 3
let A exactly one head shown up
1 3
P(A/𝐸1 ) = 2 and P(A/𝐸2 ) = 8

2 1
𝑃(𝐸1 )𝑃(𝐴/𝐸1 ) × 8
P(𝐸1 /A)= = 3 2
2 1 1 3 =
𝑃(𝐸1 )𝑃(𝐴/𝐸1 )+𝑃(𝐸2 )𝑃(𝐴/𝐸2 ) × + × 11
3 2 3 8
Random Variable: A random variable is a real-valued function, whose domain is the sample
space of a random experiment. Generally, it is denoted by capital letter X.
Note: More than one random variables can be defined in the same sample space.
Probability Distributions: The system in which the values of a random variable are given along
with their corresponding probabilities is called probability distribution.
Let X be a random variable which can take n values x1, x2,…, xn.
Let p1, p2,…, pn be the respective probabilities.
Then, a probability distribution table is given as follows
X 𝑋1 𝑋2 𝑋3 …….. 𝑋𝑛
P 𝑝1 𝑝2 𝑝3 ……… 𝑝𝑛

such that p1 + p2 + p3 +… + pn = 1
Note: If xi is one of the possible values of a random variable X, then statement X = xi is true only at
some point(s) of the sample space. Hence ,the probability that X takes value x, is always non-zero,
i.e. P(X = xi) ≠ 0

Q1. Find the probability distribution of the number of heads when three coins are tossed.
Sol: S = {HHH, HHT, HTH, THH, HTT, THT, TTH, TTT} → Total outcomes are 8.
Because number of heads may be 0, 1, 2 or 3 , therefore X = 0, 1, 2, 3
P(X=0) = means probability of not getting any head = P(TTT) = 1/8
P(X=1) = means probability of getting exactly one head = P(HTT, THT, TTH) = 3/8
P(X=2) = means probability of getting exactly two heads = P(HHT, HTH, THH) = 3/8
P(X=3) = means probability of getting three heads = P(HHH) = 1/8
Therefore, probability distribution is
X 0 1 2 3
P(X) 1/8 3/8 3/8 1/8
Q2. Two cards are drawn from a well shuffled deck of 52 cards. Find the probability distribution
of the number of aces if the cards are drawn (i) with replacement (ii) without replacement.
Sol : We have total ace cards = 4 and total non-ace cards = 48
Two cards are drawn, so number of aces may be 0, 1 or 2. Therefore, X = 0, 1, 2
(i) With replacement P(X=0) = means prob. of getting no ace card = P(both cards are non-
48 48 12 12 144
ace) = 52 × 52 = 13 × 13 = 169
P(X=1) = means prob. of getting exactly one ace card = P(ace and non-ace OR non-ace and
ace)
4 48 48 4 1 12 12 1 24
=52 × 52 + 52 × 52 = 13 × 13 + 13 × 13 = 169
4 4 1 1 1
P(X=2) = P(both cards are ace) = 52 × 52 = 13 × 13 = 169 .Therefore, probability distribution
is

70
X 0 1 2
P(X) 144/169 24/169 1/169
48 47 4 47 188 48
(ii) Without replacement P(X=0) = 52 × 51 = 13 × 17 = 221 [ OR we can use 52𝐶 2]
𝐶2
P(X=1) = P(ace then non-ace OR non-ace then ace)
4 48 48 4 1 16 16 1 32 4𝐶 1 ×48𝐶
= 52 × 51 + 52 × 51 = 13 × 17 + 13 × 17 = 221 [ OR we can use 1
]
52𝐶 2
4 3 1 1 1 4
P(X=2) = 52 × 51 = 13 × 17 = 221 [ OR we can use 52𝐶 2 ] .Therefore, probability distribution is
𝐶2

X 0 1 2
P(X) 188/221 32/221 1/221
Q3. The probability that X=x is given by
0.1 if x = 0
P(X = x) = { kx if x = 1 or 2
k(5 − x) if x = 3 or 4
0 otherwise
(a) Find the value of k
(b) What is the probability that x is at least 2? Exactly 2? And atmost 2?
Sol: According to given conditions, we put the values of x and prepare the following table.
{hint. Put x = 1 and 2 in kx and put x = 3 and 4 in k(5–x) }
X 0 1 2 3 4 otherwise
P(X) 0.1 k 2k 2k k 0
9 3
(a) ∵ ∑𝑝𝑖 = 1 ⇒ 0.1 + 𝑘 + 2𝑘 + 2𝑘 + 𝑘 + 0 = 1 ⇒ 6𝑘 + 0.1 = 1 ⇒ 6𝑘 = 0.9 ⇒ 𝑘 = =
60 20
(b) 𝑃( 𝑥 𝑖𝑠 𝑎𝑡 𝑙𝑒𝑎𝑠𝑡 2) = 𝑃( 𝑥 𝑖𝑠 2 𝑜𝑟 𝑚𝑜𝑟𝑒) = 𝑃( 𝑥 𝑖𝑠 2 𝑜𝑟 3 𝑜𝑟 4 𝑜𝑟 𝑜𝑡ℎ𝑒𝑟𝑤𝑖𝑠𝑒) = 2𝑘 + 2𝑘 +
3 3
𝑘 + 0 = 5𝑘 = 5 × 20 = 4
3 3
𝑃(𝑥 𝑖𝑠 𝑒𝑥𝑎𝑐𝑡𝑙𝑦 2) = 2𝑘 = 2 × 20 = 10
𝑃(𝑥 𝑖𝑠 𝑎𝑡𝑚𝑜𝑠𝑡 2) = 𝑃( 𝑥 𝑖𝑠 2 𝑜𝑟 𝑙𝑒𝑠𝑠) = 𝑃(𝑥 𝑖𝑠 0 𝑜𝑟 1 𝑜𝑟 2) = 0.1 + 𝑘 + 2𝑘 = 0.1 + 3𝑘
1 3 11
= +3× =
10 20 20

CASE STUDY BASED PROBLEMS

Q1. Let X denotes the number of colleges when you will apply for admission after your result of
class XII and P(X = x) denotes your probability of getting admission in x number of colleges. It is
given that

𝑘𝑥, 𝑖𝑓 𝑥 = 0 𝑜𝑟 1
𝑃(𝑋 = 𝑥) = { 2𝑘𝑥, 𝑖𝑓 𝑥 = 2 and P(X > 4) = 0, where 𝑘 is a constant.
𝑘(5 − 𝑥)𝑖𝑓 𝑥 = 3 𝑜𝑟 4

Based on the above information, answer the following:


(i) Find the value of k
Ans P(𝑋0 ) + P(𝑋1 ) + P(𝑋2 ) + P(𝑋3 ) + P(𝑋4 ) = 1
K× 0 +k× 1 + 2𝑘 × 2 + 𝑘(5 − 3) + 𝑘(5 − 4) = 1
8k=1 or k= 1/8
(ii) The probability that you will get admission in at least two college
Ans P(X≥ 2) = 1 − 𝑃(𝑋 < 2) = 1 − {𝑃(𝑋 = 0) + 𝑃(𝑋 = 1)}

71
1
= 1-{k× 0 + 𝑘 × 1) = 1 − 𝑘 = 1 − 8
Q2. In a village there are three Mohallas A, B and C . In A , 60% Farmers believe in new
technology of agriculture, while in B ,70% and in C , 80% . A farmer is selected at random from
village on the basis of above information , answer the following questions
(a) Find the conditional probability that a farmer believe in new technology if he belongs to
Mohalla A
(b) Find the total probability that a farmer believe in new technology of agriculture.
Ans (a) Let 𝐸1 = Selecting mohalla A A= farmer believe in new technology
60 3
P(A/𝐸1 )= 100 = 5
1 60 1 70 1 80 7
b) Required probability = 3 × 100 + 3 × 100 × 3 × 100 = 10
Q3. A Coach is training 3 players. He observes that the player A can hit a target 4 times in 5 shots,
Player B can hit 3 times in 4 shots and the player C can hit 2 times in 3 shots on the basis of
above information , answer the following
(a) What is the probability that ‘any two’ of A, B and C will hit
(b) What is the probability that at least one of A, B or C will hit the target ?
4 3 2
Ans : P(A)= 5 P(B)= 4 P(C)= 3
(a) P(any two will hit)= P(A)P(B)P(𝐶̅ ) + P(A)P(C)P(𝐵̅)+ P(C)P(B)P(𝐴̅)
4 3 1 4 2 1 1 3 2 1 2 1 13
= 5 × 4 × 3 + 5 × 3 × 4 + 5 × 4 × 3 = 5 + 15 + 10 = 30
4 3 2
(b) P(at least one of A, B or C will hit the target)= 1- P(𝐴̅) P(𝐵̅) P(𝐶̅ ) =1-(1--5) (1 − 4) (1 − 3)
1 1 1 1 59
= 1-5 × 4 × 3 = 1 − 60 = 60

Miscellaneous Exercise
Q1 Given that A and B are two independent events such that P(A) = 0.3 and P(B) = 0.5. Find
P(A/B). (C.B.S.E. 2019 C)
Solution: We have:
P(A)= 0.3 and P(B) = 0.5. Now P(A ∩ B) = P(A). P(B) [∵A and B are independent events]
= (0.3) (0.5) = 0.15.
Hence, P(A/B) = P(A∩B)/P(B)=0.15/.5 =0.3.
Q2. A bag contains 3 white and 2 red balls, another bag contains 4 white and 3 red balls. One ball
is drawn at random from each bag. Find the probability that the balls drawn are one white and
one red. (C.B.S.E. 2019 C)
Solution: Required. probability
3 3 2 4 17
= P(White, Red) + P (Red, White) =5 × 7 + 5 × 7 = 35

Q3. The probabilities of A, B and C solving a problem independently are 1/2,


1/3 and 1/4 respectively. If all the three try to solve the problem independently, find the
probability that the problem is solved. (C.B.S.E. 2019 C)
Solution:
Given: P(A) = 1/2, P(B) = 1/3 and P(C) = ¼
1 1 1 2 1 3
P(𝐴̅) = 1 − 2 = 2 , P(𝐵̅ ) = 1 − 3 = 3 P(𝐶̅ ) = 1 − 4 = 4
Probability that the problem is solved
= Probability that the problem is solved by at least one person
1 2 3 1 3
= 1- P(𝐴̅)P(𝐵̅)P(𝐶̅ ) = 1- 2 × 3 × 4 = 1 − 4 = 4

72
Q4 A die marked 1, 2, 3 in red and 4, 5, 6 in green is tossed. Let A be the event “number is even”
and B be the event “number is marked red”. Find whether the events A and B are independent or
not. {Delhi 2019)
Solution:
Here, A : number is even i.e.,
A = {2,4,6} and B : number is red i.e., B = {1,2,3} P(A) =1/2 P(B)= 1/2
P(A∩ 𝐵) = 𝑃(𝑛𝑢𝑚𝑏𝑒𝑟 𝑖𝑠 𝑒𝑣𝑒𝑛 𝑎𝑛𝑑 𝑟𝑒𝑑) = 1/6
1 1
1/6 ≠ 2 × 2 Thus, P(A ∩ B) ≠ P(A). P(B) hence the events A and B are not independent
Q5. 12 cards numbered 1 to 12 (one number on one card), are placed in a box and mixed up
thoroughly. Then a card is drawn at random from the box. If it is known that the number on the
drawn card is greater than 5, find the probability that the card bears an odd number.(Delhi 2019)
Solution:
Let the events be as :
A : Card bears an odd number.= {1,3,5,7,9,11}
B : Number on the card is greater than 5.= { 6,7,8,9,10,11,12} now, A∩B = {7, 9, 11}.
3 7 3
Hence, P(A/B) = P(A∩B)/P(B) = 12 ÷ 12 = 7
Q6. Two numbers are selected at random (without replacement) from first 7 natural numbers. If
X denotes the smaller of the two numbers obtained, find the probability distribution of X.
6 6 5 5 4 4 3 3
P(X=1) =7 = 21 P(X=2) =7 = 21 P(X=3) =7 = 21 P(X=4) =7 = 21
𝐶2 𝐶2 𝐶2 𝐶2
2 2 1 1
P(X=5) =7 = 21 P(X=6) =7 = 21
𝐶2 𝐶2
Xi 1 2 3 4 5 6
pi 6/21 5/21 4/21 3/21 2/21 1/21
Q7. There is a group of 50 people who are patriotic out of which 20 believe in non-violence. Two
persons are selected at random out of them, write the probability distribution for the selected
persons who are non-violent. Also find the mean of the distribution.
Solution:
Let ‘X’ be the number of non-violent persons. Here ‘X’ takes values 0, 1,2.
Thus no. of non-violent persons = 20 and no. of violent persons = 50 – 20 = 30.
30 29 87 30 20 20 30 120 20 19 38
P(X=0)=50 × 49 = 245 P(X=1)=50 × 49 + 50 × 49 = 245 P(X=2)50 × 49 = 245

Xi 0 1 2
pi 87/245 120/245 38/245
Q8. Two groups are competing for the positions of the Board of Directors of a corporation. The
probabilities that the first and second groups will win are 0.6 and 0.4 respectively. Further, if the
first group wins, the probability of introducing a new product is 0.7 and the corresponding
probability is 0.3 if the second group wins. Find the probability that the new product introduced
was by the second group.
(C.B.S.E. 2018 C)
Solution:
Let E1 = First group wins, E2 = Second group wins H = Introduction of new product.
P(E1) = 0.6, P(E2) = 0.4
P(H/E2) = 0.3 P(H/E1) = 0.7
𝑃(𝐸2 )𝑃(𝐻/𝐸2 ) .4×.3 2
Now, P(𝐸2 /H)= 𝑃(𝐸 )𝑃(𝐻/𝐸 )+𝑃(𝐸 )𝑃(𝐻/𝐸 ) = .4×.3+.6×.7 = 9
2 2 1 1

73
Q9. If A and B are two independent events such that:
P(𝐴̅∩B)=2/15 and P(A∩𝐵̅)=1/6, then find P(A) and P(B). Ans: P(A)=1/5 P(B)=1/6
Q10. Given that the events A and B are such that P(A) = 1/2, P (A ∪ B) = 3/5, and P(B) = p. Find p if
they are (i) mutually exclusive Ans :1/10 (ii) independent Ans: 1/5
Q11.Two numbers are selected at random from the integers 1 through 9. If the sum is even, find
the probability that both the numbers are odd. (Answer: 5/8).
Q12. A die is rolled. If the outcome is an odd number, what is the probability that it is prime? (Ans:
2/3).
Q13. A coin is tossed twice. If the outcome is at most one tail, what is the probability that both head
and tail have appeared? (Answer: 2/3).
Q14. An unbiased die is tossed twice. Find the probability of getting a 4, 5, 6 on the first toss and a
1, 2, 3, 4 on the second toss. (Answer: 1/3).
Q15. The probability that person A hits a target is 1/3 and the probability that person B hits it is
2/5. What is the probability that the target will be hit if both person A and person B shoot at
it? (Answer: 3/5).
Q16. Evaluate P(A ∪ B), if 2P(A) = P(B) = 513 and P(A/B) = 25 (C.B.S.E. 2018 C)
Q17. Out of 8 outstanding students of a school, in which there are 3 boys and 5 girls, a team of 4
students is to be selected for a quiz competition. Find the probability that 2 boys and 2 girls are
selected. (Outside Delhi 2019 Ans 3/7
CASE STUDY QUESTIONS:
Q1. Two groups Aggarwal and Malhotra are competing for the position on the board of directors
of a corporation. The probabilities that aggarwal group and malhotra group will win are
3 2
𝑎𝑛𝑑 5 𝑟𝑒𝑠𝑝𝑒𝑐𝑡𝑖𝑣𝑒𝑙𝑦 . 𝐹𝑢𝑟𝑡ℎ𝑒𝑟 𝑖𝑓 𝑎𝑔𝑔𝑎𝑟𝑤𝑎𝑙 𝑔𝑟𝑜𝑢𝑝 𝑤𝑖𝑛𝑠 the probability of introducing a new
5
7 3
product is 10 and the corresponding probability is 10 𝑖𝑓 𝑚𝑎𝑙hotra group wins
Based on the above information, answer the following:
27
(a)What is the total probability of introducing a new product? Ans= 50
7
(b)If new product is introduced, then find the probability that aggarwal group wins. Ans= 9
Q2. After observing attendance register of class XII , class teacher shri mishra comes on
conclusion that 30% students have 100% attendance and 70% students are irregular to attend
class .When he observed
Previous year result, he found that 70% of all students who have 100% attendance attains
distinction marks While 10% irregular students attain distinction marks. At the end of the year,
one student is chosen at Random from the class. Based on the above information, answer the
following:
(a)Find the conditional probability that selected student has distinction marks, given that the
7
student has 100% attendance. Ans = 10
(b)If in random selection, selected student has distinction marks. Find the probability that the
1
student Is irregular. Ans = 4

74
Sample Question Paper -1
CLASS: XII
Session: 2021-22
Mathematics (Code-041)
Term - 2
Time Allowed: 2 hours Maximum Marks: 40
General Instructions:
1. This question paper contains three sections – A, B and C. Each part is compulsory.
2. Section - A has 6 short answer type (SA1) questions of 2 marks each.
3. Section – B has 4 short answer type (SA2) questions of 3 marks each.
4. Section - C has 4 long answer type questions (LA) of 4 marks each.
5. There is an internal choice in some of the questions.
6. Q14 is a case-based problem having 2 sub parts of 2 marks each.

SECTION -A
1 Evaluate: 2
2 x dx
 ( x 2 + 1)( x 2 + 3)
OR
Evaluate:
. sec 4 x.dx
2 Find the area of the parallelogram whose diagonals are determined by the vectors 2
     
are 2 i − j + k , 3 i + 4 j − k
3 Find the degree and order of the following D.E. 2

2 2
 dy   dy 
y = x  + a 1 +  
 dx   dx 
4 A line makes an angle of π/4 with each of x-axis and y-axis. Find the angle 2
between this line and the z-axis
5 Find K if the following probability distribution is possible. 2
X 0 1 2 3
P (X) K K2 K 0.04

6 A die is thrown twice. Find the probability of getting a number 6 on the first 2
throw and a number greater than 4 on the second
SECTION – B
7 3x + 2 3
 ( x − 1)(2 x + 3)dx
Evaluate
8 Solve the following differential equation: ( x3 + y 3 ) dy − x 2 y dx = 0 . 3

OR

75
 dy 
. Solve sin −1  = x+ y
 dx 

    
9
( 
)
If vectors a , b , c satisfy the condition a + b + c =0 and  a = 1,  b = 4,  c = 3
  
2, then find the value of .b + b.c + c.a .
a
10 Find the shortest distance between the lines: 3
( ) ( ) ( ) (
r = i + j − k +  3 i − j and r = 4 i − k +  2 i + 3k . )
OR
Find the equation of the plane which contains line of intersection of planes
( )
r . i + 2 j + 3k − 4 = 0
,
(
r . 2i + j − k + 5 = 0 )
and which passes through the point (1,0,-2).

SECTION – C
11 Evaluate: ∫0
𝜋 𝑥 𝑑𝑥
𝑑𝑥 4
𝑎2 𝑐𝑜𝑠2 𝑥+𝑏 2 𝑠𝑖𝑛2 𝑥

12 Using integration find the area of the triangle bounded by the lines x+2y=2, y-x=1 4
and 2x+y=7.
OR
Find the area of the region bounded between the line x=4 and the parabola y2=
16x
13 Find the equation of the plane containing the lines r = (iˆ + ˆj ) +  (iˆ + 2 ˆj − kˆ) and 4

r = (iˆ + ˆj ) +  (iˆ + ˆj − 2kˆ) . Find the distance of this plane from the origin
14 CASE BASED /DATA BASED
Bag I contains 3 red and 4 black balls and Bag II contains 4 red and 5 black balls. One
ball is transferred from Bag I to Bag II and then a ball is drawn from Bag II . The ball
is so drawn is found to be red in colour.
Based on the given information answer the following
(i) Find the probability that the transferred ball is red. 2

(ii) Find the probability that the transferred ball is black. 2

76
MARKING SCHEME (SQP-1)
SECTION -A
1 2 x dx
.
( x + 1)( x 2 + 3)
2

dt
=
(t + 1)(t + 3)
1 dt 1 dt 1
=  − 
2 t +1 2 t + 3
Put x2 = t 2xdx = dt
1 1
= log t + 1 − t + 3 + c
2 2
1 t +1
= log +c
2 t +3 1
1 x +1
2
= log 2 +c
2 x +3
OR
. sec 4 x.dx Put t = tan x; dt = sec 2 x dx
1
=  sec 2 x(1 + tan 2 x)dx

=  (1 + t 2 )dt
t3
=t + +c 1
3
1
= tan x + tan 3 x + c
3
2 →   →   
d1 = 2 i − j + k and d 2 = 3 i + 4 j − k
  
i j k
1 → →   
Area of parallelogram = d1  d 2 = 2 − 1 1 = −3 i + 5 j + 11 k 1
2
3 4 −1

Correct answer 1
3 Order=1 Degree=4 1
1
4 𝜋 1+1
2
5 We have  P (Xi) = 1 1
K +k2+k +.04=1 or K2+2K +.96=0
or K = 0.4, -2.4  K≠ - 2.4  K= 0.4 1
6 2 1
Favourable cases are { 65, 66}  Probability = = 1 1
36 18
SECTION -B

77
7 3x + 2 A B 1
= +
( x − 1)(2 x + 3) x − 1 2 x + 3

3x+2=A(2x+3)+B(x-1)  A=1 B=1


dx dx 1
I=  +
x −1 2x + 3
1
 log x − 1 + log 2 x + 3 + c 1
2
8 dy x2 y
=
dx x 3 + y 3
1
dy dv
Let y=vx  =v+x
dx dx 1
dv v dv v
v+x = x = −v
dx 1 + v 3
dx 1 + v 3
dv v3 1 + v3 1 1
x
dx
=−
1+ v 3
  v 3
dx = −  dx   3 dv +  dv = − log x + c
x v
−2 1
v 1 − x2 y
 + v = − log x + c  − 2 + v = − log x + c  + = − log x + c
−2 2v 2y2 x
OR

 dy 
Given D.E. sin −1   = x + y
 dx 
dy
= sin( x + y )
dx
dv dv
− 1 = sin v  = 1 + sin v
dx dx
1
dv (1 − sin v)dv
 = dx  = dx
1 + sin v 1 − sin 2 v
(
sec 2 v − sec v tan v dv = dx )
Integrating both sides, 1

 (sec )
v − sec v tan v dv =  dx
2

tan v − sec v + c = x 1
Gen.Sol. x = tan(x + y ) − sec(x + y ) + c
9 For squaring the given equation and calculation 1+1
Getting – 21/2 1
10 → ^ ^ ^ → ^ ^
a1 = i + j − k b1 = 3 i − j
→ ^ ^ → ^ ^
a2 = 4 i − k b2 = 2 i + 3 k
→ →
1/2
→ →
S.D = (a − a1) • (b  b2)
2 1
→ →
(b  b )
1 2

78
^ ^ ^

→ →
i j k
^ ^ → → ^ ^ ^

a2− a1 = 3 i− j and b1 b2 = 3 − 1 0 = −3 i − 9 j + 2 k 1/2


2 0 3
→ → 1
b1 b2 = 94 .
^ ^ ^ ^ ^
S.D = (3 i− j ) • ( − 3 i − 9 j + 2 k ) / 94
OR 1
Equations of plane in Cartesian form are x+2y+3z-4=0 and 2x+y-z+5=0
Required equation is (x+2y+3z-4)+k(2x+y-z+5)=0----------(1) 1/2
It passes through (1,0,-2)
=> (1+0-6-4) + k ( 2+0+2+5)=0
 k =1 1/2
 Equation of plane is (x+2y+3z-4) + 1(2x+y-z+5)=0
 3x+3y+2z+1=0 1
 Vector form.
SECTION -C
11 𝜋 𝑥 𝑑𝑥 𝜋 (𝜋−𝑥) 𝑑𝑥
I=: ∫0 𝑎2𝑐𝑜𝑠2 𝑥+𝑏2𝑠𝑖𝑛2𝑥 𝑑𝑥 = ∫0 𝑎2 𝑐𝑜𝑠2 (𝜋−𝑥)+𝑏2𝑠𝑖𝑛2(𝜋−𝑥) 𝑑𝑥 1
𝜋 𝑥 𝑑𝑥
1
2I=: 𝜋 ∫0 𝑎2 𝑐𝑜𝑠2 𝑥+𝑏2𝑠𝑖𝑛2 𝑥 𝑑𝑥 1
On calculation
𝜋2
Answer : 2𝑎𝑏 1
12 by the lines x+2y=2, y-x=1 and
2x+y=7.
Let AB: y-x=1, BC: 2x+y=7
CA: x+2y=2
On solving every two equations, we
1
get
The vertices A (0,1),B(2,3) and C(4,-1)
3 1 3
Area of ABC =  xdy −  xdy −  xdy
−1 −1 1
BC AC AB

7− y
3
I1 =    dy
−1 
2 
BC

3 1
1 y2 
= 7 y − 
2 2  −1

= 3 + 1 − 9 − 1
7 1
2 4
= 14 − 2 = 12
1
1
1
y  2
I2 =  (2 − 2 y )dy − = 2 y  − 2
1
−1 
−1
AC
 2  −1
= 21 + 1 − 1 − 1 = 4 1
3
3
y  2

 −  y 1 = (9 − 1) − (3 − 1)
1
I3 =  ( y − 1)dy = 
3

1
2 1 2
AB

4−2 = 2
Area of ABC = 12 − 4 − 2 = 6 sq.units

79
1
OR 1
Area of the region bounded by parabola y2 = 1
16x and line x = 4 given by 1
4
Area = 2 ∫0 𝑦 𝑑𝑥 (parabola symmetric to x -axis)
4
= 2 ∫0 4 √𝑥𝑑𝑥 = 128/3 sq. unit

13 r = (iˆ + ˆj ) +  (iˆ + 2 ˆj − kˆ) (i)

r = (iˆ + ˆj ) +  (iˆ + ˆj − 2kˆ) (ii)

 Line (i) passes through (1, 1, 0) and has dr’s 1, 2, –1 and line (ii) passes
1
through (1,1,0) and has dr’s –1, 1, –2  required plane contains (i) and
(ii)
1
 The plane is parallel to the vectors

b = iˆ + 2 ˆj − kˆ and c = − iˆ + ˆj − 2kˆ
 Required plane is parallel to the vector

iˆ ˆj kˆ 1

b c = 1 2 −1 = iˆ(−4 + 1) − ˆj (−2 − 1) + kˆ(1 + 2) = −3iˆ + 3 ˆj + 3 ˆj


−1 1 −2

 The reqd. plane passing through (1, 1, 0) and parallel to the vectors

b and c is (r − a ).(b  c ) = 0 1

[(r − (iˆ + j )].[−3iˆ + 3 ˆj + 3kˆ] = 0 or r .(−3iˆ + 3 ˆj + 3kˆ) = 0

or r .(−iˆ + ˆj + kˆ) = 0 its Cartesian form is −x + y + z = 0


Its distance from origin = 0
14 E1- Red ball is drawn from Bag I and E2 - Black ball is drawn from Bag
II 1
E=drawn ball is red 1
P(E1)=3/7 P(E2)=4/7

P(E/E1) = 1/2 P(E/E2) = 2/5


1
𝑃(𝐸/𝐸2 ) 𝑃(𝐸2 )
(i) P(E2/E) = 𝐸 =16/31
𝑃( )𝑃(𝐸1 )+ 𝑃(𝐸/𝐸2 ) 𝑃(𝐸2 )
𝐸1 1
𝑃(𝐸/𝐸1 ) 𝑃(𝐸1 )
(ii) P(E1/E) = 𝐸 =15/31
𝑃( )𝑃(𝐸1 )+ 𝑃(𝐸/𝐸2 ) 𝑃(𝐸2 )
𝐸1

80
Sample Question Paper -2
CLASS: XII
Session: 2021-22
Mathematics (Code-041)
Term - 2
Time Allowed: 2 hours Maximum Marks: 40
General Instructions:
1. This question paper contains three sections – A, B and C. Each part is compulsory.
2. Section - A has 6 short answer type (SA1) questions of 2 marks each.
3. Section – B has 4 short answer type (SA2) questions of 3 marks each.
4. Section - C has 4 long answer type questions (LA) of 4 marks each.
5. There is an internal choice in some of the questions.
6. Q14 is a case-based problem having 2 sub parts of 2 marks each.

SECTION –A
1 2

Evaluate:  sec x −1 dx
OR

dx
x 1/ 2
+ x1 / 3
Evaluate:

2 Find the sum of order and degree of the differential equation.. 2


dy d 2 y
+ +y=0
dx dx 2
3 If a = 2iˆ + 2 ˆj + 3kˆ, b = −iˆ + 2 ˆj + kˆ and c = 3iˆ + ˆj are such that a +  b is 2
perpendicular to c . Find the value of 
4 Using direction ratio’s, show that the points (2, 3, 4), (–1, –2, 1) and (5, 8, 7) are 2
collinear
5 IF P (B) = 5/11, P (A) = 6/11 and P (AB) =7/11 find P (B/A) 2
6 12 cards, numbered 1 to 12 are placed in a box, mixed up thoroughly and then a card is 2
drawn at random from the box. If it is known that the number on the drawn card is more
than 3, find the probability that it is even number.
SECTION –B
7 3
4− x 3
Evaluate :  dx
1 x + 4− x

8 3
dy
Solve : sin x + y cos x = 2 sin 2 x cos x.
dx
OR

81
Solvethe DifferentialEquation x(1 + y 2 )dx − y (1 + x 2 )dy = 0 , giventhat y = 0 when x = 1

9 3
If a  b = c  d and a  c = b  d , show that a − d is parallel to b − c where a  d and
b c
10 Find the shortest distance between the lines l1 and l2 given by the following : 3
x −3 y −5 z −7 x +1 y +1 z +1
l1 : = = , l2 = = =
1 −2 1 7 −6 1
OR
x + 2 y +1 z − 3
Find the point on the line = = at a distance 3 2 from the point (1, 2, 3).
3 2 2
SECTION –C
11 4
1
 sin x + sin 2 x dx
Evaluate:

12 Find the area of the region (x, y ): x 2 + y 2  1  x + y 4


OR
Using integration, Find the area bounded by the parabola x2 = 4y and the line x=4y-2
13 From the point P (1,2,4), a perpendicular segment is drawn on the plane 2x +y – 2z + 4
3= 0 find its equation and its length. Also, find the co-ordinates of the foot of the
perpendicular
14 CASE BASED/DATA BASED 4
Coloured balls are distributed in three bags as shown in the following table
Colour of the ball
Bag
Red White Black
I 1 2 3

II 2 4 1

III 4 5 3
A bag is selected at random and two balls are randomly drawn from the selected bag.
They happen to be black and red.
Based on the given information, answer the following questions.

(i)What is the probability of one black and one red ball if they are drawn from bag III.

(ii)What is the probability that the balls are drawn from bag I

82
MARKING SCHEME (SQP-2)
SECTION –A
1
1 − cos x
 sec x − 1 dx = 
cos x
dx

(1 − cos x)(1 + cos x) 1 − cos 2 x


= dx =  dx
cos x(1 + cos x) cos x + cos 2 x
1
sin xdx − dt
= =
cos x + cos 2 x t2 +t
2 2
dt  1  1 1
= − = − log  t +  +  t +  −   + c
2
 1 1
2
 2  2 2
t +  −  
 2 2
 1  1
= − log  t +  + t 2 + t + c1 = − log  cos x +  + cos 2 x + cos x + c 1
 2  2
Put t = cosx
(- ) dt = sinx dx ½
OR

dx 6t 5 dt Put x = t6 1
 x1 / 2 + x1 / 3 =  t 3 + t 2
dx = 6t5dt
t 3 dt (t 3 + 1) − 1
= 6 = 6 dt x1/2= t3 & x1/3 = t2
t +1 (t + 1)
1/2
(t 3 + 1) 1 1
= 6 − dt = 6  (t 2 − t + 1 − ).dt
(t + 1) (t + 1) 1+ t
 t3 t2 
= 6 − + t − log 1 + t  + c = 2 x − 3 x1 / 3 + 6 x1 / 6 − 6 log x1 / 6 + 1 + c
3 2 
2 2+1=3 1+1
3
a + b = (2iˆ + 2 ˆj + 3kˆ) +  (−iˆ + 2 ˆj + kˆ) ½

= (2 −  )iˆ + (2 + 2 ) ˆj + (3 +  ) kˆ
½
Now (a + b ) is perpendicular to c.
 (a + b ) . c = 0

 [(2 −  )iˆ + (2 + 2 ) ˆj + (3 +  ) kˆ].(3iˆ + ˆj ) = 0


1
or (2 −  ).3 + (2 + 2 ).1 + (3 +  ).0 = 0
or 8 −  = 0 or  = 8
4 Let the points be A (2, 3, 4), B (–1, –2, 1) and C (5, 8, 7)

DR ' s of AB = 2 + 1,3 + 2, 4 − 1 i.e. 3,5,3 ½


½

83
DR ' s of BC = 5 + 1,8 + 2,7 − 1 i.e. 6,10,6
3 5 3 1
As = =  AB | | BC, B is common
6 10 6
Hence A, B, C, are collinear
5 P(A B) = P(A)+ P(B)- P(AB) 1
= 6/11 + 5/11 - 7/11 = 4/11
P(B/A) = P (AB) = 4/11 = 2
P (A) 6/11 3 1

6 S = {1, 2, 3, 4, ......................12}
A = {2, 4, 6, 8, 10, 12}
B = {4, 5, 6, 7, 8, 9, 10, 11, 12}
and A  B = {4, 6, 8, 10, 12}
6 1 9 3 5
 P( A) = = ; P( B) = = ; P( A  B) = 1
12 2 12 4 12
P( A  B) 5/12 5 1
P( A / B) = = =
P( B) 9 /12 9
SECTION –B
7 3
4− x
Evaluate :  dx
1 x + 4− x
4− x
3
I = dx − − − − − − − (1)
1 x + 4− x
3
4 − (1 + 3 − x) 1
= 
1 (1 + 3 − x) + 4 − (1 + 3 − x)
dx

3
x
I = dx − − − − − − − −(2)
1 4− x + x

1
Adding (1) & (2)
4− x + x
3
2I =  dx
1 4− x + x
3
1
2 I =  dx = x  = 3 − 1 = 2
3
1
1

I =1

84
8

dy
Given D.E. : sin x + y cos x = 2 sin 2 x cos x.(divide by sin x)
dx
dy cos x dy
+ y = 2 sin x. cos x ( Form : + py = q ) 1
dx sin x dx
cos x
 sin x dx = log sin x  I .F . = e = sin x
log sin x

Gen.Sol : 1

y. sin x =  2 sin x. cos x. sin xdx + c Put v = sin x ; dv = cos x.dx

= 2  v 2 dv + c
v3 1
=2 +c
3
2 sin 3 x
y sin x = +c
3
OR
given : x(1 + y )dx − y (1 + x )dy = 0
2 2 1

x(1 + y 2 )dx = y (1 + x 2 )dy


x y
dx = dy
(1 + x )
2
(1 + y 2 )
Integrate both sides, 1
x y
 (1 + x 2 ) dx =  (1 + y 2 ) dy
1 1
log 1 + x 2 = log 1 + y 2 + log c
2 2
1 1
log 1 + x 2 − log 1 + y 2 = log c
2 2
1 1+ x 2
log = log c
2 1 + y2
1 + x2 1
log = log c 2
1+ y 2

1 + x2
=C
1 + y2
General Solution : 1 + x 2 = C 1 + y 2 ( )
when x = 1 y = 0; C = 2
Particular Solution : 1 + x 2 = 2 1 + y 2 ( )
9. Two non-zero vectors are parallel if and only if their vector product is a zero vector. 1
Here (a − d )  (b − c ) = (a  b − a  c − d  b + d  c )
= c d −b d +b d −c d = 0 (Using given results) 1
Hence ( a − d ) is parallel to (b-c) 1
10 Writing the given equations in vector form

85
r = (3iˆ + 5 ˆj + 7kˆ) +  (iˆ − 2 ˆj + kˆ)
r = (−iˆ − ˆj − kˆ) +  (7iˆ − 6 ˆj + kˆ) 1

here a1 = 3iˆ + 5 ˆj + 7kˆ, a = −iˆ − ˆj − kˆ


a2 − a1 = −4iˆ − 6 ˆj − 8kˆ
b1 = iˆ − 2 ˆj + kˆ , b2 = 7iˆ − 6 ˆj 6 + kˆ ½
iˆ ˆj kˆ
b1  b2 = 1 −2 1
7 −6 1

= iˆ(−2 + 6) − ˆj (1 − 7) + kˆ(−6 + 14) = 4iˆ + 6 ˆj + 8kˆ


| b1  b2 |= (4) 2 + (6) 2 + (8) 2 = 116 = 2 29 ½

(a2 − a1 ).(b1  b2 )
S .D. =
| b1  b2 )
(−4iˆ − 6 ˆj − 8kˆ).(4iˆ + 6 ˆj + 8kˆ)
= 1
2 29
−16 − 36 − 64 116
= = =| −2 29 | = 2 29 Units 1
2 29 2 29
OR
x + 2 y +1 z − 3
Any point P on the line = = = (say)
3 2 2
is (3 − 2, 2 − 1, 2 + 3)
Let Q be the point (1, 2, 3)
1
It is given PQ = 3 2 or PQ2 = 18.

or (3 − 2 − 1) 2 + (2 − 1 − 2) 2 + (2 y + 3 − 3) 2 = 18

or (3 − 3) 2 + (2 − 3) 2 + (2 ) 2 = 18


1
or 17 − 30 = 0  (17 − 30) = 0
2

30
 =0 or =
17
 90 60 60 
 Coordinates of P are (−2, −1,3) or  − 2, − 1, + 3 
 17 17 17 
 56 43 111 
i.e. (−2, −1,3) or  , , 
 17 17 17 
SECTION –C

86
11

1 dx
 sin x + sin 2 x dx =  sin x(1 + 2 cos x) 1
sin xdx sin xdx
= =
sin x(1 + 2 cos x)
2
(1 − cos x)(1 + 2 cos x)
2

Put t = cosx ; (-)dt=sinxdx 1


− dt dt
= = −
(1 − t )(1 + 2t )
2
(1 + t )(1 − t )(1 + 2t )
1
1 1 1 1 4 1
=  dt −  dt −  dt
2 1+ t 6 1− t 3 1 + 2t
1 1 2
= log 1 + t + log 1 − t − log 1 + 2t + c
2 6 3
Partial Fraction:
A B C 1
1= + +
1 + t 1 − t 1 + 2t
1 = A(1 − t )(1 + 2t ) + B(1 + t )(1 + 2t ) + c(1 − t )(1 + t )
Solving : A = (−1 / 2) , B = (1 / 6) , C = (4 / 3)
12 x2 + y2 = 1 is a circle with center (0, 0) and radius = 1; Point of intersection B(0, 1) and A(1, 0). 1
1 1
Required area = 
0
1 − x 2 −  (1 − x )dx
0
B (0,1)

1
 x 1 − x 2 1 −1   x2 
1

=  
+ sin x −  x + 
   1
 2 2 0 2 0
  1
= − 1 −  A(1,0)
4  2 1
 1
=  −  Sq. Units
 4 2
OR
1

1
 x+2
2 2
 x2 
Reqd area =    dx −   4  dx
−1 
4  −1 
2 2
 1  x2  x   1  x3 
=    +  −   
 4  2  2  −1  4  3   −1 1
15 3
= −
8 4
9 1
= sq units
8 1

87
13 Equation of the ⊥ drawn from the point to the plane 2 x + y − 2 z + 3 = 0 is
x −1 y − 2 z − 4
= = = r ( say ) [ ⊥ will be parallel to normal]
2 1 −2
Any point Q on this perpendicular is
(2r + 1, r + 2, −2r + 4)
 It lies on the plane 2 x + y − 2 z + 3 = 0
 2(2r + 1) + ( r + 2) − 2( −2r + 4) + 3 = 0 1

 9r = 1 or r = 1 1
9
2 1 −2 
 foot of ⊥ Q is  + 1 , +2 , + 4  or
9 9 9 
 11 19 34 
 , ,  and the length of the 1
9 9 9 
2 2 2
 11   19   34 
Perpendicular segment PQ =  − 1 +  − 2  +  − 4  Units
1
9   2   9 
4 1 4 1 1
= + + = = Units.
81 81 81 9 3
14 Let E1 : Bag I is selected ; E2 : Bag II is selected 1
Let E3 : Bag III is selected and A = A black ball and a red ball is drawn
1
P( E1 ) = P( E2 ) = P( E3 ) =
3
1 3 3 1
P( A / E1 ) = = = 1
6
C2 15 5
2 1 2
P( A / E2 ) = 7 =
C2 21
1
43 43 2
P( A / E3 ) = 12 = =
C2 66 11
Using Baye’s Theorem 1
1 1 1

P( E1 / A ) = 3 5 = 5
1 1 1 2 1 2 1 2 2
 +  +  + +
3 5 3 21 3 11 5 21 11
231
=
551

88
Sample Question Paper: 03
CLASS: XII
Session: 2021-22
Mathematics (Code-041)
Term-2
Time Allowed: 2 hours Maximum Marks: 40
General Instruction:
1. This question paper contains three sections – A, B and C. Each part is compulsory.
2. Section – A has 6 short answer type (SA1) questions of 2 marks each.
3. Section – B has 4 short answer type (SA2) questions of 3 marks each.
4. Section – C has 4 long answer type questions (LA) of 4 marks each.
5. There is an internal choice in some of the questions.
6. Q14 is a case-based problem having 2 sub parts of 2 marks each.

SECTION – A
1. 2𝑥−1
Find ∫ 2𝑥+3 𝑑𝑥 2
OR
𝑠𝑖𝑛3 𝑥+𝑐𝑜𝑠3 𝑥
Find ∫ 𝑑𝑥
𝑠𝑖𝑛2 𝑥 𝑐𝑜𝑠2 𝑥

2. Find the order and the degree of the differential equation 2


2 2 2
𝑑𝑦 𝑑 𝑦
[1 + ( ) ] = 2
𝑑𝑥 𝑑𝑥
3. ̂ ⃗ ̂
If 𝑎 = 𝑖̂ + 𝑗̂ + 2𝑘 and 𝑏 = 2𝑖̂ + 𝑗̂ − 2𝑘, then find the unit vector in the 2
direction of 2𝑎 − 𝑏⃗.
4. Find |𝑎| and |𝑏⃗|, if |𝑎| = 2|𝑏⃗| and (𝑎 + 𝑏⃗). (𝑎 − 𝑏⃗) = 12 2
5. If A and B are two events such that P (A) =1/2, P (B) =1/3 and 2
P(A/B)=1/4, then find P(𝐴’∩ 𝐵′).
6. Two cards are drawn successively with replacement from a well-shuffled 2
deck of 52 cards. Find the probability distribution of the number of aces.
SECTION – B
7. Find: ∫ 𝑒 2𝑥 (
1−𝑠𝑖𝑛2𝑥
) 𝑑𝑥 3
1−𝑐𝑜𝑠2𝑥
8. Find the particular solution of the differential equation 3
(1 + 𝑥 2 )𝑑𝑦 + 2𝑥𝑦 𝑑𝑥 = 𝑠𝑒𝑐 2 𝑥 𝑑𝑥
𝑦 = 1 when 𝑥 = 0.
OR
𝑑𝑦 𝑦2
Show the differential equation = 𝑥𝑦−𝑥 2 is homogeneous and also
𝑑𝑥
solve it.
9. Three vectors 𝑎 , 𝑏⃗ and 𝑐 satisfy the condition 𝑎 + 𝑏⃗ + 𝑐=0. Evaluate the 3
quantity 𝜇 = 𝑎. 𝑏⃗ + 𝑏⃗. 𝑐 + 𝑐. 𝑎 , if 𝑎 , 𝑏⃗ and 𝑐 are unit vectors.
10. Find the equation of the plane passing through the points (2, 1,0), (3,-2,- 3
2) and (3,1,7).
OR

89
Show that lines 𝑟 = (𝑖̂ + 𝑗̂ − 𝑘̂) + 𝜆(3𝑖̂ − 𝑗̂) and 𝑟 = (4𝑖̂ − 𝑗̂) + 𝜇(2𝑖̂ +
3𝑘̂) intersect each other. Find their point of intersection.
SECTION – C
𝜋 𝑥
11. Evaluate ∫0 1+sin 𝑥 𝑑𝑥 4

12. Find the area bounded between the curve 𝑦 2 = 4𝑥, line 𝑥 + 𝑦 = 3 and 4
y-axis.
OR
Find the area of the region in the first quadrant enclosed by the x-axis ,
the line y=x, and the circle 𝑥 2 + 𝑦 2 = 32.
13. Find the image of the point having position vector 𝑖̂ + 3𝑗̂ + 4𝑘̂ in the 4
plane 𝑟. (2𝑖̂ − 𝑗̂ + 𝑘̂) + 3 = 0.
14. CASE-BASED/DATA-BASED
The reliability of a COVID PCR test is specified as follows :
Of people having COVID, 90% of the test detect the disease but 10% go
undetected. Of people free of COVID , 99% of the test are judged COVID
negative but 1% are diagnosed as showing COVID positive. From a large
population of which only 0.1% have COVID, one person is selected
random, given the COVID PCR test, and the pathologist reports him/her
as COVID positive.
Based on the given information , answer the following questions
(i)What are the probabilities of the ‘person to be tested as COVID 2
positive’ given that (a) ‘he is actually having COVID’ and (b) ‘he is
actually not having COVID’?
(ii)What is the probability that the ‘person is actually having COVID 2
given that ‘he is tested as COVID positive’?

Marking Scheme: (SQP-3)


CLASS: XII
Session: 2021-22
Mathematics (Code-041)
Term-2

SECTION – A
1. 2𝑥 + 3 − 3 − 1 1 1/2+1/
𝐼=∫ 𝑑𝑥 = ∫ 1𝑑𝑥 − 4 ∫ 𝑑𝑥
2𝑥 + 3 2𝑥 + 3 2
log (2𝑥+3)
=𝑥−4 + 𝑐 = 𝑥 − 2 log(2𝑥 + 3) + 𝑐
2
1
OR
𝑠𝑖𝑛3 𝑥 𝑐𝑜𝑠3 𝑥 𝑠𝑖𝑛𝑥 𝑐𝑜𝑠𝑥
I= ∫(𝑠𝑖𝑛2 𝑥𝑐𝑜𝑠2𝑥 + 𝑠𝑖𝑛2 𝑥𝑐𝑜𝑠2 𝑥)𝑑𝑥= ∫( 𝑐𝑜𝑠2 𝑥 + 𝑠𝑖𝑛2 𝑥)𝑑𝑥
= ∫ 𝑠𝑒𝑐𝑥 𝑡𝑎𝑛𝑥 𝑑𝑥 + ∫ cot 𝑥 𝑐𝑜𝑠𝑒𝑐𝑥 𝑑𝑥=𝑠𝑒𝑐𝑥 − 𝑐𝑜𝑠𝑒𝑐𝑥 + 𝑐 1/2+1/
2
1/2+1/
2

90
2. Order=2 and degree =1 and the required sum=3 1+1/2
+1/2
3. Finding 2𝑎 − 𝑏⃗ = 𝑗̂ + 6𝑘̂ 1

2𝑎⃗−𝑏 ̂
𝑗̂ +6𝑘
Then required unit vector = |2𝑎⃗−𝑏⃗|=
√37 1
4. For calculation 1
For finding |𝑏⃗| = 2 For finding |𝑎| = 4 ½
1/2
5. P(A∩B)=P(A/B)P(B)=(1/4)(1/3)=1/12 1
P(𝐴’∩ 𝐵′)= 1-P(A∪ 𝐵)=1-[P(A)+P(B)-P(A∩B)]=1-[1/2+1/3-1/12]=1/4 1
6. Let the number of aces is a random variable denoted by X. ½
So X takes the values 0,1 or 2.
P(X=0)=144/169 ; P(X=1)=24/169; P(X=2)=1/169 ½+1/2
+1/2
SECTION – B
7. 1−2𝑠𝑖𝑛𝑥𝑐𝑜𝑠𝑥 1
I=∫ 𝑒 ( 2𝑠𝑖𝑛2 𝑥 ) 𝑑𝑥 = 2 ∫ 𝑒 2𝑥 (𝑐𝑜𝑠𝑒𝑐 2 𝑥 − 2𝑐𝑜𝑡𝑥)𝑑𝑥
2𝑥 1
By using integration by parts
1 1
=2 [−𝑒 2𝑥 𝑐𝑜𝑡𝑥 + ∫ 2𝑒 2𝑥 𝑐𝑜𝑡𝑥 𝑑𝑥] − ∫ 𝑒 2𝑥 𝑐𝑜𝑡𝑥 𝑑𝑥
==−
𝑒 2𝑥
𝑐𝑜𝑡𝑥 + 𝑐 1
2
8. 𝑑𝑦
Given diff equation is a linear diff equation of the form 𝑑𝑥 + 𝑃𝑦 = 𝑄
2𝑥 𝑠𝑒𝑐 2 𝑥 1
Where P=1+𝑥 2 and Q= 1+𝑥 2
For finding IF=1+x2 1/2
𝑡𝑎𝑛𝑥 𝑐
For getting General solution 𝑦 = 1+𝑥 2 + 1+𝑥 2 1
𝑡𝑎𝑛𝑥 1
For getting Particular solution 𝑦 = 1+𝑥 2 + 1+𝑥 2 1/2
OR ½
For showing this equation as homogeneous diff equation. 1+1/2
𝑑𝑣 𝑣2
Putting y=vx and getting the equation 𝑣 + 𝑥 𝑑𝑥 = 𝑣−1
1
After calculation getting final solution 𝑦 = 𝑥𝑙𝑜𝑔𝑦 + 𝑥𝑐
9. Since 𝑎 + 𝑏⃗ + 𝑐 = 0 , then we have 𝑎. (𝑎 + 𝑏⃗ + 𝑐 ) = 0 ½
½
By solving this , get 𝑎. 𝑏⃗ + 𝑎. 𝑐 = −1,
1
Similarly, 𝑎. 𝑏⃗ + 𝑏⃗. 𝑐 = −1, 𝑎. 𝑐 + 𝑏⃗. 𝑐 = −1 1
Adding above three equations , 2(𝑎. 𝑏⃗ + 𝑏⃗. 𝑐 + 𝑐. 𝑎)=-3 µ = −3/2
10. For the formula of finding the equation of the plane passing through three 1
non collinear points
For Putting the Values (2,1,0), (3,-2,-2) and (3,1,7) and calculation 1
For correct equation of the plane 7𝑥 + 3𝑦 − 𝑧 = 17 1
OR
These lines will intersect , if
(𝑖̂ + 𝑗̂ + 𝑘̂) + 𝜆(3𝑖̂ − 𝑗̂) = (4𝑖̂ − 𝑘̂) + µ(2𝑖̂ + 3𝑘̂ ) 1
On equating the coefficients of 𝑖̂, 𝑗̂ 𝑎𝑛𝑑 𝑘̂
We get , 𝜆=1 ad µ=0 1
On putting the value of 𝜆 in given equation of line (i), we get 𝑟 =
(𝑖̂ + 𝑗̂ − 𝑘̂) + 1(3𝑖̂ − 𝑗̂) = 4𝑖̂ + 0𝑗̂ − 𝑘̂ Required point = (4,0,-1)
1

91
SECTION – C
𝜋 𝑥
11. I=∫0 1+sin 𝑥 𝑑𝑥 ….(1)
𝜋 𝜋−𝑥 𝜋 𝜋−𝑥
I=∫0 1+sin(𝜋−𝑥) 𝑑𝑥 = ∫0 1+sin 𝑥 𝑑𝑥 ……(2)
1
Adding equations (i) and (ii), we get 1
𝜋 1
2I=π∫0 1+sin 𝑥 𝑑𝑥 1+1/2
Calculation and applying limits Final answer = π 1/2
12. For finding the point of intersection of the 1
given line and parabola
(1,2) and (9,-6) 1
For Correct figure
2
Required area= ∫0 𝑥(𝑝𝑎𝑟𝑎𝑏𝑜𝑙𝑎)𝑑𝑦 +
3 1
∫2 𝑥(𝑙𝑖𝑛𝑒)𝑑𝑦
2 𝑦2 3
= ∫0 ( 4 ) 𝑑𝑦 + ∫2 (3 − 𝑦)𝑑𝑦
Calculation 1
Final answer = 7/6 sq units
OR
For finding the point of intersection of 1/2
the given line and circle
(4,4) and (4,-4)
For Correct figure 1
4
Required area=∫0 𝑦(𝑙𝑖𝑛𝑒)𝑑𝑥 +
4√2
∫4 𝑦(𝑐𝑖𝑟𝑐𝑙𝑒)𝑑𝑥 1
4 4√2 1
=∫0 𝑥𝑑𝑥 + ∫4 √32 − 𝑥 2 𝑑𝑥
after calculation , answer=4π sq units 1/2
13. Let the given point P (𝑖̂ + 3𝑗̂ + 4𝑘̂) and Q be the image of P in the given
plane.Then PQ is the normal to the plane.
Equation of PQ is 𝑟 = (𝑖̂ + 3𝑗̂ + 4𝑘̂) + 𝜆(2𝑖̂ − 𝑗̂ + 𝑘̂)
Since Q lies on the line PQ, the position vector of Q can be expressed
(1 + 2𝜆)𝑖̂ + (3 − 𝜆)𝑗̂ + (4 + 𝜆)𝑘̂ 1
If R is the mid point of PQ , the position vector of R is
[(1 + 2𝜆)𝑖̂ + (3 − 𝜆)𝑗̂ + (4 + 𝜆)𝑘̂]) + [𝑖̂ + 3𝑗̂ + 4𝑘̂] 1
2
R lies on the given plane 𝑟. (2𝑖̂ − 𝑗̂ + 𝑘̂) + 3 = 0
1
Then the position vector of R will satisfy this equation , then we get 𝜆=-2
Hence the position vector of image Q is −3𝑖̂ + 5𝑗̂ + 2𝑘̂ 1
14. CASE-BASED/DATA-BASED
(i)E1=people having COVID and E2=people free of COVID
A=Person to be tested as COVID positive
(a) P(A/E1)=90%=0.9 1
(b) P(A/E2)=1%=0.01 1
𝐸1 𝑃(𝐸1 )𝑃(
𝐴
𝐸1
) 0.1%×90% 2
(ii) 𝑃 ( 𝐴 ) = 𝐴 𝐴 =0.1%×90%+99.9%×1% = 0.083
𝑃(𝐸1 )𝑃( )+𝑃(𝐸2 )𝑃( )
𝐸1 𝐸2

92
Sample Question Paper: 04
CLASS: XII
Session: 2021-22
Mathematics (Code-041)
Term-2
Time Allowed: 2 hours Maximum Marks: 40
General Instruction:
1. This question paper contains three sections – A, B and C. Each part is compulsory.
2. Section – A has 6 short answer type (SA1) questions of 2 marks each.
3. Section – B has 4 short answer type (SA2) questions of 3 marks each.
4. Section – C has 4 long answer type questions (LA) of 4 marks each.
5. There is an internal choice in some of the questions.
6. Q14 is a case-based problem having 2 sub parts of 2 marks each.

SECTION – A
1. Find
𝑥4
∫ 𝑥 2 +1 𝑑𝑥 2
OR
sin 𝑥−cos 𝑥
Find ∫ 𝑑𝑥
√1+sin 2𝑥
2. Find the order and the degree of the differential equation 2
𝑑2 𝑦 𝑑𝑦 2 𝑑2 𝑦
+ 3 ( ) = 𝑥 2 𝑙𝑜𝑔 ( )
𝑑𝑥 2 𝑑𝑥 𝑑𝑥 2
3. Find the projection of the vector 𝑖̂ + 3𝑗̂ + 7𝑘̂ on the vector 2𝑖̂ − 3𝑗̂ + 6𝑘̂ . 2
4. Find the direction cosines of the line passing through two points (2,-4,5) 2
and (0,1,-1). Also find the equation of line.
5. Two events E and F are independent. If P(E)=0.3 and P(E∪F)=0.5, then 2
find the value of P(E/F)-P(F/E).
6. A coin is biased so that the head is 3 times as likely to occur as tail. If the 2
coin is tossed twice, then find the probability distribution of number of
tails.
SECTION – B
7. 2𝑥+1
Find : ∫ √3+2𝑥−𝑥 2 𝑑𝑥 3
8. Find the particular solution of the differential equation 3
𝑑𝑦 𝑦 𝑦
− + cosec( ) = 0
𝑑𝑥 𝑥 𝑥
𝑦 = 0 𝑤ℎ𝑒𝑛 𝑥 = 1
OR
Solve the differential equation
𝑑𝑦 2𝑥𝑦
− = 𝑥2 + 2
𝑑𝑥 1 + 𝑥 2
9. Find a unit vector perpendicular to both the vectors 𝑎 and 𝑏⃗, where 3
𝑎 = 𝑖̂ − 7𝑗̂ + 7𝑘̂ and 𝑏⃗ = 3𝑖̂ − 2𝑗̂ + 2𝑘̂.
10. Find the shortest distance between the following lines : 3
𝑟 = 𝑖̂ + 2𝑗̂ − 4𝑘̂ + 𝜆(2𝑖̂ + 3𝑗̂ + 6𝑘̂)
𝑟 = 3𝑖̂ + 3𝑗̂ − 5𝑘̂ + µ(4𝑖̂ + 6𝑗̂ + 12𝑘̂)

93
OR
Find the distance of the point P(-2,-4,7) from the point of intersection Q
of the line 𝑟 = (3𝑖̂ − 2𝑗̂ + 6𝑘̂) + 𝜆(2𝑖̂ − 𝑗̂ + 2𝑘̂ ) and the plane 𝑟. (𝑖̂ − 𝑗̂ +
𝑘̂) = 6.
SECTION – C
11. Evaluate
𝜋 𝑥 𝑡𝑎𝑛𝑥
∫0 sec 𝑥+tan 𝑥 𝑑𝑥 4
12. Draw a rough sketch of the curve 𝑦 = |𝑥 − 3|. Find the area under the 4
curve and lines 𝑥 = 0 𝑎𝑛𝑑 𝑥 = 4.
OR
Using integration find the area of region bounded by the triangle whose
vertices are (1,0), (2,2) and (3,1).
13. 𝑥−2 𝑦−4 𝑧−6 𝑥+1 𝑦+3
Prove that the lines 1 = 4 = 7 𝑎𝑛𝑑 3 = 5 = 7 are coplanar.
𝑧+5 4
Also, find the plane containing these lines.
14. CASE-BASED/DATA-BASED
There is a top class plastic factory in a city. In this factory, if a machine
is correctly set up, it produces only 90% acceptable items. If it is
incorrectly setup, it produces only 40% acceptable items. Past
experience shows that 80% of the set ups are correctly done. One day,
an inspection team visited the factory and gave his report to the owner
of the factory.
Based on the given information, answer the following questions
(i)What is the probability of the event that the machine produces 2 2
acceptable items given that (a) when the machine is correctly setup and
(b) when the machine is incorrectly setup?
(ii)If after a certain setup, the machine produces 2 acceptable items, 2
Find the probability that the machine is correctly setup.

Marking Scheme: (SQP-4)


CLASS: XII
Session: 2021-22
Mathematics (Code-041)
Term-2

SECTION – A
1. 𝑥4 − 1 + 1 (𝑥 2 − 1)(𝑥 2 + 1) 1 1/2+1/2
𝐼=∫ 𝑑𝑥 = ∫{ + }𝑑𝑥
𝑥2 + 1 𝑥2 + 1 𝑥2 + 1
𝑑𝑥 𝑥3
= ∫ 𝑥 2 𝑑𝑥 − ∫ 𝑑𝑥 + ∫ 2 = − 𝑥 + tan−1 𝑥 + 𝑐 1/2+1/2
𝑥 +1 3
OR
𝑠𝑖𝑛𝑥−𝑐𝑜𝑠𝑥 𝑠𝑖𝑛𝑥−𝑐𝑜𝑠𝑥 1/2+1/2
I=∫ √𝑠𝑖𝑛2 𝑑𝑥=∫ 𝑠𝑖𝑛𝑥+𝑐𝑜𝑠𝑥 𝑑𝑥
𝑥+𝑐𝑜𝑠2 𝑥+2𝑠𝑖𝑛𝑥𝑐𝑜𝑠𝑥
By using substitution method , getting the answer
I=-log(sinx+cosx)+c 1
2. Order=2 and degree is not defined. 1+1

94
3. Getting 𝑎. 𝑏⃗ = 35 1

𝑎⃗.𝑏 35
⃗⃗⃗ 𝑜𝑛 𝑏⃗ = |𝑏⃗| = 7 = 5
Projection of 𝑎 1
4. Direction ratios =0-2 , 1+4 , -1-5 i.e. -2,5,-6
Direction cosines=
−2
,
5
,
−6 1
√65 √65 √65
𝑥−2 𝑦+4 𝑧−5 1
Equation of line : −2 = 5 = −6
5. Let P(F)=x, then finding x=2/7 by using P(E∪F)=P(E)+P(F)-P(E∩F)= 1
P(E)+P(F)-P(E)P(F)
𝑃(𝐸∩𝐹) 𝑃(𝐹∩𝐸)
P(E/F)-P(F/E)= 𝑃(𝐹) − 𝑃(𝐸) =P(E)-P(F)=3/10-2/7=1/70 1
6. Let X be the random variable which denotes the number of tails ½
when a biased coin is tossed twice. So x may have values 0,1 or 2.
P(H)=3/4 and P(T)= 1/4
P(X=0)=9/16 ; P(X=1)=3/8 ; P(X=2)=1/16 ½+1/2+1/2
SECTION – B
7. 𝑑
Let 2𝑥 + 1 = 𝐴 𝑑𝑥 (3 + 2𝑥 − 𝑥 2 ) + 𝐵 Finding A=-1 and B=3 1
(2−2𝑥) 1
Getting I=− ∫ √3+2𝑥−𝑥 2 𝑑𝑥 + 3 ∫ √3+2𝑥−𝑥 2 𝑑𝑥 and calculation ½+1
Final answer=−2√3 + 2𝑥 −
𝑥−1
𝑥 2 +3sin−1 ( 2 ) +𝑐 1/2

8. Given differential equation is a homogeneous differential equation.


𝑑𝑦 𝑑𝑣
On putting 𝑦 = 𝑣𝑥 and 𝑑𝑥 = 𝑣 + 𝑥 𝑑𝑥 ,we get the equation
𝑑𝑥 1
sin 𝑣𝑑𝑣 = −
𝑥
𝑦
On solving this equation , we finally get cos (𝑥 ) = 𝑙𝑜𝑔|𝑥| − 𝑐 1
By putting y=0 and x=0 , we get the particular solution
𝑦
cos ( ) = 𝑙𝑜𝑔|𝑥| + 1
𝑥 1
OR
𝑑𝑦
This is the linear differential equation of the form 𝑑𝑥 + 𝑃𝑦 = 𝑄
−2𝑥
Where P=1+𝑥 2 and Q=𝑥 2 + 2
1 1
For finding IF=𝑥 2 +1 ½
Solution of given diff equation
1 1
𝑦. (𝑥 2 +1) = ∫(𝑥 2 + 2) (𝑥 2 +1) 𝑑𝑥 + 𝑐 and calculation 1
Final answer , 𝑦 = 𝑥(𝑥 2 + 1) + tan−1 𝑥. (𝑥 2 + 1) + 𝑐(𝑥 2 + 1) ½
9. For 𝑎 × 𝑏⃗ = 0𝑖̂ + 19𝑗̂ + 19𝑘̂ 1
For |𝑎 × 𝑏⃗| = 19√2 1
1 1 1
For required unit vector = 𝑗̂ + 𝑘̂
√2 √2
10. On showing that given lines are parallel lines. 1
𝑎1 = 2𝑖̂ + 𝑗̂ − 𝑘̂ 𝑎𝑛𝑑 𝑏⃗ = 2𝑖̂ + 3𝑗̂ + 6𝑘̂
𝑎2 − ⃗⃗⃗⃗
⃗⃗⃗⃗
Applying the formula of finding distance between parallel lines and ½+1
√293 ½
calculation, Required distance = 7

95
OR
𝑥−3 𝑦+2 𝑧−6
Cartesian equation of the line is 2 = −1 = 2 = 𝜆(𝑠𝑎𝑦) ½
½
A general point Q on this line is (2𝜆+3, -𝜆-2, 2𝜆+6)
1/2
Cartesian equation the plane is 𝑥 − 𝑦 + 𝑧 = 6
1
If the point Q lies on this plane then we get 𝜆=-1
Then the point Q =(1,-1,4)
½
Required Distance PQ=3√3units.
SECTION – C
𝜋 𝑥 𝑡𝑎𝑛𝑥
11. I=∫0 sec 𝑥+tan 𝑥 𝑑𝑥 ……..(i)
𝜋 (𝜋−𝑥) tan (𝜋−𝑥) 𝜋 (𝜋−𝑥) 𝑡𝑎𝑛𝑥
I= ∫0 sec(𝜋−𝑥)+tan(𝜋−𝑥) 𝑑𝑥=∫0 sec 𝑥+tan 𝑥 𝑑𝑥………(ii)(using property) 1
𝜋 𝑡𝑎𝑛𝑥
Adding (i) and (ii), 2I=π∫0 sec 𝑥+tan 𝑥 𝑑𝑥 1
1
For calculation and applying the limits
𝜋 1
Final answer = 2 (𝜋 − 2)
12. For the correct rough sketch. 1
It is an absolute function. Therefore it makes
two straight lines, which is defines as 𝑦 =
𝑥 − 3, 𝑥 ≥ 3
{
3−𝑥, 𝑥 <3
3
The area of bounded region =∫0 (3 − 𝑥)𝑑𝑥 + 1
4 1
∫3 (𝑥− 3)𝑑𝑥
1
Solving the above integration. Required area = 5 sq units
OR
Let A(1,0), B(2,2), and C (3,1) be the
vertices of a triangle ABC.
AB : 𝑦 = 2(𝑥 − 1) ; BC: y=4-x ; CA: y=(x-
1
1)/2
2
Required Area = ∫1 2(𝑥 − 1)𝑑𝑥 + 3×(½)
3 3 𝑥−1
∫2 (4− 𝑥)𝑑𝑥 − ∫1 2 𝑑𝑥 1
After calculation , final answer=3/2 sq units ½
13. For the using co planarity condition (determinant formula) 1
−1 − 2 −3 − 4 −5 − 6 1
Det 1 4 7 =0
3 5 7 1
Again using the determinant formula of plane containing two lines ,
we get the equation of the plane x-2y+z=o 1
14. CASE-BASED/DATA-BASED
(i)A= Machine produces 2 acceptable items
B1=Event of correct setup B2=Event of incorrect setup
(a) P(A/B1)=0.9×0.9=0.81 1
(b) P(A/B2)=0.4×0.4=0.16 1
(ii)P(B1/A)=81/85=0.95( by using Bayes’ Theorem and calculation) 2

96
Sample Question Paper -5
CLASS: XII
Session: 2021-22
Mathematics (Code-041)
Term - 2
Time Allowed: 2 hours Maximum Marks: 40
General Instructions:
1. This question paper contains three sections – A, B and C. Each part is compulsory.
2. Section - A has 6 short answer type (SA1) questions of 2 marks each.
3. Section – B has 4 short answer type (SA2) questions of 3 marks each.
4. Section - C has 4 long answer type questions (LA) of 4 marks each.
5. There is an internal choice in some of the questions.
6. Q14 is a case-based problem having 2 sub parts of 2 marks each.
SECTION – A
3𝑎𝑥
Find ∫ 𝑏2+𝑐 2𝑥 2 𝑑𝑥 2
1.
OR
1
Evaluate ∫ 𝑒 𝑥 (log 𝑥 + 𝑥 2 ) dx

Write the order and the degree of the following differential equation: 2
2. 𝑑2𝑦
2
𝑑𝑦 2 𝑑𝑦
( 2 ) + ( ) = 𝑥 𝑠𝑖𝑛 ( )
𝑑𝑥 𝑑𝑥 𝑑𝑥
̂ ̂
Find 𝜆 𝑎𝑛𝑑 𝜇, 𝑖𝑓 (2𝑖̂ + 6𝑗̂ + 27𝑘) Χ (𝑖̂ + 𝜆𝑗̂ + 𝜇𝑘) = 0 2
3.

Find the coordinate of the point, where the line


𝑥+2
=
𝑦−5
=
𝑧+1
cuts the yz- 2
1 3 5
4.
plane.
The random variable X has a probability distribution P(X) of the following form, 2
5. where k is some number:
𝑘, 𝑖𝑓 𝑥 = 0
2𝑘, 𝑖𝑓 𝑥 = 1
𝑃(𝑋) = {
3𝑘, 𝑖𝑓 𝑥 = 2
0, 𝑖𝑓 𝑜𝑡ℎ𝑒𝑟𝑤𝑖𝑠𝑒
(𝑎) Determine the value of k. (𝑏) Find P(x≥2)
A and B are events such that P(A) = 0.4, P(B) = 0.3 and P(A∪ 𝐵) = 0.5, then find 2
6. P(B’∩ 𝐴).

SECTION – B
Find: ∫
𝑥 3 +𝑥+1
𝑑𝑥 3
7. 𝑥 2 −1

Find the general solution of the following differential equation : 3


8. 𝑑𝑦 2
𝑥 log 𝑥 𝑑𝑥 + 𝑦 = 𝑥 log 𝑥 .
𝑂𝑅
Find the particular solution of the differential equation

97
𝑑𝑦 𝑦 𝜋
𝑥 = 𝑦 − 𝑥 𝑡𝑎𝑛 ( ) , 𝑔𝑖𝑣𝑒𝑛 𝑡ℎ𝑎𝑡 𝑦 = 𝑎𝑡 𝑥 = 1
𝑑𝑥 𝑥 4

Find the vector of magnitude 6, which is perpendicular to both the vectors 2𝑖̂ − 3
9. 𝑗̂ + 2𝑘̂ and 4𝑖̂ − 𝑗̂ + 3𝑘̂ .

Find the coordinates of the point where the line through (3, – 4, – 5) and (2, –3, 3
10. 1) crosses the plane 2x + y + z – 7 = 0.
OR
Find the equation of the plane which is perpendicular to the plane 5x + 3y - 6z +
8 = 0 and which contains the line of intersection of the planes x + 2y + 3z – 4 = 0
and 2x + y – z + 5 = 0.

SECTION – C
8 √10−𝑥 4
Find ∫2 √𝑥+ √10−𝑥 𝑑𝑥
11.

and the 4
3 𝑥2
Find the area of the region included between the parabola 𝑦 =
12. 4
line 3x – 2y + 12 = 0.
OR
Find the area of a minor segment of the circle x2 + y2 = a2 cut off by the line
x = a/2 .
Find the distance and the foot of perpendicular from the point (1, 3/2, 2) to 4
13. the plane 2x – 2y + 4z + 5 = 0.

14.

CASE-BASED/DATA-BASED
In answering a question on a multiple-choice test for class XII, a student either
knows the answer or guesses. Let 3/5 be the probability that he knows the
answer and 2/5 be the probability that he guesses. Assume that a student who
guesses at the answer will be correct with probability 1/3. Let E1, E2, E be the
events that the student knows the answer, guesses the answer and answers
correctly respectively.
Based on the above information, answer the following
1. Value of P(E | E1) is 2

2. What is the probability that the student knows the answer given that 2
he answered it correctly?

98
MARKING SCHEME (SQP-5)
Section A
1. Put b2+c2x2 =t ⇒ 2𝑐 𝑥 𝑑𝑥 = 𝑑𝑡 2
1
3𝑎 𝑑𝑡
=2𝑐 2 ∫ 𝑡 𝑜𝑛 𝑠𝑜𝑙𝑣𝑖𝑛𝑔
3𝑎 1
= 2𝑐 2 log 𝑒 (𝑏 2 + 𝑐 2 𝑥 2 ) + 𝑐
OR
1 1 1
Can be written as ∫ 𝑒 {(log 𝑒 𝑥 − 𝑥) + (𝑥 + 𝑥 2 )} 𝑑𝑥
𝑥 1
1
=𝑒 𝑥 (log 𝑒 𝑥 − 𝑥)+c using ∫ 𝑒 𝑥 {𝑓(𝑥) + 𝑓′(𝑥)}𝑑𝑥
1
2. Order=2 , Degree= not defined 1 +1
3. (6𝜇 − 27)𝑖̂ + (−2𝜇 + 27)𝑗̂ + (2𝜆 − 6)𝑘̂) = 0 1
Getting 𝜆 = 3 , 𝜇 = 27⁄2 1
4. 𝑥+2
Let 1 = 3 = 5 = 𝜆,
𝑦−5 𝑧+1 ½
½
General point on line (𝜆 − 2, 3𝜆 + 5, 5𝜆 − 1)
½
at YZ plane 𝜆 − 2 = 0, 𝜆 = 2
½
therefore point is ( 0,11,9)
5. (𝑖) ∑ 𝑝(𝑥𝑖 ) = 0 ⇒ 𝑘 + 2𝑘 + 3𝑘 + 0 = 0 ⇒ 𝑘 =
1
1
6
1
(ii) P(x≥2) = P(x=2) +P(x>2)= 3k+0= 3/6=1/2
6. P(A∩ 𝐵) = 𝑃(𝐴) + 𝑃(𝐵) − 𝑃(𝐴 ∪ 𝐵) = 0.4 + 0.3 − 0.5 = 0.2 1
P(B’∩ 𝐴) = 𝑃(𝐴) − 𝑃(𝐴 ∩ 𝐵) = 0.4 − 0.2 = 0.2 1
Section B
7. 3
𝑥 +𝑥+1 2𝑥 + 1 2𝑥 1 1
2
=𝑥+ 2 =𝑥+ 2 + 2
𝑥 −1 𝑥 −1 𝑥 −1 𝑥 −1
2𝑥 1
Now ∫ (𝑥 + 𝑥 2 −1 + 𝑥 2 −1) 𝑑𝑥 1
𝑥2 1 𝑥−1 1
= + log 𝑒 (𝑥 2 − 1) + log 𝑒 ⌈ ⌉+𝑐
2 2 𝑥+1
8. 𝑑𝑦 𝑦 2
+ 𝑥𝑙𝑜𝑔 𝑥 = 𝑥 2 . it is linear diff. equation in dy/dx + Py = Q ½
𝑑𝑥
½
Where P= 1/ x logx and Q = 2/x2
1 1
∫ 𝑑𝑥
I.F. = 𝑒 𝑥 𝑙𝑜𝑔𝑥 = 𝑒 log 𝑡 = 𝑡 = 𝑙𝑜𝑔𝑥 (after put logx = t)
2
Solution y . logx = ∫ log 𝑥 . 𝑥 2 𝑑𝑥 solving by parts 1
−2
ylogx = (𝑙𝑜𝑔𝑥 + 1) + 𝑐
𝑥
OR ½
𝑑𝑦
∴ 𝑑𝑥
𝑦 𝑦
= 𝑥 − tan (𝑥 ) it is homogeneous diff. equation. ½
𝑑𝑦 𝑑𝑣 𝑑𝑣 ½
Let y= vx ⇒ =𝑣+𝑥 Now diff. equation ⇒ 𝑣 + 𝑥 𝑑𝑥 = 𝑣 − tan 𝑣
𝑑𝑥 𝑑𝑥
𝑑𝑥
⇒ 𝑐𝑜𝑡𝑣 𝑑𝑣 = ⇒ log 𝑠𝑖𝑛𝑣 = −𝑙𝑜𝑔𝑥 + 𝑙𝑜𝑔𝑐 1
𝑥
𝑦
⇒ 𝑥 𝑠𝑖𝑛 𝑥 = 𝑐 getting c = 1/√2 after putting x=1, y = 𝜋/4
½
𝑦 1
Hence particular solution is 𝑥 𝑠𝑖𝑛 𝑥 =
√2
9. 𝑖̂ 𝑗̂ 𝑘̂ 2
Let 𝑐 = 𝑎 × 𝑏⃗ = |2 −1 2| = −𝑖̂ + 2𝑗̂ + 2𝑘̂
4 −1 3

99
𝑐 6
Now 6 𝑐̂ = 6 |𝑐| = 3 (−𝑖̂ + 2𝑗̂ + 2𝑘̂) 1
10. Equation of line through (3,-4,-5) and (2,-3,1) is 1
𝑥−3 𝑦+4 𝑧+5
= = ½
−1 1 6
Any point on line is (– λ + 3, λ – 4, 6 λ – 5). This point lies on plane 2x+y+z = 7 1
therefore, ½
2 (–λ + 3) + (λ – 4) + (6 λ – 5) – 7 = 0, i.e. λ =2
Hence the required point is (1, – 2, 7).
OR
Let equation of plane passing through the line of intersection of two given 1
plane are (x + 2y + 3z – 4 )+λ( 2x + y – z + 5) = 0.
Or (1+2λ)x +(2+λ)y + (3- λ)+(-4+5 λ)=0 ….. 1
This plane perpendicular to 5x+3y-6z+8=0 1
So, 5 (1+2λ) +3(2+λ)- 6(3- λ)=0 or λ=7/19
Now equation of required plane 33x+45y+50z-41=0 1
Section C
11. 8
I= ∫2
√10−𝑥
𝑑𝑥 …… (1) ½
√𝑥+ √10−𝑥
𝑏 𝑏
1
Apply property ∫𝑎 𝑓(𝑥)𝑑𝑥 = ∫𝑎 𝑓(𝑎 + 𝑏 − 𝑥)𝑑𝑥
8 √10−(2+8−𝑥) 8 √𝑥 1
We get 𝐼 = ∫2 𝑑𝑥 = ∫2 𝑥+ 𝑑𝑥 …(2)
√2+8−𝑥+√10−(2+8−𝑥) √ √10−𝑥

On adding
8 √𝑥+ √10−𝑥
(1) and (2) , 2𝐼 = ∫2 𝑥+ 𝑑𝑥 = [𝑥]82 = 6 1
√ √10−𝑥 ½
⇒𝐼=3
12. draw figure and getting point of intersection 1
A(-2,3) and B(4,12)
4 3𝑥+12
required area=∫−2 ( 2 ) − (
3𝑥 2
) 𝑑𝑥 1
4
getting area = 27 sq unit 2
OR
Drawing fig. and 1
correct shade
𝑎
Required area = 2 × ∫𝑎/2 √𝑎2 − 𝑥 2 𝑑𝑥 1
𝜋
Getting area = 𝑎2 ( 3 −
√3
) 𝑠𝑞 𝑢𝑛𝑖𝑡 2
4

13. Find the distance and the foot of perpendicular from the
point (1, 3/2, 2) to the plane 2x – 2y + 4z + 5 = 0. ½
Equation of AB line which is normal to given plane and
passing through A(1, 3/2,2)
𝑥 − 1 𝑦 − 3/2 𝑧−2
= = =𝜆
2 −2 4 1
3
General point on line AB = 2𝜆 + 1, −2𝜆 + 2 , 4𝜆 + 2
It will satisfy the equation of plane
3 ½ +1
Therefore 2(2𝜆 + 1) − 2 ( −2𝜆 + 2) + 4( 4𝜆 + 2) + 5 = 0 ⇒ 𝜆 = −1/2 ½+½
Foot of perpendicular B( 0,5/2, 0) and AB = √6
14. 1. 1 2. 9/11 2 +2

100

You might also like